考研论坛

 
查看: 10212|回复: 51

[学习资料] 阅读40分高分研究_完整版_Free

[复制链接]

0

主题

78

帖子

104

积分

一般战友

Rank: 2

精华
2
威望
46
K币
58 元
注册时间
2010-7-2
发表于 2010-8-2 10:07 | 显示全部楼层 |阅读模式
阅读40分高分研究
       考研与高考、各类出国考试(TOEFL、GRE、GMAT等)被认为是改变中国学生人生命运的三大考试。英语一直是筛选学生人选硕士研究生的最权威尺度,考研外语录取最低分数线(50~60分)的通过率被严格地控制在15~18%,一个的命运与其外语能力,在这里更确切地说是与考研英语能力(注意,有人的外语水平不低,但考研英语成绩却不理想)息息相关。
阅读理解与考研英语
     长期以来考研英语阅读理解部分的教与学总是处于一种尴尬状态之中。
一方面,人人知道它的重要性:阅读理解占研究生入学考试英语总分的50%,每道阅读题分值为2分,往往因一念之差,选错一题,就会痛失2分,需要提醒大家注意的是,有的时候半分就能决定成败。
站在更深的层次,考研英语虽然从形式上分为词汇、语法、改错、完形填空、阅读理解、翻译和作文等几部分,但归根结底都是在变相地考阅读理解。所谓完形填空无非是把阅读理解的文章去掉若干个词,要求您在阅读理解的基础上补缺;翻译就是把阅读理解文章选出几句话来,同样需要您在阅读理解的前提下,用通顺甚至是华丽的汉语表达一下而已;词汇和语法都要求在对题干句阅读理解的基础上,再结合词汇及语法的具体特点才能作出最佳选择,而我们许多同学做词汇、语法题目时,非常重视词汇和语法的自身特点,却忽视了对于题干句语义的理解,这是非常简单而又常见的错误,当然也是这类题目准确率不高的一个主要原因;阅读理解不过关,作文也受到直接影响。阅读是别人写的文章让您读,而作文是自己写文章让别人看,别人写的文章您看不懂,头脑里没有素材、没有模型,自己笔下何来好文章。
对于广大考生而言,阅读理解部分完成的好坏关系研究生入学考试的成败,阅读理解过关等于研究生入学考试轻松了一大半。
但另一方面,考生大量背诵篇章,拼命作题,外加分析、总结,经过长期不懈的努力后,结果收效甚微。通常情况下,有很多考生阅读理解分数可以达到20至25分,但超过35分的同学却廖廖无几。

新与旧
   我认为一本书的新旧不在于其形式、不在于是否在其封面上打上2000年版或3000年版的标志,而在于其内容,在于其思想是否领先于那个时代,一个伟大思想辉煌时代的到来往往是在这个思想的创造者死后的半个或一个世纪——这就是人类的历史!
本书力排对着答案讲题、将阅读当成翻译来讲解的传统弊端,从分析应试高手心得出发,精研历年考研阅读理解试题,深入体会和理解真题的题型与思路;充分考虑到中国学生的实际水平和常见错误,针对各类题型为考生提供一套实用操作性很强的技巧和方法,提高考生临场实战能力,为未来的考试做好准备,这是同类书籍中的独家做法!同时也避免了考研讲解传统上的事后明白——“马后炮”的缺陷。要知道看了答案,再查词典,人人都会讲考研。

考生难点分析
   根据考生水平的水同,一般存在如下三种问题:
*部分考生读不懂文章,对于复杂句尤其是长句、多义词及难词理解有误。这些考生的英语阅读的基本功还有待提高。
*还有的考生文章读得懂,但抓不住重点,不知道到文章的什么地方去找答案,他们常常抱怨时间不够,来不及答题!
*对于水平较高的学生而言,尽管不存在上述问题,可以读懂文章,也知道答案在文章中所处位置,但由于受到陷阱题的蒙蔽而犯错误,事后会觉得冤枉。常常听到考生这样的对话:
考生甲:“这种题太阴险了,我看懂了文章就应该能作对题,但我看懂了,却做不对,没道理!”
考生乙:“那您还是没看懂!”

阅读高分对策   一半是基本功,一半是技巧。

基本功漫谈
  阅读的基本功是取得阅读高分重要保证,但这需要从长计议,非短期能奏效。中国大学英语教学体制是中国考生阅读理解能力低的主要原因。几乎所有到美国读书的中国学生会感觉到自己的阅读能力明显不足,因为在国外,通常学生每次课后作业需要阅读长达数十页到数百页的文章,非常不适应,感到自己多年来在中国炼就的高超语法水平(如虚拟语气有多少种特例、很多八百年都用不上一次的词语搭配)却无用武之处。
据我多年应试与教学的体会,中国学生阅读最大的障碍在于对英语长句、复杂句的感觉和把握。文章读不懂,其实读不懂的还是难的句子,目前在研究生入学考试阅读部分中出现含有50~70个词汇长度的句子已属司空见惯,如果文中充满了主系和主谓宾单句的话,恐怕没有谁读不懂!对于大多数同学而言,个别单词不认识并不会造成语义理解的障碍,除非您的单词量实在是太小了,事实上很多同学阅读水平处在这样的一个阶段:很多单词是认识的,但句子长的,结构复杂了,句子语义理解起来就很慢、很吃力,或根本不理解,从而导致整个文章理解的障碍。
不见高山不显平地,事实上,只有当您能够读懂高于考研水平的句子的时候,才能真正享受到读考研文章的乐趣,才能够体会到考场的做阅读题时的轻松感。那么对于难句怎样才能过关?请参照王若平《硕士研究生英语入学考试阅读基本功——难句过关》、石春桢《精编英语阅读220篇》、《新概念英语4》。

考研阅读能力的最低要求
  文章读一遍,能知道文章的中心、段中心及文章结构是对于考研阅读的最低要求,否则,无论您如何努力,不提高基本功,都无济于事。

我不是教您诈——技巧的实质就是反命题
取得阅读高分的另一半就是技巧,技巧的实质是什么?可以说技艺就是反命题,技巧的运用过程就是命题与反命题的较量过程。其实命题和小时玩的捉迷藏相似,每一道题目都是经命题者精心设计的。很多同学都有这样的体会,当题目吃不准的时候,做对很难,做错却很容易。我对题目的四个选项是这样划分的,其中作为答案的选项是解,其余几个选项都是干扰项,所谓干扰项就是您特别容易去选但又是错误选项,是非常有诱惑力的选项。其实命题的过程就是一个合理地设计陷阱的过程,反命题就是反其道而行之,理智地躲避陷阱。
所谓技巧是通过对标准化命题特点的理性分析、研究命题大路套路,在掌握命题规律的基础上,预测题目,或在对题目理解不全面的情况下,抓住标准化命题的致命弱点,最大概率求解。甚至只看选择项,不看文章,即可选对答案。
本书是一本什么书?可以说本书是帮您从思维上战胜阅读的专用书。利用命题规律和解的特征提高答题命中率,这正是本书的重中之重,芸芸考生的一大法宝。

阅读应试技巧来源于英语应试鬼才们的应试心得
   中国目前有一些英语应试的精英,被西方人认为是英语应试鬼才。为什么说是鬼才呢?因为这些人英语考试经常会取得一些难以令人置信的好成绩,他们考研究生就得97分,考TOEFL就得677分,考GRE就得2340分以上,考GMAT就得750分以上。当初,中国人GRE一考到2300分以上,美国人就会取消这些成绩,理由是作弊,因为正常“人类”是不应当考到这样高分的,连命题人自己也做不到,除非作弊。后来每年都有中国学生GRE考分超过2300分,他们才不得不承认中国人的智商确实达到这个程度了,西方人就将他们称为是英语应试的奇才、鬼才、怪才。技巧从何而来?技巧是中国英语应试鬼才的考试心得!本书的考研英语之九阴真经部分集英语应试鬼才们的应试心得之大成与考研命题专家之精华,希望大家细心体会高手们对于命题的理解与临场应试的心法。

题型就是出题的套路
  相声中讲的黄半仙算命,老百姓说他算得很准。比方一个农村妇女丢了首饰,也就告诉她,我已经算清楚了,丢的东西定是在锅台边、灶坑边,或床底下,这些都是她他常“出没”的地方,结果当然能找到。这和抓贼也一样,如果要黑社会老大抓一个小偷,就易如反掌,因为他对贼路很熟悉,知道他们哪些地方会去,那些地方根本不会去,可谓探囊取物,手到擒来。以上的事例无非是想说明,“仙有仙路”,黑社会有黑道,解题也同样要熟悉命题规律,掌握题路。本书中真题挖掘部分为原命题组长的对真题命制规律的剖析。

技巧的总结需要理智的头脑
   历史上范蠡辅佐越王功成身退,不是他不想当官,而是他明白越王是一个只能共患难、不能同享乐的人而已,但文仲却没有这个智慧而被杀。后来,范蠡在民间做生意,发了财,他的一个儿子犯了死罪,他要派他的小儿子去行贿,但长子坚持要去,不得已,只好让他去了。范蠡说:长子此去,一定会失败,因为长子同他一起创业,倍感挣钱的艰辛,对于财物很珍惜,出手不大方,所以不会成功;而小儿子是在优越的环境下长大的,自然能够挥金如土,才有望成功。后来果如其言,其子被杀。我讲了范蠡的故事无非是提醒大家,洞察力是一个人应付一切包括考试在内的一种十分重要的能力。与以往的技巧书比起来,理性和洞察力是本书的最大特色。

战斗经验的总结——“考过的是解”
   每套试题无非是各种题型按一定难度的组合,所以作完题目后的总结很重要。总结什么?大多数老师和同学的作法是总结词汇及用法,这有助于累积英语的零碎知识。但需要提醒大家的是对于题目的总结更重要的是对于题型的归纳和分析,要知道,凡是全真题出现过的题型、考过的思路都极可能是重复的考点。要记住:“考过的是解。”
记得我的一个学生在考六级之前一个星期,很没信心,说自己的基本功太差,后来我就说你就将从前做的全真题的解都看一遍,总结一下解的规律,看看有没有感觉。后来他对我说:确实很多解都有规律,尤其是难题,如:“考过的是解”、“隐蔽外有解”、“合理项不是解;不合理项是解”、“中心思想是解”、“照抄原文不是解;同义替换是解”、“含义肯定的不是解,含义不肯定的是解”、“具体的不是解,概括性的是解”、“系表结构是解”、“some是解”、“具体是不是解;抽象的是解”、“简单的不是解,复杂的是解”、“含义矛盾的是解”、“复合句是解”、“比较结构是解”等等。后来他在考研考试后说,考场上感觉好级了,因为这次考试难题太多,如果没有对解的深入理解,恐怕即使花再多时间,也很难做对。当自己做完题后,看到别人仍然苦苦奋战于考场上的时候,心中暗暗得意。后来此人果然成绩不错。此君悟性极高,故能顿悟标准化命题的真谛。


目   录

1考研阅读之倚天屠龙篇--------考研阅读心理学
2考研阅读之九阴真经篇--------阅读应试理论
3考研阅读之盘龙云海篇---------考研阅读潜在命题点的挖掘
4考研阅读之射雕英雄篇—————最新阅读练习精选
5考研阅读笑傲江湖篇—————最新考题全真测试

第一部分 考研阅读之倚天屠龙篇
------------考研阅读心理学
最好的精神食粮
出人头地、实现自我价值的抱负,坚忍不拔、苦中作乐的毅力,非凡的智慧、“宁静以致远”的心态和独孤求败的必胜信念,是考研最好的精神食粮。剑未出,意先至,降龙伏虎。

成功者
在从千军万马中杀将出来的成功者中,有的是大学一毕业的就考研、占有天时之利的应届生,有的是屡战屡败、屡败屡战的“老运动员”,还有的是辞去公职、孤注一掷的考研勇士,还有的是为摆脱不幸婚姻羁绊、改变不良生活环境苦心孤诣的拓荒者。
自信
自信至关重要,自信在某种情况下超越了学习本身。当年考研究生的时候,我和一个师兄说,“我考的这个专业仅我知道的报考人数就超过20个,确实挺担心的!”师兄笑了笑说:“要是高手,一个就够了,要是不行的,一百个也没用。”的确,在这庞大的报考大军里有许多人只是想试一试,这些人的竞争力是不强的。一个强者足以制你死地,一百个弱者不构成威胁。
我相信自己的能力,这咱自信来自于本身的素质,更来自于对自己的思考。自信与勇气是人最可贵的品质,它不是与生俱来的,而是需要一点点的培养和呵护。而我对考研的最深的体会之一便是始终如一地相信自己:你就是自己的主宰之神。这样才有一种持续不断的能力去支撑自己,踏踏实实地向前走,便会使所谓的坚持、韧力成为自然而然的东西。这段时间,婚恋、房子、金铁必须暂时搁置,一切凡尘杂念,抛诸脑后。记住老子的这句话:圣人无为而无所不为,圣人私而故能其私。凝聚你的精力于一点,就会无坚不摧,也就是所谓的无欲则刚。

“苦大仇深者必胜”
考研与其说是知识的竞争,不如说是意志力的竞争。某君的家乡在一个小山村里,那里的条件特别差,“在座诸位生长在大城市,比我当时的条件好多了。那里别说辅导,连上学都难。”后来他去大城市进修,一心一意想考研,国内当时如果考不上就得回农村,终于拼了老命考上了,跳出了农门。自古那些流芳千古的文学大家(在古代,人才和文才几乎可以画等号,那时候几乎没数理化)一般都是在仕途不利的背景上抒发自己郁闷的情怀而有所成就的;而那些通过科举当上官的人则易于碌碌无为,并不是那些当官的人资质差,而是因为他们已经满足现状从而限制了他们上进。当今那些大城市的、条件优越的、有后路的同学不容**上研究生的原因也在于此。要想考研,必须从思想上断绝自己的退路,置之死而后生,苦大仇深者必胜!
“从绝望中寻找希望,人生终将辉煌”——一位智者如是说。


考研方法论——不动声色、暗渡成仓成功范例
  一位老兄,在某中专任教,觉得将来实在没有出息,就暗下决心,刻苦考研。但是周围环境不允许,他的酒肉朋友、同事和其他人经常找他玩。他每天总是趁别人不在的时候,暗暗地学,或偷偷地带书出去学。该人确实是一位“武林奇才”,他的考研书一般都包上牛皮纸书皮,上书“笑傲江湖”、“射雕英雄传”、“天龙八部”等武侠书名。有时学校开大会,领导在上面讲话,他在下面津津有味地读“金庸小说”,别人也就见怪不怪。平时朋友请喝酒,照去;打牌,照去;单位举办舞会,照去。表面上一切如常,但他每天有一条铁的纪律:必须潜心读三小时的考研书。因为装得太像,别人根本没有觉察到他在准备考研。后来他去考研,周围的人都以为他没戏,最后他们单位几个大张旗鼓考研的人反倒没考上,他却成功了。
我讲这个例子不是让你去学他,而是让你体会方法的多样性,每个人都应找到最适合自己的方法。具体的方法因为而异,有人适合早睡早起,有人适合开夜车,有人适合以时间求成绩,有人善用效率换分数,有人明着学,有人偷着练。总之,“不管白猫黑猫,抓着耗子就是好猫。”
  禅说
  心烦意乱是考研的大敌,本人消除烦恼的办法很简单。凡事对于我而言都可以分成两类:一类是目前可以解决的事情,即使难度很大,也是属于困难,困难是可以通过努力来解决的。对于这类事情一定要立即去做,否则很多现在可以通过努力解决的事情,如果拖沓不做,将来做起来会很难,将会变成困扰;另一类是现在不能解决的事,则是困扰,对于这类事情就无需考虑。困扰和困难不同,困扰是你所不能解决的问题。如果硬着头皮总想着做,却做不成,只能所思不遂,这就是困扰的根源。目前解决不了的事情很多是可以通过时间来解决的,那么就留给将来去做。
一位老者对自己的儿孙们说,“不要有过多的担心,想想在我这一生中所有担心的事情中,真正发生的有几件?”是呀,想想看,在生活中,我们是不是有太多没必要的担心?
“不思过去,不思未来,我生只有一呼一吸须臾之间”,佛说。

平常心
   不见高山不显平地,平常心的获得往往并不是在平常的环境下轻易得来的。
“考前几个月总是担心考不上,又想找工作,怀疑自己辞职考研这步棋是不是走错了。晚上常常失眠,白天无精打采,寝食不安,体重下降了十多斤。突然在我所在地区发生了一场强烈的地震,在地震带来的恐慌恐惧之中体验到了生死轮回,在生死关头,原先折磨那些忧虑都显得那么渺小、不屑一提。当一夜未眠的我看到太阳出现在地平线上,心里终于打定了主意。谋事在人,成事在天,只要我努力了,即使失败也没有什么后悔的。卸却了精神枷锁(其实这一枷锁是自加的),全力一搏,考取了研究生。”一个成功的考生现身说法。
“二战新期间,我是盟军的一个潜艇上的一名工作人员,不幸潜艇被德军发现,包围在一块水域中,德军使用深入密集轰炸,我此时此刻深切地感到生命的脆弱,时刻都有粉身碎骨的危险,轰炸持续进行了三天,在惊恐万状的三天中,每一分、每一秒都显得那样地漫长,三天仿佛三年!后来德军以为我们已经不复存在了,便离开了。度过了这样的灭顶之灾,以后遇到什么艰难的事情都不会感到紧张了,都能用一种平和的心态去面对它。”一位第二次世界大战中的老兵回忆说。

境界
孔子曰:“生而知之者上也,学而知之者次也,困而学者又次也,困而不学者,民斯为下矣。”自古至今,人和人的境界就存在差异,下面让我们从不同层次的人面对考研过程中一些常见问题态度探讨一下考研的境界。
    问题一:“考研太难了,不说专业课,就说外语和政治这两门公共课就要命,不知要花多少时间和精力?”
   普通境界:秉承中华民族自强不息、吃苦耐劳优良品质,心中想的是今后甚至是一生的幸福,吃苦在先,享乐在后。切记:吃得苦中苦,方为人上人。
   最高境界:与天奋斗,其乐无穷!与地奋斗,其乐无穷!与人奋斗,其乐无穷!快乐=奋斗。世界上没有比个人奋斗更有趣的事了——伟大的胸怀。

问题二:旁人对考研者有闲话、有看法,甚至打击、下绊都不足为奇,因为有些人能够容忍尸位素餐之徒在工作时间打牌、闲聊乃至吃喝嫖赌博,就是不能容忍好学之辈利用业余时间考研、考博。
   普通境界:“走自己的路,让别人去说吧!”。
   高级境界:“侮辱我吧,打击我吧,让我成为一真正的人!——钢铁怎样炼成的?”

问题三:害怕考不上,害怕失败,害怕旁人可能的闲言碎语,害怕万一自己考不上别人幸灾乐祸的神情,害怕……
普通境界:在真实的生命里,每桩事业都由信心开始,并由信心跨出第一步。当我们承担起巨大的困难的时候,千万不要被内心的怀疑所击倒。
高级境界:独孤求败——金庸心目中的偶像。

问题四:考场上什么心态答题?
   普通境界:读文章、读题、找答案,疲于奔命,被试题牵着鼻子走。
   最高境界:以一种挑剔、透视、居高临下的眼光观察试题,以轻松、愉快近似于捉迷藏的心情游玩于试题之中。看看这次出的是什么题、又是什么套路的出现?看看命题的质量如何,是否具备合理性?“这道题命题深得要领!”顺手就把题做上了——这是我答题时的状态。
     比大地宽阔的是海洋,比海洋宽阔的是天空,比天空宽阔的是人的胸怀——宰相肚里能撑船的翻版。
最后我想要说的是,过去的已经过去,留给自己的是教训和财富。要着眼未来,洒脱一点,以勇气去面对一切。我要写点东西的时候,总是想用心来交流。各位曾经过五关、折六将的朋友如能将自信、平各的心态和坚忍不拔的毅力带入考研乃至整个人生,则无往而不胜!

第二部分 英语阅读之九阴真经篇
阅读应试理论篇
一、考研英语阅读理解的特点
(一)有关文章难度方面的一些容易被人忽视的事实
    考研的文章大多选自国外比较新的书刊。对于一个英语专业的本科生而言,阅读国外比较流行的杂志如《Time》、《Newsweek》等,水平仅限于弄懂文章大意、了解内容梗概而已,若想像中国学生看《读者》一样,能够真正读懂每句话的含义、真正读出味道来,这几乎是不可能的。本人曾经用考研试题对一些英语水平较高但对考研没有专门准备过的英语专业本科生进行测试,一般成绩均在40~70分左右。如果一个人没经过考研的专门训练英语成绩就能达到70分,那就可以称为人才。难怪一个非英语专业的本科生读考研文章总是觉得那么难,这就意味着如果您要想从容地参加考研英语考试的话,您就需要至少在某些方面达到不低于本科专业英语(四年几乎是纯英语的训练)水平。那么考研英语阅读理解文章的难点在哪里呢?
     考研的考试大纲规定,文章中出现无法猜测而又影响理解的关键词,如超过大纲的范围,应用汉语注明词义。这样出题者在词汇难度方面就没有太多的发挥余地。事实上从最新的考研实考试题文章来看,绝大部分单词都是大纲中有的,少数超纲词汇也有中文注释,即使个别词不加注释,也不会影响对文章的理解。因此出题者就只能在句子难度方面做文章,通过使用复杂句来达到这一目的。简而言之,多重复合句、被动句、插入成分、分割句、省略句、倒装句和双重否定句的广泛应用,给考研阅读增加了决定性的难度。

  考研文章难度体现的另处一个方面是命题的难度。从目前来看,无论是对付考研赫赫有名的老手,还是初生牛犊不怕虎的新手都不能从本质上破解考研命题的思路,无法讲清楚为什么有些文章看懂了题会做错,无非含糊地说“题出得难就是了”。难究竟难在什么地方,命题都是通过什么方法使题目变难的,增加难度是否有一些常规的模式,你自己是否也能够出几道类似的难题,有没有什么办法避免此类错误。命题思路不清楚当然就更谈不上解题的技巧了。基于中国考生在应试中的实际情况,本书将对此方面进行重点介绍。
(二)文章题材及长度
   试题中阅读理解部分一般为五篇文章,每篇约400~600字,阅读总量在2000~2500字。文章选材广泛,出现率最高的是社会生活、科普和经济,其余的还包括体育、人物传记等。社会生活体裁包括文化、历史、家庭、教育、人口、交通、环境能源及其他社会现象。阅读这类文章的关键是抓住作者论述的中心以及作者对于理论或成果的优劣的评判。
考研阅读理解文章的题材多样,有议论文、说明文和应用文。希望大家注意不同体裁的文章不同考试要点:对于说明文,要注意事实和数据;对于议论文而言,要注意作者的结论和观点,以及作者对其他观点的态度。

二、生研阅读理解的大纲要求
*掌握所读材料的主旨和大意;
*了解用以阐述主旨的事实和有关细节;
*根据上下文判断大纲附表以外的某些词汇和短语的意义;
*既理解个别句子的意义,也理解上下文之间的逻辑关系;
*根据所读材料进行一定的判断、推断和引申;
*领会作者的观点和态度;

三、阅读的大忌——试图弄懂文章中的每一个词
     首先,中国学生通常存在的习惯性的错误做法是:考试时对于文章中的每一个词都希望尽量搞懂。遇到难的词汇的时候,尤其如此。这和中国式的英语教学有关。当养成一个不良习惯,特别注意生词、难词,却忽视了对意思的理解,很少有同学说过对大学英语课文感兴趣,因为他们学习大学英语的过程几乎就是一个背单词的过程。这样在阅读的时候,他们也按通常的习惯,注意词汇,重视局部,忽视整体。
     从应试得分的角度出发,这种做法是十分不合算的,搞清文章中每一个单词的意思固然对文章的理解以及答案的选择有帮助。但是由于考生词汇量、阅读理解能力及时间等的限制,我们无法、也不可能,同时也没必要把文章词汇全部搞清楚。对于大多数同学而言,在阅读过程中永远都要可能出现不认识的词,这是命题人员增加命题难度的一项措施。即使将来大家整体水平提高了,也会如此,因为出题的难度也会水涨船高。换句话说,您(指大多数考生)永远都会需要和生词打交道,要学会和生词打交道。
   对付生词的有效方法是根据合理化原则顺势阅读。也就是当您遇到生词的时候,如果不影响对语义的理解,就不要理它,顺势读下去;如果该词汇影响到整体语义的理解,就应当根据周围语言环境,根据合理化原则推测出一个合理的含义,继续顺势向下读。记住:您是在读文章,不是在背单词。

四、积极的阅读
   较高境界的阅读是把注意力集中在作者的思想上,而不是个别是零碎“单词”上。比如读到but的时候,应当能够“预见”到下一句的意思与上一句的不同,意思相反。能够抓住作者的文章的主题,及文章的脉络,这样就越读越明白,而不会只见树木,不见森林。
   
考研文章的常见脉络如下:
*时间顺序:按时间的先后说明某一理论的发展,某一研究成果由过去至现在的情况。
*一般 具体:首段做总的说明,其他段落分别说明或具体论述首段观点。
*具体 一般:前面几段分别说明,末段总结。
*对比:以进行对比的两个事物之间的基本共同点或差异为主题展开。
我们应注意根据文章中的结构词把握文章的脉络。文章按结构分类,又可分为顺承式文章和转折式文章:
*顺承结构指的是前后两个部分之间内容相近,相辅相成,通常由下列连接词连接:similarly , first, second, because, thus等;
*转折结构一般指两个部分之间内容相反,相互对立,通常由下列转折词连接:however, yet, but, while, nevertheless等。


区分文章是属于顺承式还是转折式,主要看文章中是否有转折词。若无,则为顺承式文章;若有,则为转折式文章。
我们还可以抓住一些结构词来判断文章结构走向,这些结构词包括:
1. 综述——详述结构词
*表示综述的词汇:often, generally speaking, thus, therefore等;
*表示详述的词汇:for example, because等。
2. 分类——列举结构词
*表示分类的词汇:there are two/three types等,其特点为“基数词”;
*表示列举的词汇:first(ly), second(ly), third(ly)…last(ly)等,其特点为“序数词”或“序数词+ly ”。
3. 对比——比较结构词
*表示比较即顺序的词汇:similarly等;
*表示对比即转折的词汇:however, but等。

五、传统上大力提倡的快速阅读法
*略读或跳读(Skimmming)即从头到尾寻找重点词、关键句去理解;
*查读或扫读(Scanning),即在文章特定部分寻找某一线索,就像我们日常阅读电视报当天节目一样。这两种方法都适用于长篇文章的阅读,目的是在尽量少的时间中了解文章,找到答案。

六、阅读速度假象
快速阅读在考试中的目的在于节省时间,做更多的题目或检查已做的题目。但快速阅读与其说是一种方法,不如说是实力的体现!以前我考六级的时候,做题常觉得时间不够用,后来考GRE、GMAT时,时间更紧张。但此后考TOEFL,却轻松得不得了,可以自由地在考试中上厕所,还可以悠闲地看看周围同学答题的样子。交卷之后,我旁边的一考生对我说,我看你答题太快了,一直往后翻,搞得我神经太紧张了。当一个人站在高处看低外的时候自然如此。很多同学在阅读的时候,往往是一目十行后,大脑中一片空白,根本不知文章所云。要解决速度和准确性这一矛盾,需要考生平时在阅读过程中多注意体会和训练,不宜过分强调速度。
一般来讲,我们提倡的快速必须是在对文章理解的基础上才能进行,也就是希望考生在一开始不要图快,不求速度,但求准确,什么时候读懂什么时候结束。然后通过一段时间的训练,理解能力上去了,速度自然就提高了。

七、阅读顺序——先读文章?还是先读题?
阅读的顺序很有讲究,仁者见仁,智者见智,有多种方法:
*大多数考生在阅读时采用下面最传统的三部曲:
第一部:读文章
第二部:做题目
第三部:有拿不准的再回头读文章
有的时候还可能重复循环下去,这种做题准确率较高,缺点是速度慢,效率很低。
*先浏览题,记住大致考点,再读文章,划下重点位置,最后做题,一气呵成。我比较喜欢此种方式。
*直接从问题入手,边读题边到前面的文章中寻找答案。
*不读文章,只做题目。本法仅适用于“狗急跳墙”的时候。

八、不良的阅读习惯
考生平时阅读文章时应注意克制以下常犯的毛病:
*“指读”:用手指、铅笔、尺子指着文章,一个单词一个单词地读。这样无形中限制了眼睛“扫描”的速度,从而降低了阅读速度。
*“头读”:每次换行读时,头作相应的摇动动作。这样时间长了,不仅人的颈部很容易出现疲劳的症状,也限制了阅读速度。
*“声读”:必须读出声音才能理解,也就是必须将书面上的字符转化成声音才能使脑子理解,这样也会影响阅读速度。因此,考生庆多进行“默读”训练,学会直接将字符转化成意思的技能。
*“回读”:一遇到生词或不熟悉的短语的时候,返回句首段首重读。
*“译读”:又称母语阅读。在阅读的过程中,不断进行单词、短语、至句子的翻译,通过译成母语来达到理解。

九、常规命题思路
中心+细节=文章
*文章中心常考:常于主题句中表现出来,主题句可以出现于文章的首句或篇中,一篇文章也可以没有明确的主题句。
*段中心常考:可以在段子的首句和末句,也可以没有明显的段中心句,需要自己概括出来。
*细节题的实质:细节题针对文章的某个局部命题,通俗地说,所谓细节题就是将文章中的一句话拿出来单考。
*指代(信息传递)常考:文章中的指代常作为考察内容。
命题模式:B指代A,B具有X特点,结论为A具有X特点。有时只是简单地提问指代关系。
*倒着考:即是将文章中的某句话,倒过来考一遍。
命题模式:文章中A导致B;问题:有B这一结果,为何?答案:因为A。
*反着考:即是将文章中的某句话,反过来考一遍。
命题模式:文章中A具有X属性,B与A不同;
问题:B有何属性?答案:非X属性。

十、阅读题类型及其特点
考研阅读题目一般有五种题型,分别是:
*主题性问题
*细节怀问题
*态度性问题
*推断性问题
*词汇性问题

下面详细介绍这5种题型的特点、解题思路:
(一)主题性问题
主题性问题,顾名思义,要求考生找到一篇文章的中心思想。命题形式有以下3种:
1. Main Idea 类型
典型问题形式:What is the main idea/point of this passage?
选项形式为陈述句。要求考生选择表达作者思想、观点的句子。
2.Main topic类型
典型问题形式:What is the main topic of this passage?
选项形式为名词或名词词组。要求考生指出文章的Topic(论题),或Subject(主题),或title(标题)等。
3.Purpose类型
典型问题形式:What is the author’s main purpose in the passage?
选项形式多为动宾结构。要求考生找到主要的写作目的。
   抓主题句是概括中心思想的一个常见方法,主题句表达中心思想,其他的句子均围绕其展开说明或议论。它通常以判断句的形式出现,在文章的通常位置为第一段首句、第一段末句和全文末句等地方。
*第一段首句 许多文章开门见山,一开始就摆出要说明的主要对对象或要论述的主要观点。接下去的句子和段落都说明或论述它。这样,文章的结构就属于一般 具体,主题句也就在首段首句。
*首段末句 也是主题句常出现的地方,在这种情况下,这个主题句不仅是对全文的总结,也是对第一段的总结,而且开启第二段及以后的说明或论述。简而言之,这种文章的结构是:具体说明或论述(一段)总结(一段末的主题句)后文的具体说明和论述。这种形式的主题句之前通常有表示总结的提示词,如:in consequence, to conclude, in summary to sum up, in short, in brief; 或表推断的提示词:therefore, thus, as a result, accordingly; 或表示转折的提示词:but, yet, however等。转折之后出现主题句的情况比较多,对于这种谋篇套路编写的文章,笔者给它起个名字叫新老观点对应性文章。文章中出现诸如a popular belief, frequently assumed, universally accepted 等词句提示作者将要在下文提出一个标新立异的与之不同的新观点,那么新观点就是主题大意,而作者对老观点的态度则是批判的。
*文章首末两段均有主题句 主题句到末段才出现的情况较少,常见的情况是文章开门见山提出主题句,中间加以论述,文末复述首段主题,这也是写作的方法之一。
主题性问题干扰项的特征:
*局部信息:是主旨题干扰项最常见的特点,是指选项中出现之涉及文章的细节,如一句无展开论述的话;某自然段的大意,此种选项干扰性很强,做题时尤其需要留神。有超过一半的文章中心在文章的首句和末句。从解的特征来看一般符合“概括性的是解;中心思想是解”。参见解是特征节。
*概括范围太宽:指选项包含的内容超出或多于文章阐述的内容。提问标题的干扰项常有此特点。此种干扰项是“概括性的是解”的克星,需要注意。
*无关信息:即文章中未提到、或找不到语言依据的信息。

(二)细节性问题
     细节性问题为阅读考题的基本问题,占很大比重。此类考题比较简单,因此是我们拿分的部分,有必要争取全部细节性考题答题正确。体会:大部分细节题体现“中心思想是解”。下面将细节性考题分类进行讨论。
1.According题型
此题型为最基本题,由According to the passage或The passage states that引导,考查学生对于文章中Factual(事实)内容的掌握,如时间、地点、事件等细节问题。
对策:利用题目中的关键词寻找文中对应词,在对应词周围查找到答案。
2.数字题型
此种题型为比较基本的题目,考查学生对数字的理解能力,一般有如下3种考查方法:
*运算型:通过四则运算(主要是加减法)求解。
对策:注意原始数据不是解。
*多个选一个题型:文中出现多个时间或数字,对应不同的事物,问题只考其中一点。
对策:对号入座。
*域型:文中时间后数字为一区域/范围,求最高、最低或其中某一点。
*世纪型:已知某一年份,求其为哪个世纪。如已知1906年,问其为19世纪初。这种题极易做错,请考生注意。
*文字表达型:文章并未出现数字,只有表示数字概念的文字,求其表示的数量。此种题目稍难,因为文章中没有现成的数字,要求考生通过表达数字要领的文字加以推断,如:一个圆被两条相交直线分割,分成几部分?
3.是非题型
*三误一正
该题型指三项错误,只有一项符合原文内容。典型提问方式:
Which of the following is true?
根据题干或选项中的关键词回返原文,找到相关句与选项相比较就能得到答案。如果定位困难,可以考虑运用排除法,不符合解的特征及“含有原文中未提到的新概念”等选项可以排除,这样找到正确答案的可能性就大。
*EXCEPT题型
该题型俗称“三缺一”题型,即题目四个选项中有三个符合文章内容,剩下一个不符合,题目便是要求选出这个不符合文章内容的选项。典型提问形式如:
Which of the following is true except……?
Which of the following is mentioned except……?
Which of the following is not mentioned?
       这种题型有两种考法,一种是考并列、列举句,只考一个句子,考局部。这是因为它要求其三个选项中的每一个都是文章中并列或列举句中出现的。利用这一特点,我们在读文章的时候就多留意,如看到并列、列举句,可预想到它有可能考到,如发现题目中有“三缺一”题型,则应到并列、列举外找答案。另一种是篇章性的考法:三个干扰英在文章中不同部分出现,难度很大。这种题目难度大于TOEFL,接近GRE。做EXCEPT题时需注意不要漏看题干中的EXCEPT,NOT等词。
4.例证题型
例子是考研的阅读理解常考点之一,典型提问方式如下:
The author provides in Line…(Paragraph…) an example in order to…
   例子是中心思想的体现,解的特征符合“中心思想是解”。文章出现例子无非是为了说明一定的道理,关键在于这个例子在文章中出现的位置,但不管如何,这个例子之前或之后不远处通常总有一句总结说明就是答案,即被例证对象。如果例子与全文主题有关,则例证主题,“中心思想是解”;如果例子与段落主题有关,则例证明段落主题,“段中心是解”。
细节题干扰项的特征
     细节题有难有易,较容易的可以根据题干或选项的线索回原文定位,然后由相关句得到正确答案。细节题干扰项有如下几种:
*无:选项中所讲内容在原文中根本不存在,或找不到语言依据:如本身正确但在原句中无语言依据的无关选项,原文中没有提到甲,但选项中有甲;原文中有甲与乙,但未对甲与乙做比较,而选项中说明甲比乙如何;原文中说甲有乙特征,但选取项中说甲有乙之外的特征;选项中的限定词表示的范围或程度大于或小于文中提到的范围或程度。
*反:选项的内容与原文相矛盾一原文说西,选项说东;原文说低,选项说高。
*混:原文说甲事物有某特征,但选项却张冠李戴,挪乙事物上。
*偏:原文中说某事物有诸多特征,而且题干要求多种特征,但选项仅讲了一个特征,这就属于偏。

(三)态度性问题
   态度性问题考查学生是否了解文章作者或文中某人对某事物所持观点或态度。命题形式为The attitude of the author towards something is one of …?
态度性问题的选项一般分为以下三种:
*褒义词:positive, wonder, support, useful, interesting, enthusiasm, admiring 等;
*中性词:indifferent, impassive, uninterested, ambivalent, neutral, apathetic, humor, disinterested, disinterested, disinterest等;
*贬义词:disgust, critical, negative, tolerant, disappointment 等。
策略:
*遇到这样的词,要用笔画下来,以免以后不好找。
*根据文中有典型褒贬含义的词汇判断。
*当题目要求找出作者对某人的态度时,答案多为褒义词,其他词类可能性较小。

(四)推断性问题
  典型提问方式t can be inferred from the passage that 。
此类型问题考查学生根据已知内容推断引伸含义的能力。命题形式包括下列五个动词:infer, inply, suggest, conclude 和assume。
此类型问题要求考生根据文章中的关键词、短语、结构等进行推断,或要求考生通过阅读某段或几段内容,推断出一个结论,类似于主题性问题。注意防止不依据关键词而凭空进行推断,这是许多考生存在一的个通病。

(五)词汇性问题
  根据上下文判断大纲词汇表以外某些词汇和短语的意义,主要考察两种情况:
*熟词僻义或具体词义在特定语言环境的场合考法。注意在这种情况下,常规含义不是解。
*超出词汇表的生词含义的推断。
主要策略:
*根据构词法推测超纲词的含义:熟读王争《词汇的奥秘》,不但可以使你速记考研词汇,熟悉词汇考试要点,而且通过由此掌握的构词法对临场推测超纲词汇很有价值。
*心里一定要清楚,只有根据上下文来判断该词真正具有合理性的词义,才是您唯一的出路。这里的上下文指该词所在句子本身,也包括句子的上下句或更远的上下文。一般而言,上下文线索所提示的意义与所考词汇的关系要么为同义关系,要么为反义关系。例如二者为同义关系,那么所考词汇的意思就是线索提示词的意思 假如二者为反义关系,所考词汇的意思就是线索提示词的反义。提示词与所考词汇关系的形式与结构如下:
同义关系:所考词汇在文章中被同义词、同位语、定语从句等解释或定义时,那么它与提示词构成同义关系,这些解释有时伴随过渡词,如namely, in other word, this means.
下定义的典型方式如下:
名词(被定义对象)+定语从句(定义内容)
名词(被定义对象)+be+名词(定义内容)
名词(被定义对象)+be called +(定义内容)
by+名词(被定义对象)+be meant+名词(定义内容)
反义关系:凡是所考词汇与线索提示词之间以下列结构表示转折或对立关系的,二者可构成反义关系。例如:but, yet, however, although, while, whereas, despite, in site of, in contrast, on the contrary等。
利用逻辑和语法:有时根据上下文提供的线索或其他方法,仍不能得到唯一答案,就可以利用语法和逻辑的规则来判断。

十一、阅读的路标-----文章十大命题原则
任何事物的运作都有其一定的规律,考研阅读理解试题也不例外,它完全体现测试法的理论,表现在命题上便是依据一定的命题原则进行题目设计。我们将这种规律归纳总结成以下十条原则,即十大考试要点:

(一)列举处常考
列举指的是First,… Second,…Third,…等逐条出,并列指的是A、B、C逐项列出。它们共同的特征是列出三点或三点以上的条目。本书统称为“列举处常考”。该类型语句常考的题型是“细节性问题”,主要有两种:
* Which 题型
该题型只要求从并列的三项中选一项作为答案,其它选项与题目无关。
*EXCEPT题型
该题型要求考生通过核对文章内容,排除满足文章内容的三个选项,而留下“不”符合文章的“错误”选项,作为答案,通常称为“三缺一”题型。参见阅读类型与特点节EXCEPT题型。

(二)转折处与强对比处常考
转折处常常是语义的重点,命题常常要涉及,转折一般通过however, but yet, in fact等引导。
强对比常由 unlike, until, however, but…引导。命题模式如下:文章中说A具有X属性,B与A不同。问题是B有可属性?参见常规命题思路节“反着考”。

(三)例子常考
句中由as, such as , for example, for instance等引导的短语或句子为举例句,需要注意的是例子一般是和文章的中心或段中心紧密相关的,常考“推断性问题”和“细节性问题”,而大多数这类问题的解都符合“中心思想是解”的解题思路。参见解的特征节“中心思想是解”。

(四)数字与年代常考
文中的数字、年代、日期等常常是命题者的命题点。参见数字题型。

(五)最高级及绝对性词汇常考
文章中常出现first, must, all, only, anyone, always, never, none等绝对性词汇或 most +形容词(副词)和形容词(副词)+est等最高级词汇,或only, sole, unique, simply(只要),just(只要)等表示唯一的词汇往往是考题要点,一般出“细节性问题”。这是因为它们都有一个共同的特点,那就是概念绝对,答案唯一,无论是命题还是答题,不会产生歧义和疑问,因此很容易命题,答案绝对正确。相反地,如果文章中出现相对性的词汇,就很难有唯一的答案的。请大家琢磨一下下边的例题,文中:Some of the people chose red hats, some chose green hats, and others blue ones. 问题:What color hats did some people choose? 因为其中some为相对性词汇,所以就没有唯一正确的答案,答案可能为red,green或blue,这样就会给评卷造成了困难。大家体会一下,其实命题也受到许许多多的限制,要按照一个固定套路进行。



(六)专有名词常考
包括人名、地名、或其他的专有名词。当您阅读之前,先扫视一下文章后的题,如果有专有名词,划下来,这样在阅读的时候才能有的放矢。

(七)隐蔽处常考
隐蔽处顾名思义,即是指句子中隐蔽之处、容易忽视之处,包括:
*同位语 *插入语 *定语 *长句后半句 *从句 *副词 *不定式

(八)因果句常考
句中含有如下结构或词汇的称为因果句:
*表示因果的连词:because, since, for, as, therefore, so, consequently等;
*表示因果的动词:cause, result in, originate form等;
*表示因果的名词:base, basis, result, consequence等。
这些因果句都是指明某两个事件之间因果关系的,尤为命题者所青睐,因为通过命题可以考查文中两个事件内在的因果关系。
此种句型一般出“推断性问题”。

(九)段落句常考
文中首段第一句(段首句)和末段最后一句(文尾句)都是十分重要的地方,往往是文章作者表达中心思想,进行总结综述的地方,因此常出主题性问题、细节性问题,和结构性问题。

(十)特殊标点常考
有一些特殊标点的含义也属于考查范围:
*破折号:表示理解,考细节性问题;
*括号:表示解释,考细节性问题;
*冒号:表示解释,考细节性问题;
*引号:表示引用,考细节性问题;
综上所述,我们可以利用上面所介绍的考题点,作为“路标”,迅速地指引我们找到正确答案;同时作为命题规律,检验自己所选答案的正确性。有些同学做题很快,其中一个很本质的原因在于他对考点非常敏感,见到“路标”就能预见考题,未见考题,已知答案。如果说读完文章后就能预测出后边会考哪五道题,确实是言过其实,但对应试高手而言,一篇文章读过,心理已预见到七、八道考题,而之后真实考题尽在预料之中确实是常见的。

十二、解的特征
     每道都由题干(即问题)与四个选项组成。我对四个选项这样划分:其中一个是答案的选项称为解,其余三个选项不称为选项,而称为干扰项,因为几乎每个选项都被命题者加上了十足的功力,麻痹考生,以达到不容易被考生答对的目的。这样由命题难度梯度决定,解必然有如下特点:
“中心思想是解”
对一主题题型“中心思想是解”非常好理解。但大家一定要注意到大多数细节题也符合“中心思想是解”,因为很多细节刀是围绕中心的细枝末节。
“隐蔽处有解”
指方章中的或一名话的隐蔽之,常见有同位语、插入语、定语、长句后半句、从句、副词、不定式等。
“合理项不是解;不合理项是解;无关项是解;事实是解”
问:“狗是干什么的?
答:“狗是看门的”
在各类英语考试中,这种选项显然是错误的因为人人都知道狗是看门的,狗是看门的是常识,是典型的合理项,合理项不是解。那么,狗是干什么的?我给大家举个例子:狗是锻炼身体的。很多同学不理解,觉得没有道理。其实文章中很可能六我们家养了一条狗,每天早上我和狗都去跑步,狗跑多快,我就跑多快,后来我身体就越来越好,所以狗就成了锻炼身体的了。但问题问狗是干什么的,干扰项中必然出现诸如狗是看门的选项,那是给那些想瞎猜的人准备的,理由很简单,就是不能让考生不看文章答对题。
总之合理项是指合理的选项,合理项主要分为两种情况:一种是生活常识,另一种是在不看文章的前提下,问题与答案极其吻合。
但想想看,反其道而行之呢?对策:“合理项不是解;不合理项是解;无关项是解;事实是解”。
“照抄原文不是解;同义替换是解”
如果选项中出现照抄原文的情况,有时选项同文章中的某句话一模一样,此选项高度可疑,而某选项通过同义替换,或词类转换如由名词转换成动词,则“同义替换是解”。
“含义肯定的不是解,含义不肯定的是解”
“人之初,性本善。”人类的本质是善良、这一点在考试上也一样,很多同学出于善良、朴实的天性喜欢得出一个确定的结论。“文章我看就要看懂懂,结论一定要明确”,很多同学就是这样想的,但一选就容易错误。命题者从加大题目难度的考虑,当考到对一个问题的结论的时候,通常不会让您得到肯定的一个结论,即使有非常明确结论的东西他也不喜欢考,要不怎么达到15%~18%通过率哪?这样我们很同学答题很不对路,而我则不一样,一见到含义不肯定的、模模糊糊的东西就喜欢,就觉得心潮澎湃,因为我知道不但我能答对此题,而且还知道别人错在那里,这里提醒大家一定要从思维上战胜考试。
“can是解” “could是解”“may是解” “usually是解” “Right是解” “most是解”
“more or less是解” “relatively是解” “be likely to是解” “whether or是解” “not definitely是解” “possible是解” “suspicion是解” “not necessarily是解” “dubious是解” “hesitate是解” “suggest是解” …
“潜在的是解” “potential是解” “trend是解” “threat是解” ...
“未知是解” “unknown是解”…
“绝对的不是解” “must不是解” “always不是解” “never不是解”“the most不是解”
“all不是解” “only不是解” “any不是解” “none不是解” “entirely不是解” “absolute不是解”
“最高级不是解” …
*例外:在表示原因,表示方式、方法时。
“具体的不是解,概括性的是解”
只见树木,不见森林,这是很多学生阅读的一个阶段,命题者常利用这一点“欺负”考生,将解做成概括性的选项,干扰项使用具体的内容,使同学犯瞎子摸象的错误。
“both是解” “various是解” “and 是解” “名词复数是解” “many是解” “general是解” “not only…but also是解” “系表结构是解” …

“some是解”
some作解,既符合概括性的是解,也符合含义相对的是解。具备此两者“美德”的单词不多,所以some倍受命题者青睐,古今中外的各种考试中不时出现。别忽视了长相和some不一样的certain,其实质也是some的一种。
“someone是解” “somebody是解” “something是解” “someday是解” “certain(一些)是解”

“具体的不是解,抽象的是解”
与具体的选项相比,抽象的不容易理解,这成了命题者增加难度的常见手段。
“approach是解” “concept是解” “misconception是解” “awareness是解” “property是解” “character是解” “chance是解” “opportunity是解”

“重要是解”
重要是解,是“抽象的是解”的一种常见形式。
“important是解” “necessity是解” “essential是解” “significant是解” “dominant是解” “special是解” “vital是解” “particular是解” “fundamental是解” …
“基础是解” “basis是解” “be based on 是解” “basic 是解” “in the nature是解” “origin是解” “originate是解” …


“虚词型的解”
“another是解”  “other是解” “more是解” “either是解” “also是解” “beside是解” “additional是解” “extra是解” “same是解” … “部分是解” “nearly是解” “not enough是解” “part是解” “inadequate是解” …
“简单的不是解,复杂的是解” “含义矛盾的是解” “different是解” “separation是解” “division是解” “X and not X是解” “instead of 是解” “complex是解” “discuss是解” “极端的不是解,中庸是解” “复合句是解” “长的是解” “比较结构是解” “字面意思不是解,深刻含义是解” “双重否定是解” “三重否定是解” “难的是解” “difficult是解” …

“相互作用是解” “interfere是解” “effect是解” “each other是解” “affect是解” “respond是解” “adapt to是解” “influence是解” “compensate是解” “associate with是解” “relationship是解” “cooperation是解” “depend是解”

“变化是解”
“change是解” “shift是解” “vary是解” “alter是解” “variation是解” “formation是解” “no longer是解” “delay是解” “improve是解” “postpone是解” “increase是解” “convert是解” …

“积极向上的是解”
凡是参加考研的同学,都是国家重点培养的人才,作为国家的未来的代表,一定要注意灌输正面的、积极向上的思想,考研试题也不例外,这即是“积极向上的是解”的根源。
“objective是解”:记住作者的态度一般永远是客观的,如果选项中出现此选项的话。
“独出心裁是解” “new是解” “inventive是解” …

“主观是解”
主观的难于客观的,符合“难的是解”
“忽视是解” “neglect 是解” “ignore是解” “expect是解” “speculate是解” “suspect是解” “overlook是解” “overestimate是解” “like是解” “dislike是解” …

二选一,50%概率求解的方法:
“反义项,解在其中”
“形似项,解在其中”
“近义项,解在其中”
实战出真知,技巧是从大量试题中总结出来的,要想真正掌握技巧,那就要去实战中去。请大家继续阅读本书,当您读完本书后,再重读本篇,体会一下是否对此有更深入的理解?是否有新的感觉?
十三、万无一失的答题方法
*从正面解题,抓住文章的中心思想,掌握文章的脉络结构,明查文章的细枝末节。
*从反面验证解的合理性,及命题的合理性,避开陷阱,排除隐患。
*不但要知道哪一个是解,解对在那里,而且还要知道其余三个非解选项(即干扰项)错在什么地方。
如果您能够做到以上三个方面,(注意此三者并无因定顺序,尤其是在答题遇到不顺利的情况下),那您才真正达到高手做题的那种与世无争、独孤求败的境界。这与其说是方法,不如说是境界,没有非凡的阅读能力,只能是纸上谈兵。如果您现在没有此能力,不要着急,那就请您将它看作是您今后奋斗的目标,催人奋进的动力。







第三部分 考研阅读之盘龙云海篇---------考研阅读潜在命题点的挖掘

第一节 阅读理解基本理论

一、备考
1.体裁
虽然大纲说文章材料可多样化,包括记叙文、说明文和议论文,但从实际考题来看,以议论文为主,说明文为辅。请看对最近5年考题的统计:
年份 议论文 说明文
2003 4 0
2002 4 0
2001 4 1
2000 5 0
1999 4 1
考研的议论文一般都存在鲜明的观点和态度,作者的写作目的就是为了陈述观点,或表明态度。论点的特征是其争议性,内容包括支持或反对某一观点,提倡某一思想或就某一话题表示明确的态度。文章的组织结构逻辑性强,段落内部句子之间联系紧密,段与段之间意思紧密相连,整篇文章围绕一个中心展开,各个段落和中心意思存在某种联系。文章内容抽象,并且有充足的证据。
说明文大部分属于新闻报道。其特点是以事实为主,观点为辅,从不同方面论文某一话题。
出题者不会考虑你爱不爱读他所选的文章,他关心的是文章的语言难度、背景难度、适不适合出题,因此所选的文章大都较枯燥,即使不枯燥,由于语言难度大,也会使你感到乏味,尤其是碰到平时不甚了解的主题。考生在阅读心理上要自我调节,当文章读着费劲,感到烦躁时,告诫自己:我在考试,文章乏味是正常的。
2.题材
主要包括广泛关注的社会话题(注意:这里所说的社会主要不是中国,而是西方国家尤其是美、英、加、日等国),如社会生活、文化、教育等的发展问题,科技新进展和商业经济等题材。
3.语言
考研阅读文章最明显的特征是句子的结构复杂,而且较长。我们采用语言难度测试软件对英文书刊文章进行了分析:一篇文章的阅读舒适度在0~100之间,难度与舒适度成反比,该数值越大文章越容易读,越小则阅读难度越大。教研英语文章大部分的阅读舒适程度在10~50之间,平均为40左右,总体来说读着费劲,很不舒服。根据可读性可把文章按难度分12个级别,即1~12级:1级最容易,12级最难。大多数教研阅读文章的难度级别在7~12之间。可见如果考生能把教研文章非常顺利地读懂,再注意扩充单词,你的英语阅读就有相当的水平了。为简化起见,我们用可读性发表示阅读舒适程度。本书所选的每个段落或每篇文章都注明这两个指数,供考生参考。
4.与本书同时阅读或先于本书阅读的书籍:
报刊选读
商业类:Business Week(商业周刊),Economists(经济学家杂志)。
时事/社会问题类:Newsweek(新闻周刊),Times(时代周刊),The Washington Post(华成顿邮报),USA Today(今日美国),The Times(泰晤士报),The Guardian(卫报)。
科普类:National Geographic(国家地理杂志),Scientific American(科学美国人),Science(科学杂志),New Scientists(新科学家),Discovery(发现杂志)。
准备词汇的书
《傻哥考研词频归类记忆》:傻哥编编著。
《硕士研究生入学考试星火词汇巧记速记》:“星火考研系列”体系,青岛海洋出版社出版。
准备长难句的书
《硕士研究生入学考试阅读基本功——难句过关》:“考试虫”体系,航空工业出版社出版。


二、阅读的顺序——先读文章还是先读题?

     这个问题可以说因人而异,各有利弊。常见的有三种阅读方法:
1) 先读问题,读完一题然后去文章相应的地方找答案,看一题做一题。优点是:针对性强,阅读时无关内容可略过,阅读速度快,省时间。教研阅读题总体来看,题目的先后顺序与其对应于文章中的先后顺序大致相同。但肯定有少数题目不按原文的顺序出现。这种情况往往出现在下列题目上:段落大意题、个别句子题、词汇题。缺点是:对文章的整体把握差。对于下列题型可能会感到无从下手:主旨题、上下文逻辑关系题、作者观点/态度题、判断推理题。
2) 通读文章后看题,然后再回来读有关段落。优点是:对文章有一个总的概念和印象。对于下列题型有利:文章主旨题、上下文逻辑关系题、作者观点/态度题、判断推理题。缺点是费时间,对文章的细节记不清楚。
3) 读一层意思做几道题,阅读和做题交叉进行。优点是:速度适中,对于刚读完的内容印象深,记忆效果好。特别当考题集中在前面的段落时,优越性最明显。缺点是:对文章的整体理解不如第二种方法强。如果出题者不按文章内容顺序出题,此方法就行不通了。请大家试用上述三种方法做下面的的阅读理解题。

  It’s a rough world out there. Step outside and you could break a leg slipping on your doormat. Light up the stove and you could burn down the house. Luckily, if the doormat or stove failed to warn of coming disaster, a successful lawsuit might compensate you for your troubles. Or so the thinking has gone since the early 1980s, when juries began holding more companies liable for their customers’ misfortunes.
  Feeling threatened, companies responded by writing ever-longer warning labels, trying to anticipate every possible accident. Today, stepladders carry labels several inches long that warn, among other things, that you might — surprise! — fall off. The label on a child’s Batman cape cautions that the toy “does not enable user to fly”.
While warnings are often appropriate and necessary — the dangers of drug interactions, for example — and many are required by state of federal regulations, it isn’t clear that they actually protect the manufacturers and sellers from liability if a customer is injured. About 50 percent of the companies lose when injured customers take them to court.
Now the tide appears to be turning. As personal injury claims continue as before, some courts are beginning to side with defendants, especially in cases where a warning label probably wouldn’t have changed anything. In May, Julie Nimmons, president of Schutt Sports in Illinois, successfully fought a lawsuit involving a football player who was paralyzed in a game while wearing a Schutt helmet. “We’re really sorry he has become paralyzed, but helmets aren’t designed to prevent those kinds of injuries,” says Nimmons. The jury agreed that the nature of the game, not the helmet, was the reason for the athlete’s injury. At the same time, the American Law Institute — a group of judges, lawyers, and academics whose recommendations carry substantial weight — issued new guidelines for tort law stating that companies need not warn customers of obvious dangers or bombard them with a lengthy list of possible ones. “Important information can get buried in a sea of trivialities, ” says a law professor at Cornell Law School who helped draft the new guidelines. If the moderate end of the legal community has its way, the information on products might actually be provided for the benefit of customers and not as protection against legal liability. [390 words 阅读舒适度:46.5(非常不舒服) 难度:11.5(较难)]
1. What were things like in 1980s when accidents happened?
[A] Customers might be relieved of their disasters through lawsuits.
Injured customers could expect protection from the legal system.
[C] Companies would avoid being sued by providing new warnings.
[D] Juries tended to find fault with the compensations companies promised.
2. Manufacturers as mentioned in the passage tend to
[A] satisfy customers by writing long warnings on products.
become honest in describing the inadequacies of their products.
[C] make the best use of labels to avoid legal liability.
[D] feel obliged to view customers’ safety as their first concern.
3. The case of Schutt helmet demonstrated that
[A] some injury claims were no longer supported by law.
helmets were not designed to prevent injuries.
[C] product labels would eventually be discarded.
[D] some sports games might lose popularity with athletes.
4. The author’s attitude towards the issue seems to be
[A] biased. indifferent. [C] puzzling. [D] objective.

答案和解析
注意:为简明起见,在本书中规定第一段用 <1 >表示,第二段用 <2 >表示,依此类推;某段中的第一句用①表示,第三句用②表示,依此类推。
1. 。细节题。 <1 >④⑤“幸运的是,假如门口的擦鞋棕垫或炉灶上没有警示语告诉你可能遭受的危害,那么就可以提起诉讼,官司打赢就可获得赔偿,以弥补你所遭受的损害。自20世纪80年代初以来,陪审团成员开始认为更多的公司应对其顾客所遭受的不幸负法律责任。”可见“受伤的消费者可望得到法律制度的保护”正确。[A]不对,因为文章没有说“通过法律诉讼可以减轻他们所受的灾难”,灾难是无法减轻的。[C]、[D]均与文章明显不符。
2. [C]。细节题。 <2 >①“公司感觉受到了威胁,便作出了反应,其做法是写出越来越长的警示标识语,力图将种种可能发生的事故预先标明。”可见,许多厂商往往“利用标签避免法律方面的责任”,故[C]正确;写标签并非为了“让顾客满意”,故[A]错。、[D]均与文章不符。
3. [A]。逻辑细节题。问作者在文中援引Schutt头盔案例是何用意。作者举这样一个例子,肯定是为了证明自己所要表达的某个观点。在相关的上下文中,即 <4 >①②“现在看来这种潮流正在转变。尽管人身伤害索赔一如既往,但有些法庭已开始站到被告一方,特别是在那些有警示标语可能也改变不了什么的案件中。”可见下文所举Schutt头盔安案例是为了表明“某些受伤索赔已得不到法律的支持”,故[A]正确。、[C]、[D]皆错。
4. [D]。态度题。纵观全文,作者客观地叙述了从80年代初至今美国法律在这一问题上的态度变化,并没有表明自己的主观意图,因此,作者的态度是不偏不倚的是,是“客观的”,故[D]正确。[A]“有偏见的”;“冷漠的”;[C]“令人迷惑的”,皆不正确。
这篇文章属于报刊类文章,文章各段的大意清晰,细节部分为具体事例,适合看题做题阅读法,但最后一题可能会出错,所以没有十全十美的阅读方法。我们建议先花30秒钟看题,最好用笔划上题干中的关键词:
1. What were things like in 1980s when accidents happened?
[A] Customers might be relieved of their disasters through lawsuits.
Injured customers could expect protection from the legal system.
[C] Companies would avoid being sued by providing new warnings.
[D] Juries tended to find fault with the compensations companies promised.
2. Manufacturers as mentioned in the passage tend to
[A] satisfy customers by writing long warnings on products.
become honest in describing the inadequacies of their products.
[C] make the best use of labels to avoid legal liability.
[D] feel obliged to view customers’ safety as their first concern.
3. The case of Schutt helmet demonstrated that
[A] some injury claims were no longer supported by law.
helmets were not designed to prevent injuries.
[C] product labels would eventually by discarded.
[D] some sports games might lose popularity with athletes.
4. The author’s attitude towards the issue seems to be
[A] biased. indifferent. [C] puzzling. [D] objective.
如果题干中没有关键词,如:Which of the following is true?就要看选项中反复提到的词汇。浏览完选项头脑中留下关键词的痕迹再去看文章,效果就比较好。


三、如何在文章中做记号
善于阅读的人往往在文章的某些句子或词语下面划线,表示重要,以便以后查找方便。实践证明,这种方法在考试中是非常有效的。因为通过标出重点,你的阅读更活跃,思维更积极,注意力集中,减少大脑“走私”。考研文章较长(450词左右),考生很难记住所有的内容,甚至段落大意都可能忘记,而考题只有五个,不可能所有的句子都重要,都被考到。实际上看懂大部分句子就可以了。对主要句子,如表达中心思想、段落大意的句子,应该标出并重点阅读。
1. 文章主旨句
主旨句也就是中心思想句,通常在第一段,可以是第一句,也可能是最后一句,据有人统计位于最后一句的几率高于第一句。主旨句偶尔出现在末段,有的主旨句甚至是文章中间某段中的某句。主旨句的特点是:1)是作者的观点(不是他人的观点,不是描写/说明,也不是事实)。2)该观点可能是作者提倡的,也可能是作者对他人观点的批驳。3)作者的观点只能有一个,其他的内容都起说明/论证作用。4)主旨句具有归纳性、概括性、抽象性的特点。5)常常结构复杂(命题者有意安排所致)。
例1
An invisible border divides those arguing for computers in the classroom on the behalf of students’ career prospects and those arguing for computers in the classroom for broader reasons of radical educational reform. Very few writers on the subject have explored this distinction — indeed, contradiction — which goes to the heart of what is wrong with the campaign to put computers in the classroom. (可读性:33 难度:12级)
①是说明句,交代背景“计算机走进课堂的支持者有两派,这两派之间的界限是无形的。”②是主旨句,说明主题——两派之间的这种差异说明“计算机走进课堂的运动”本质上是错误的。
例2
Few creations of big technology capture the imagination like giant dams. Perhaps it is humankind’s long suffering at the mercy of flood and drought that makes the ideal of forcing the waters to do our bidding so fascinating. But to be fascinated is also, sometimes, to be blind. Several giant dam projects threaten to do more harm than good. (可读性:58.5 难度: 8.7级)
①引出话题“巨型水坝”。②解释修水坝的原因。③转折。④是主旨句“一些巨型水坝弊大于利。”
例3
Exceptional children are different in some significant way from others of the same age. For these children to develop to their full adult potential, their education must be adapted to those differences. (可读性:45 难度:10.9级)
①引出话题“超常儿童”。②是主旨句“为了充分发展超常儿童的潜力,对他们的教育必须适应他们的特点”。
例4
Money spent on advertising is money spent as well as any I know of . It serves directly to assist a rapid distribution of goods at reasonable price, thereby establishing a firm home market and so making it possible to provide for export at competitive prices. By drawing attention to new ideas it helps enormously to raise standards of living. By helping to increase demand it ensures an increased need for labour, and is therefore an effective way to fight unemployment. It lowers the costs of many services: without advertisements your daily newspaper would cost four times as much, the price of your television licence would need to be doubled, and travel by bus or tube would cost 20 per cent more.
And perhaps most important of all, advertising provides a guarantee of reasonable value in the products and services you buy. Apart from the fact that twenty-seven acts of Parliament govern the terms of advertising, no regular advertiser dare promote a product that fails to live up to the promise of his advertisements. He might fool some people for a little while through misleading advertising. He will not do so for long, for mercifully the public has the good sense not to buy the inferior article more than once. If you see an srticle consistently advertised, it is the surest proof I know that the article does what is claimed for it claimed for it ,and that it represents good value.
Advertising does more for the material benefit of the community than any other force I can think of.
There is one more point feel I ought to touch on. Recently I heard a well-known television personality declare that he was against advertising because it persuades rather than informs. He was drawing excessively fine distinctions. Of course advertising seeks to persuade.
If its message were confined merely to information — and that in itself would be difficult if not impossible to achieve, for even a detail such as the choice of the colour of a shirt is subtly persuasive — advertising would be so boring that no one would pay any attention. But perhaps that is what the well-known television personality wants.(360 words 可读性:49.8 难度:11.5级)
文章话题是广告。这篇文章主旨句显然不在第一段,而是在第三段。该段是前两段的内容的总结:广告是带来前所未有的物质上的好处的力量。后两段(4、5段)话题为与他人争辩广告的劝说作用,认为广告就是要劝另人购买。
2. 段落主题句(TOPIC SENTENCE)
除了标出篇章的主旨句外,标出各段的主题句同样重要。每一段都有一个段的中心句,也有人叫它段落大意句、段中心思想句等等。主题句是考试重点,很多题目都是围绕着段中心设计的。确定主题句不仅有利于确定文章主旨,还有利读懂细节题,因此主题句的确定是阅读的关键。
主题句的特点:1)通常是第一句话,偶尔是最后一句,段中少见。2)是观点(不是描写/说明、不是事实)。可能是作者的观点,也可能是他人的观点。3)该观点可能是作者提倡的,也可能是作者认为其他人的观点是错误的/偏激的等等。4)作者的观点只能有一个。主题句只能有一个,其他的内容都起说明/论证作用。5)具有归纳性、概括性、抽象性等特点。
例1
And yet, the myth of controlling the waters persists. This week, in the heart of civilized Europe, Slovaks and Hungarians stopped just short of sending in the troops in their contention over a dam on the Danube. The huge complex will probably have all the usual problems of big dams. But Slovakia is bidding for independence from the Czechs, and now needs a dam to prove itself. (可读性:68.6 难度:7.8级)
Q:What is the myth concerning giant dams?
[A] They bring in more fertile soil.
They help defend the country.
[C] They strengthen international ties.
[D] They have universal control of the waters.
解析:主题句为①“控制该河段水域的神话依然继续着。”下面的句子是两个个体例子说明主题句。所以[D]“他们有对河流水域的完全控制权”正确。
例2
You can begin to think of yourself as truly intelligent on the basis of how you choose to feel in the face of trying circumstances. The life struggles are pretty much the same for each of us. Everyone who is involved with other human beings in any social context has similar difficulties. Disagreements, conflicts and compromises are a part of what it means to be human. Similarly, money, growing old ,sickness, deaths, natural disasters and accidents are all events which present problems to virtually all human beings. But some people are able to make it , to avoid immobilizing depression and unhappiness despite such occurrences, while others collapse or have an N.B.D. Those who recognize problems as a human condition and don’t measure happiness by an absence of problems are the most intelligent kind of humans we know; also,the most rare. (可读性:51.4 难度:11.1级)
1. In the paragraph, the author tells us that
[A] difficulties are part of everyone’s life.
depression and unhappiness are unavoidable in life.
[C] everybody should learn to avoid trying circumstances.
[D] good feelings can contribute to eventual academic excellence.
2. According to the passage, what kind of people are rare?
[A] Those who don’t emphasize bookish excellence in their pursuit of happiness.
Those who are aware of difficulties in life but know how to avoid unhappiness.
[C] Those who measure happiness by an absence of problems but seldom suffer from N.B.D.’S.
[D] Those who are able to secure happiness though having to struggle against trying circumstances.
分析:①为主题句“知道如何面对逆境才是真正的聪明人。”②~⑦支持主题句:②③④⑤说“对于所有人来说困难和矛盾是人生的不可避免的一部分”。⑥但有些人成功地避免了不幸,而有些人却精神崩溃。⑦重申主题句。“那些认识到困难是人生必然存在的事物,不以有无困难来衡量愉快与否,是最聪明的人,也是很罕见的人。”答案:1.[A] 2.

3. 题干与选项中的关键词
仔细读考题,从考题中找出关键词,目的是确定考的是什么,考文章中的哪一段、哪一句。常见的关键词包括题干中重要的普通名词或专有名词,形容词、副词的最高级,四个选项中相同、相近词或者相关内容。在这些关键词的指引下寻找对应的句子或段落。有的时候关键词可以是各种词类,非常隐蔽,需要非常细心才能找到。
例1
“I have great confidence that by the end of the decade we’ll know in vast detail how cancer cells arise,” says microbiologist Robert Weinberg, an expert on cancer. “But ,” he cautions, “some people have the idea that once one understands the causes, the cure will rapidly follow. Consider Pasteur. He discovered the causes of many kinds of infections, but it was fifty or sixty years before cures were available.”(可读性:57.3 难度:9.6)
Q: The example of Pasteur in the passage is used to
[A] predict that the secret of cancer will be disclosed in a decade.
indicate that the prospects for curing cancer are bright.
[C] prove that cancer will be cured in fifty to sixty years.
[D] warn that there is still a long way to go before cancer can be conquered.(实考试题)
解析:Pasteur是关键词,在文章中先找到Pasteur很容易,这样就确定了所考的段落。由于篇幅所限,本文没有引证所有的段落(请参看1994年全国统考试题Passage4)。纵读全文,大家会进一步体会到这种方法的好处的。正确理解提到Pasteur的相关句子便可知道:治愈癌症还要等很多年。所以答案是[D]。
例2
Cars account for half the oil consumed in the U.S., about half the urban pollution and one fourth the greenhouse gases. They take a similar toll of resources in other industrial nations and in the cities of the developing world. As vehicle use continues to increase in the coming decade, the U.S. and other countries will have to deal with these issues or else face unacceptable economic, health-related and political costs. It is unlikely that oil prices will remain at their current low level or that other nations will accept a large and growing U.S. contribution to global climatic change. (可读性:47.47 难度:12级)
Q:From the passage we know that the increased use of cars will
[A] consume half of the oil produced in the world.
have serious consequences for the well-being of all nations.
[C] widen the gap between the developed and developing countries.
[D] impose an intolerable economic burden on residents of large cities.
解析:关键词是increased use of cars, 并且和③中vehicle use continues to increase相呼应,确定考点在此句。③“美国和其他国家将不得不做出选择,或者处理这些问题,或者面临不堪设想的经济、健康以及政治后果。”可见答案为。
四、正确选项和错误选项的特点
   在本书中,正确的选项称为解,错误的选项称为干扰项。了解这两种选项的特点就是要了解命题者的思路。命题者都是英语水平相当高的英语专家,专门研究过英语测试理论,有丰富的命题经验,对中国考生的英语水平和特点有非常深的了解。这些特点,是外国专家所难以具备的。
出题者命题碰到的第一个问题是选材。文章的来源决不可能是学生学过的教材,或者市场上出售的教辅和模拟题。他们要从考生不易得到的、不熟悉的、不经常读的材料中选择。这些材料包括英语国家的报刊杂志,以及众多的普通读物。命题要考虑到文章要有一定的难度,对于话题考生不能太陌生,当然太熟悉也不行。文章要有一定的完整性,即使是从一篇长文章摘下来的一部分,其本身也要相对独立,要有中心思想等。然后就是确定题眼(考点),一篇文章只考5道题,不可能所有内容都考,这就要确定考什么。总的原则是文章哪里难就考哪。特别是中国学生不易理解之处就可能成为考点。常考的包括长难句的理解、逻辑推测能力以及段落或文章的中心思想等。考点确定之后,就是写解和设置干扰项。两者的特点如下:
1. 正确选项的特点
设置正确选项(解)的一个常用方法就是同义替换或释义(paraphrase),即把文中语言改头换面,运用与原文不同的结构、不同的词汇,来表达相同或类似的意思。常见的形式有:
1)正确选项大量使用原文的同义词或同义结构
例1
Useful as half-sleeping might be, it’s only been found in birds and such water mammals as dolphins, whales, and seals. Perhaps keeping one side of the brain awake allows a sleeping animal to surface occasionally to avoid drowning. (可读性:56.8 难度:10.1)
Q:While sleeping, some water mammals tend to keep half awake in order to
[A] alert themselves to the approaching enemy.
emerge from water now and then to breathe.
[C] be sensitive to the ever-changing environment.
[D] avoid being swept away by rapid currents.(实考试题)
解析:解是②的改写,原文和答案词汇比较如下:

答案中的词汇实际上是文中词汇的英语释义。答案是。
例2
We live in a society in which the medicinal and social use of substances (drugs) is pervasive: an aspirin to quiet a headache, some wine to be sociable, coffee to get going in the morning, a cigarette for the nerves. When do these socially acceptable and apparently constructive uses of a substance become misuses? First of all, most substances taken in excess will produce negative effects such as poisoning or intense perceptual distortions. Repeated use of a substance can also lead to physical addiction or substance dependence. Dependence is marked first by an increased tolerance, with more and more of the substance required to produce the desired effect, and then by the appearance of unpleasant withdrawal symptoms when the substance is discontinued. (可读性:42.4 难度:12级)
Q:Physical dependence on certain substances results from
[A] uncontrolled consumption of them over long periods of time.
exclusive use of them for social purposes.
[C] quantitative application of them to the treatment of diseases.
[D] careless employment of them for unpleasant symptoms.(实考试题)
解析:解是④的改写,原文和答案词汇与结构比较如下:

解中词汇比文中词汇难而且长。答案是[A]。

例3
There is, as Robert Rubin, the treasury secretary, says, a “disjunction” between the mass of business anecdote that points to a leap in productivity and the picture reflected by the statistics. (可读性:37.8 难度:12级)
Q:The official statistics on productivity growth
[A] exclude the usual rebound in a business cycle.
fall short of businessmen’s anticipation.
[C] meet the expectation of business people.
[D] fail to reflect the true of economy.
解析:原文和答案词汇与结构比较如下:

答案词汇比文中词汇简单,主要考查对原文词汇是否真正理解了。答案是。

2)正确选项频繁使用原文的反义词加上反义结构来表达与原文相同的意思,如:
He is old. He is no longer young. 句子结构一个是肯定形式,一个是否定形式,考查考生对这两种结构的理解。

Experts suggest that speech stages are reached in a fixed sequence and at a constant age, but there are cases where speech has started late in a child who eventually turns out to be of high IQ. At twelve weeks a baby smiles and makes vowel-like sounds; at twelve months he con speak simple words and understand simple commands; at eighteen months he has a vocabulary of three to fifty words. At three he knows about 1,000 words which he can put into sentences, and at four his language differs from that of his parents in style rather than grammar. (可读性:73.2 难度:8.0级)
Q:If a child starts to speak later than others, he will
[A] have a high IQ.
be less intelligent.
[C] be insensitive to verbal signals.
[D] not necessarily be backward.(实考试题)
解析:解是①后半句双重否定的改写,原文和答案词汇比较如下:

答案是[D]

3)正确选项是原文的总体或局部
总体和局部(或称一般和特殊)的关系主要表现为:一般是特殊的总和,特殊为一般的属性。这种题型考查类比推理能力。

There are two basic ways to see growth: one as a product, the other as a process. People have generally viewed personal growth as an external result or product that can easily be identified and measured. The worker who gets a promotion, the student whose grades improve, the foreigner who learns a new language — all these are examples of people who have measurable results to show for their efforts. (可读性:51.9 难度:11.6级)
Q: A person is generally believed to achieve personal growth when
[A] he has given up his smoking habit.
he has made great efforts in his work.
[C] he is keen on learning anything new.
[D] he has tried to determine where he is on his journey. (实考试题)
解析:题干中generally是关键词,与②中generally相对。②:“人们通常认为个人的成长是外在的成果或产物,是容易辨别和衡量的”,正确答案必须符合“外在”、“易辨别和衡量”这两个一般特征。[A]“当一个人戒了烟”,烟戒了是一种易辨别的、外在的“成果”,与“升职”等相似,属于特殊的情况。故[A]正确。

4)正确选项常常是原文长难句的简单化解释
难句是所有考试必考的内容。难句之所以难有三方面的原因:A)句子结构复杂;表达抽象;C)词难。所以很多题目都围绕着难句做文章,通常使用简单的结构或词汇对难句做浅显、具体的解释。
例1
Whether the eyes are “the windows of the soul” is debatable, that they are intensely important in interpersonal communication is a fact. During the first two months of a baby’s life, the stimulus that produces a smile. Significantly, a real human face with eyes covered will not motivate a smile, nor will the sight of only one eye when the face is presented in profile. This attraction to eyes as opposed to the nose or mouth continues as the baby matures. In one study, when American four-year-olds were asked to draw people, 75 percent of them drew people with eyes. In Japan, however, where babies are carried on their mother’s back, infants do not acquire as much attachment to eyes as they do in other cultures. As a result, Japanese adults make little use of the face either to encode or decode meaning. In fact, Argyle reveals that the “proper place to focus one‘s gaze during a conversation in Japan is on the neck of one’s conversation partner.” (可读性:58.1 难度:10.6级)
Q: Babies will not be stimulated to smile by a person
[A] whose front view is fully perceived.
whose face is covered with a mask.
[C] whose face is seen from the side.
[D] whose face is free of any covering.
解析:本题考查对④的理解。特别是presented in profile。profile这里的词义是“侧面像”,所以[C]whose face is seen from the side“从侧面看去”正确。
例2
But my own worry today is less that of the overwhelming problem of elemental literacy than it is of the slightly more luxurious problem of the decline in the skill even of the middle-class reader, of his unwillingness to afford those spaces of silence, those luxuries of domesticity and time and concentration, that surround the image of the classic act of reading. It has been suggested that almost 80 percent of America’s literate, educated teenagers can no longer read without an accompanying noise (music) in the background or a television screen flickering at the corner of their field of perception. We know very little about the brain and how it deals with simultaneous conflicting input, but every commonsense intuition suggests we should be profoundly alarmed. This violation of concentration, silence, solitude goes to the very heart of our notion of literacy; this new form of part-reading, of part-reading, of part-perception against background distraction, renders impossible certain essential acts of apprehension and concentration, let alone that most important tribute any human being can pay to poem or a piece of prose he or she really loves, which is to learn it by heart. Not by brain, by heart; the expression is vital. (可读性:35.7 难度:12级)
1.The author’s biggest concern is
[A] elementary school children’s disinterest in reading classics.
the surprisingly low rate of literacy in the U.S.
[C] the musical setting American readers require for reading.
[D] the reading ability and reading behavior of the middle class.
2.A major problem with most adolescents who can read is
[A] their fondness of music and TV programs.
their ignorance of various forms of art and literature.
[C] their lack of attentiveness and basic understanding.
[D] their inability to focus on conflicting input.
解析:
1.本题考查对①的理解。该句长达62个词。全句主干结构为my worry…is less that of…than…“与其说……不如说……”。作者说“我虽然担心初级读者在读写方面存在的问题,但我更担心的是中等阶层读者阅读技能的问题”。接着作者描述了问题所在,从of his unwillingness开始,作者说“他们不能在安静的环境下读书,不能专心……”。所以[D]“中等阶层的阅读能力和阅读行为”是整句的概括。
2.本题考查对倒数第二句的理解,该句长达66个词,有两个分句,用分号隔开。第一个分句大意为:这种阅读时不专心、不安静、不独处的情况是读写最大的问题。go to the heart of 是固定短语,意为“(问题)最重要的方面”。第二个分句更复杂,难点为作者把render sth. +adj.(使什么怎么样)句型倒装,形容词提前,名词退后,因为名词部分较长。难词有apprehension“理解”,pay tribute to “崇敬”。大意为:这种伴随着背景音乐边读边理解的阅读方式是不可能做到真正理解和专心致志的,更不用说我们欣赏诗歌时还需要的崇敬之情了。所以[C]“他们不专心,缺乏基本的理解”是两个分句的综合,是答案。
5)正确选项是对原文引语的解释
文章的引语通常较难,因为作者之所以引述他人的话是因为自己的话无论怎么说都没有他人说得准确、说得好。引语的目的不是夸耀他人,而是用来论证说明自己的观点。引语是经常考查的内容,题干形式有两种:The author quotes…because…;the author quotes…to illustrate…

“Creative thinking may mean simply the realization that’s no particular virtue in doing things the way they have always been done,”wrote Rudolph Flesch, a language authority, this accounts for our reaction to seemingly simple innovations like plastic garbage bags and suitcases on wheels that make life more convenient:“How come nobody thought of that before?”
The creative approach begins with the proposition that nothing is as it appears. Innovators will not accept that there is only one way to do anything. Faced with getting from A to B, the average person will automatically set out on the best-known and apparently simplest route. The innovator will search for alternate courses, which may prove easier in the long run and are bound to be more interesting and challenging even if they lead to dead ends.(可读性:473 难度:12级)
Q:The author quotes Rudolph Flesch in paragraph 1 because
[A] Rudolph Flesch is best-known expert in the study of human creativity.
the quotation strengthens the assertion that creative individuals look for new ways of doing things.
[C] the reader is familiar with Rudolph Flesch’s point of view.
[D] the quotation adds a new idea to the information previously presented.(实考试题)
解析:这句引语含义:“创造性的思维也许仅仅意味着认识到按常规做事并没有什么不寻常的素质”,下一段的前两句说“创造性思维起始于这样一个命题:一切东西都不是它表现出来的那样。革新家们认为做任何事都不是只有一种方法。”将这两点综合考虑可知,作者引用Rudolph只不过是为了进一步证明自己的观点,即创新者总是寻找做事的新方法,这与完全一致。
正确选项的下面的几个特点是考研英语的重点和难点。这些特点总的特征是答案具有高度概括性、归纳性、总结性、抽象性和推理性。这或许就是考研英语为什么比六级难的原因。
6)正确选项是文中例证或事实的归纳总结
例1
Given all these disadvantages, central bankers seem to have had much to boast about of late. Average inflation in the big seven industrial economies fell to a mere 2.3% last year, close to its lowest level in 30 years, before rising slightly to 2.5% this July. This is a long way below the double-digit rates which many countries experienced in the 1970s and early 1980s.
It is also less than most forecasters had predicated. In late 1994 the panel of economists which The Economist polls each month said that America’s inflation rate would average 3.5% in 1995. In fact, it fell to 2.6% in August, and expected to average only about 3% for the years as a whole. In Britain and Japan inflation is running half a Percentage point below the rate predicted at the end of last year. This is no flash in the pan; over the past couple of years, inflation has been consistently lower than expected in Britain and America.(可读性:49 难度:10.9级)
Q:From the passage we learn that
[A] there is a definite relationship between inflation and interest rates.
economy will always follow certain models.
[C] the economic situation is better than expected.
[D] economists had foreseen the present economic situation.(实考试题)
解析:综合推断题。这两段并不难,有大量数据,问题考查从这些数据可以归纳出什么。第二句说通货膨胀率降到了2.3%;第三句:这(低膨胀率)比多数经济学家预测的还要低,综合这一切可以推断:经济形势比预计的要好。故[C]正确。
例2
One hundred and thirteen million Americans have at least one bank-issued credit card. They give their owners automatic credit in stores, restaurants, and hotels, at home, across the country, and even abroad, and they make many banking services available as well. More and more of these credit cards can be read automatically, making it possible to withdraw or deposit money in scattered locations, whether of not the local branch bank is open. For many of us the “cashless society”is not on the horizon-it’s already here.(可读性:45.3 难度:12级)
Q:According to the passage, the credit card enables its owner to
[A] withdraw as much money from the bank as he wishes.
obtain more convenient services than other people do.
[C] enjoy greater trust from the storekeeper.
[D] cash money wherever he wishes to.(实考试题)
解析:对②、③概括,信用卡使持卡者比其他人享有更多的便利服务,可知正确。[A]、[D]两项均过于绝对,且与文意不符;[C]“持卡者得到店主更大的信任”文中未提及,也应排除。
例3
For a long period of time and in many parts of the country,a traveler was a welcome break in an otherwise dull existence. Dullness and loneliness were common problems of the families who generally lived distant from one another. strangers and travelers were welcome sources of diversion, and brought news of the outside world.(可读性:54.2 难度:10.2级)
Q:Families in frontier settlements used to entertain strangers
[A]to improve their hard life.
in view of their long-distance travel.
[C]to add some flavor to their own daily life.
[D]out of a charitable impulse.(实考试题)
解析:大意为:过去很长一段时间,在美国很多地方,旅行者的到来对居民们单调生活是一种可喜的调节。无聊、孤独对相距很远的家庭来说是普遍存在的问题。陌生人和旅行者给他们的生活带来了娱乐消遣,因而受欢迎;同时还带来了外面世界的信息。可见,边远地区的人们对陌生人(游客)如此热情是因为这给他们的生活增添了乐趣,所以[C]正确。[A]“为了改善他们的艰苦生活”言过其实;“于由游客漫长的旅途”不够确切。
7)正确选项是对段落中心/大意,或者篇章中心的归纳总结
例1
Few people doubt the fundamental importance of mothers in child rearing, but what do fathers do? Much of what they contribute is simply the result of being a second adult in the home. Bringing up children is demanding, stressful and exhausting. Two adults can support and make up for each other’s strengths.(可读性:65.7 难度:7.7级)
Q:The paragraph points out that one of the advantages of a family with both parents is
[A] husband and wife can share house work.
two adults are always better than one.
[C] the fundamental importance of mothers can be fully recognized.
[D] husband and wife can compensate for each other’s shortcoming.
解析:本段的话题是子女抚养中母亲或父亲的作用,如果双方互相支持,可以弥补对方的缺陷。可见[D]“夫妇可互相补偿彼此缺点”是段落中心的概括,符合题意。[A]“夫妇可共同做家务劳动”和 “两个总比一个好”都是根据生活常识编造的选项(合理项,参见本书“错误选项的特征”一节);[C]“母亲的重要性可以得到充分认可”,表达片面。
例2
Levin would not comment on the debate last week, but there were signs that the chairman was backing off his hard line-stand, at leas to some extent. During the discussion of rock singing verses at last month’s stockholders’ meeting, Levin asserted that “music is not the cause of society’s ills” and even cited his son, a teacher in the Bronx, New York, who uses rap to communicate with students. But he talked as well about the “balanced struggle” between creative freedom and social responsibility, and he announced that the company would launch a drive to develop standards for distribution and labeling of potentially objectionable music. (可读性:39.2 难度:12级)
Q: In face of the recent attacks on the company, the chairman
[A] stuck to a strong stand to defend freedom of expression.
softened his tone and adopted some new policy.
[C] changed his attitude and yielded to objection.
[D] received more support from the 15-member board.
解析:①由于上周发生的争执,Levin可能放弃强硬立场。③Levin宣布,公司将尽力对可能招致人们反对的音乐制定各种发行和标识的标准。综合这两镍可推断“Levin缓和了语气并采取一些新政策”正确。[A]正反倒置,错误;[C]说的程度太过了,Levin还不至于会改变态度,屈服于对方;[D]论据不足。

8)正确选项是原文某段文字的推理

Kitcher is a philosopher, and this may account, in part, for the clarity and effectiveness of his arguments. The non-specialist will be able to obtain at least a notion of the sorts of data and argument that support evolutionary theory. The final chapter in the creationists will be extremely clear to all. On the dust jacket of this fine book, Stephen Jay Gould says: “This book stands for reason itself.” And so it does — and all would be well were reason the only judge in the creationism/ evolution debate. (可读性:57.2 难度:9.6级)
Q: From the passage we can infer that
[A] reasoning has played a decisive role in the debate.
creationists do not base their argument on reasoning.
[C] evolutionary theory is too difficult for non-specialists.
[D] creationism is supported by scientific findings.
解析:本段意思是:金切尔是位哲学家,这也许能部分说明他的立论所以明确而有说服力。非专业人士起码可以了解支持进化论的各种数据和观点。关于神造论者的最后一章对每个人来说都阐述得极为清楚。这部优秀作品的护封上引用了斯蒂芬.杰.古尔德的一句话,“本书代表的是理性”。的确——如果理性是神造论和进化论之争的惟一裁判,一切问题就已解决了。最后一句是个带有倒装结构的虚拟语气句子,转换成正常语序是:If reason were the only judge in the creationism/evolution debate, all would be well. 推断神造论是非理性的,故正确。

2. 干扰项的特点
错误选项,也叫干扰项,也戏称陷阱,是出题者(中国出题专家)的拿手好戏。对错选项的分析从某种角度来说比正确选项更有意义。因为在实际做题时,考生做的大部分工作是排除三个错误选项。有时问题集中在两个选项上,考生明明知道答案就在这两个选项中,可是很难判断究竟哪个是错的。
总的来说,干扰项从语言和内容两方面欺骗考生。虽然花样很多,但总是有些规律可循的。出题者在编错误选项时,就像一个大骗子,试想骗子的伎俩有哪些呢?首先是把假的说成真的,所谓以假乱真,其次是真真假假,虚虚实实,所谓掉包,再次是夸大其词,或以偏概全,一会儿把苹果说成灵丹妙药,一会儿又说灵丹妙药根本不存在。对干扰项的对策有以下7种:
1. 合理项:文章中没提到,利用生活常识编造出来的选项同。对策:做题不可凭空想象,一切都要从文中找依据。在相当多的情况下“项不是解”。
2. 断章取义:使用文章中出现的词语或类似的结构仿造。对策:原词越多,对的可能性越小。
3. 小动作:错误选项和原文句子几乎一样,但在细微之处做了手脚,改动了几个小词,使得意思和原文不相符。对策:仔细,仔细,再仔细!不仅要看大意,更要看细节。
4. 以偏概全:就是用文章中的细节片面的、次要的观点回答问题。对策:时刻要有一种博大的胸襟。
5. 张冠李戴(把一个事物的特征说成是另一个事物的特征,把他人的观点说成是作者的观点)、正反倒置(把作者观点的反面说成是作者的观点)、因果倒置(把因说成果,把果说成因)。对策:头脑清楚。
6. 过度引申:考研测试推理能力是很正常的,但有相当多的选项,过度推理,超出文章的范围。对策:掌握好分寸。
7. 字面意思:被测试的单词或短语,实际(深层)含义和表面上个别词汇的意思不同。出题者经常用表面意思作为干扰项。对策:字面意思不是解。

Aimlessness has hardly been typical of the postwar Japan whose productivity and social harmony are the envy of the United States and Europe. But increasingly the Japanese are seeing a decline of the traditional work-moral values. Ten years ago young people were hardworking and saw their jobs as their primary reason for being, but now Japan has largely fulfilled its economic needs, and young people don’t know where they should go next.
The coming of age of the postwar baby boom and an entry of women into the male-dominated job market have limited the opportunities of teen-agers who are already questioning the heavy personal sacrifices involved in climbing Japan’s rigid social ladder to good schools and jobs. In a recent survey, it was found that only 24.5 percent of Japanese students were fully satisfied with school life, compared with their jobs than did their counterparts in the 10 other countries surveyed.
While often praised by foreigners for its emphasis on the basics, Japanese education tends to stress test taking and mechanical learning over creativity and self-expression. “Those things that do not show up in the test scores — personality, ability, courage or humanity — are completely ignored,” says Toshiki Kaifu, chairman of the ruling Liberal Democratic Party’s education committee. “Frustration against this kind of thing leads kids to drop out and run wild.” Last year Japan experienced 2,125 incidents of school violence, including 929 assaults on teachers. Amid the outcry, many conservative leaders are seeking a return to the prewar emphasis on moral education. Last year Mitsuo Setoyams, who was then education minister, raised eyebrows when he argued that liberal reforms introduced by the American occupation authorities after World War Ⅱ had weakened the “Japanese morality of respect for parents.”
But that may have more to do with Japanese life-styles. “In Japan,” says educator Yoko Muro, “it’s never a question of whether you enjoy your job and your life, but only how much you can endure.” With economic growth has come centralization; fully 76 percent of Japan’s 119 million citizens live in cities where community and the extended family have been abandoned in favor of isolated, two generation households. Urban Japanese have long endured lengthy commutes (travels to and from work) and crowded living conditions, but as the old group and family values weaken, the discomfort is beginning to tell. In the past decade, the Japanese divorce rate, while still well below that of the United States, has increased by more than 50 percent, and suicides have increased by nearly one-quarter. (可读性:34.9 难度:12级)
1. In the Westerner’s eyes, the postwar Japan was
[A] under aimless development. a positive example.
[C] a rival to the West. [D] on the decline.
2. According to the author, what may chiefly be responsible for the moral decline of Japanese society?
[A] Women’s participation in social activities is limited.
More workers are dissatisfied with their jobs.
[C] Excessive emphasis has been placed on the basics.
[D] The life-style has been influenced by Western values.
3. Which of the following is true according to the author?
[A] Japanese education is praised for helping the young climb the social ladder.
Japanese education is characterized by mechanical learning as well as creativity.
[C] More stress should be placed on the cultivation of creativity.
[D] Dropping out leads to frustration against test taking.
解析:
1. <1 >①“缺乏目标从来就不是战后日本的典型特征,日本国的生产率和社会的和谐令欧美人羡慕”,可见战后日本是一个正面榜样,故正确。[A]小动作:把 <1 >①关键词保留下来,但hardly去掉了,结果意思大相径庭,原文的否定变成了肯定;[C]字面意思:envy(羡慕,嫉妒)的字面意思;[D]断章取义:使用②中“decline”编造。
2. 题干关键词为chiefly 和be responsible for。 <3 >末句:日本教育大臣说过,二战后占领日本的美国当局进行自由化改革削弱了日本人尊敬父母的道德观。 <4 >①:但是,这(道德)也计与日本人的生活方式关系更大。将两句联在一起看,作者认为日本社会道德下降的主要原因是[D]“生活方式受了西方价值观的影响”,故[D]正确。[A]正反倒置。 <2 >①提到女性进入男性占主导地位的人才市场,和[A]正好相反,而且[A]也是符合生活常识的合理项。以偏概全。借用了 <2 >末句的完整意思,但是该句不是观点,而是一次调查结果,是具体的事实,并非全部。[C]以偏概全。 <3 >①:由于重视基础知识,所以忽视了其他方面的教育。 <4 >①:但是,这(道德下降)也许与日本人的生活方式关系更大。所以[C]是次要原因。
3. <3 >①:“尽管由于强调基础教育而受到外国人的赞赏,但是日本教育往往强调应试和机械性学习而忽视创造性及自我表现”,作者暗示日本教育更应该强调创造能力的培养,所以[C]正确。[A]断章取义。将 <2 >①中“climbing…social ladder”和 <3 >①中“often praised by foreigners for”拼在一起唬人。小动作。将 <3 >①中over改成as well as。失之毫厘,谬以千里。[D]因果倒置。 <3 >③:考试失败导致学生辍学。[D]颠倒了因果关系。
五、题型
1.    主旨题
此类题型分为3种:1)中心思想:考查整篇文章的主旨、中心意思、核心观点;2)段落中心:即某一段的大意;3)谈论话题:问全篇讨论的话题是什么。一篇文章的中心思想通常出现在第一段,有时在末段,偶尔在中间的某一段。常见主旨题的形式:The passage is mainly about…; The main idea of the passage is that…; Which of the following can best sum up the passage?
例1
Why do so many Americans distrust what they read in their newspapers? The American Society of Newspaper Editors is trying to answer this painful question. The organization is deep into a long self-analysis known as the journalism credibility project. (可读性:33.1    难度:11.8)
Q: What is the passage mainly about?
[A] Needs of the readers all over the world.
Causes of the public disappointment about newspapers.
[C] Origins of the declining newspaper industry.
[D] Aims of a journalism credibility project.
解析:首段主题句用的是疑问句,变为肯定句就是“很多美国人不相信报纸上所读到的东西的原因。”所以正确。
例2
When a new movement in art attains a certain fashion, it is advisable to find out what its advocates are aiming at, for, however farfetched and unreasonable their principles may seem today, it is possible that in years to come they may be regarded as normal. With regard to Futurist poetry, however, the case is rather diffcult, for whatever Futurist poetry may be — even admitting that the theory on which it is based may be right — it can hardly be classed as Literature.
This, in brief, is what the Futurist says: for a century, past conditions of life have been conditionally speeding up, till now we live in a world of noise and violence and speed. Consequently, our feelings, thoughts and emotions have undergone a corresponding change, this speeding up of life, says the Futurist, requires a new form of expression. We must speed up our literature too, if we want to interpret modern stress. We must pour out a large stream of essential words, unhampered by stops, or qualifying adjectives, of finite verbs. Instead of describing sounds we must make up words that imitate them; we must use many sizes of type and different colored inks on the same page, and shorten or lengthen words at will.
Certainly their descriptions of battles are confused. But it is a little upsetting to read in the explanatory notes that a certain line describes a fight between a Turkish and a Bulgarian officer on a bridge off which they both fall into the river and then to find that the line consists of the noise of their falling and the weights of the officers: ‘Pluff! Pluff! A hundred and eighty-five kilograms.’
This, though it fulfills the laws and requirements of Futurist poetry, can hardly be classed as Literature. All the same, no thinking man can refuse to accept their first proposition: that a great change in our emotional life calls for a change of expression. The whole question is really this: have we essentially changed?
Q: This passage is mainly
[A] a survey of new approaches to art.
a review of Futurist poetry.
[C] about merits of the Futurist movement.
[D] about laws and requirements of literature.
解析:纵观全文, <1 >点出了未来主义诗歌这个话题,并且表明观点:未来主义诗歌不能被看作文学, <1 >、 <2 >作者详细地剖析了未来主义诗人的诗, <4 >重申未来主义诗歌不能看作文学,可见,文章是对未来主义诗歌党派的总体评价。因此“未来主义诗歌述评”正确。本文论述的核心是Futurist poetry,文中提到最多的就是Futurist poetry,反复提到的词是话题。可以排除[A]、[D]。[A]“新艺术方法概览”和[D]“文学的规律和要求”范围太宽,与文章中心不符。[C]含有该词,但错在movement这个词上。



2.    细节题
这类题在总题量中占相当大的比例,大约80%,而主旨题占20%。有的同学可能会问难道没有别的题型了?其实题型这种东西讲来讲去最没意思,很多题型之间的关系是重叠的,划分不很清楚。很多时候是对同一问题不同角度的描述。比如说,某推断题针对文章的细节进行推断,我们既可以说它是细节题也可以说它是推断题。态度题如果是对全文的态度很可能又属于主旨大意题,如果是针对文中某一具体事实,很可能属于细节题。所谓细节,就是说明文章主题特别是说明论证段落大意,作者使用的具体信息。细节题最主要的特征是针对文章中的一句或两句的局部提问。解题方法为:1)仔细读题,确定问题的关键词/词组;2)以此为线索,确定是哪一段;3)确定哪一句(或两句);4)仔细研读这1~2句;5)和选项对照阅读,确定答案。
例1
Researchers in the field of psychology have found that one of the best ways to make an important decision, such as choosing a university to attend or a business to invest in, involves the utilization of a decision worksheet. Psychologists who study optimization compare the actual decisions made by people to theoretical ideal decisions to see how similar they are. Proponents of the w**dure believe that it will yield optimal, that is, the best decisions. Although there are several variations on the exact format that worksheets can take, they are all similar in their essential aspects. W**quire defining the problem in a clear and concise way and then listing all possible solutions to the problem. Next, the pertinent considerations that will be affecteb by each decision are listed, and the relative importance of each consideration or consequence is determined. Each consideration is assigned a numerical value to reflect its relative importance. A decision is mathematically calculated by adding these values together. The alternative with the highest number of points emerges as the best decision. (可读性:33.1  难度:12级)
1.    Of the following steps, which occurs before the others in making a decision worksheet?
[A] Listing the consequences of each solution.
Calculating a numerical summary of each solution.
[C] Deciding which consequences are most important.
[D] Writing down all possible solutions.
2.  According to decision-worksheet theory, an optimal decision is defined as one that
[A] has the fewest variables to consider.
uses the most decision worksheets.
[C] has the most points assigned to it.
[D] is agreed to by the greatest number of people.
解析:
1.    本题剖制定工作记录单,所给的四个任务哪一个应该先做?这是一道排序题,关键是在文中找到表示顺序的信号词。从w**quire开始,本段叙述了制定工作记录单的各个步骤,信号词为next,所以答案应是next前面的一句话,在前一句中提到了两件事,用then表示次序。首先是定义问题,然后是列出问题的所有可能的解决方案。所以[D]是答案。
2.    本题问根据制定工作记录单理论,最佳决策的定义是什么?根据最后一句“具有最高点数的选择就是最好的决策。”判断[C]“被赋予了最多的点数”是答案。其他三个选项均是假信息,文章未提到。
例2
Likewise, if you want to find a job, take a sheet of paper, and write a brief account of yourself. In making a blueprint for a job ,begin with yourself, for when you know exactly what you have to offer, you can intelligently plan where to sell your services. (可读性:68.0  难度:9.8级)
Q:  According to the passage, one must write an account of himself before starting to find a job because
[A] that is the first step to please the employer.
that is the requirement of the employer.
[C] it enables him to know when to sell his services.
[D] it forces him to become clearly aware of himself.
解析:题干中关键词为because, 与②中for呼应。②“因为当你准确知道你能(为未来的雇主)做些什么时,你才能正确地计划到哪儿去谋职。”可见,找工作前写一份个人材料是为了更加明确地了解自己,符合[D]。[A]、均与文章不符,[C]错在“when”字上。
例3
Today there are many charitable organizations which specialize in helping the weary traveler. Yet, the old tradition of hospitality to strangers it still very strong in the US, especially in the smaller cities and towns away from the busy tourist trails. “I was just traveling through, got talking with this American, and pretty soon he invited me home for dinner — amazing.” Such observations reported by visitors to the US are not uncommon, but are not always understood properly. The casual friendliness of many Americans should be interpreted neither as superficial nor as artificial, but as the result of a historically developed cultural tradition. (可读性:31.5   难度:12级)
Q: The tradition of hospitality to strangers
[A] tends to be superficial and artificial.
is generally well kept up in the United States.
[C] is always understood properly.
[D] was something to do with the busy tourist trails.
解析:②“这种好客传统在美国仍然保持得很好”,可见正确。[A]与末句明显相反[C]与④、⑤不一致,因为作者在这里强调的就是“来美国的游客中碰到这类事(美国人好客)的人不在少数,但对此并非都能正确理解”;[D]与②恰好相反。
在细节题型中有一种是非题型,可分为三正一误题型(EXCEPT题型)和三误一正题型。形式如下:
Which of the following is true?
Which of the following is not true?
Which of the following is true except?
引种试题多半根据文章的一段 提问,在考研试题中有时也针对文章的好几段提问,难度很大。
例4
The “shareholders” as such had no knowledge of the lives, thoughts or needs of the workmen employed by the company in which he held shares, and his influence on the relations of capital and lab our was not good. The paid manager acting for the campany was in more direct relation with the men and their demands, but even he had seldom that familiar personal knowledge of the workmen which the employer had often had under the more patriarchal system of the old family business now passing away. Indeed the mere size of operations and the numbers of workmen involved rendered such personal relations impossible. Fortunately, however, the increasing power and organization of the trade unions, at least in all skilled trades, enabled the workmen to meet on equal terms the managers of the companies who employed them. The cruel discipline of the strike and lockout taught the two parties to respect each other’s strength and understand the value of fair negotiation.
Q: According to the passage, all of the following are true except that
[A] the shareholders were unaware of the needs of the workers.
the old firm owner had a better understanding of their workers.
[C] the limited liability companies were too large to run smoothly.
[D] the trade unions seemed to play a positive role.
   解析:根据①[A]正确;根据②后半句正确;根据④[D]正确。只有[C]文章没提到,是答案。
细节题的另一种常见的形式是例子题,例子题主要问的是文章举例子的目的是什么。常见形式:The author uses the example of… to show that…例子是中心的体现,可以是段中心也可以文章中心。
例5
There is another way to commit evolutionary suicide: stay alive, but have fewer children. Few people are as fertile as in the past. Except in some religious communities, very few women has 15 children. Nowadays the number of births, like the age of death, has become average, most of us have roughly the same number of offspring. Again, differences between people and the opportunity for natural selection to take advantage of it have diminished. India shows what is happening. The country offers wealth for a few in the great cities and poverty for the remaining tribal peoples. The grand mediocrity of today — everyone being the same in survival and number of offspring — means that natural selection has lost 80% of its power in upper-middle class India compared to the tribes. (可读性:50.6   难度:9.7级)
Q:What does the example of India illustrate?
[A] Wealthy people tend to have fewer children than poor people.
Natural selection hardly works among the rich and the poor.
[C] The middle class population is 80% smaller than that of the tribes.
[D] India is one of the countries with a very high birth rate.
解析:通过例子考大意的题。问文中用印度这个例子说明什么。⑦提到了印度这个国家,⑥和⑦提到“自然淘汰”在富人与穷人之间已不起什么作用。换言之,作者举印度这个例子是为了阐明这个观点:自然选择在富人与穷人中已不起作用。故 正确。如果你根据文章中80%选择[C]的话,你太不了解命题了。

3.推理题
对于推理题,有“狭义”和“广义”两种理解方法。从广义上讲,几乎所有的考研题都是推理题,不太可能直观地告诉你答案。考研的试题给人的感觉就是:即便你在一定程度上把文章读懂了,题也做不对。考研命题追求的是对文章的深层理解和推理,所以从这个意义上讲全部的考研题都是推理题。
从狭义上讲,推理题是指题干中出现infer, imply, conclusion字样的试题,把这些试题分别称为INFER型、IMPLY型、CONCLUSION型。如果在你头脑中存在“所谓的推理题型和其他题型有本质的差别”的话,说明你对考研题理解还仪在表面阶段。

INFER型
  It can be inferred from paragraph X…针对某一段的推断题
What can be inferred from the passage? 可能是某一段,也可能是整篇文章。
IMPLY型
  The author implies that…给出线索中心词,通常是名词主语,要求选谓语。
  The author implies that the results of scientific research…
CONCLUSION型
  We can draw the conclusion from the passage that…针对全文的题型。
  例1
  NBAC members also indicated that they will appeal to privately funded researchers and clinics not to try to clone humans by body cell nuclear transfer. But they were divided on whether to go further by calling for a federal law that would impose a complete ban on human cloning. Shapiro and most members favored an appeal for such legislation, but in a phone interview, he said this issue was still“up in the air”.(可读性:53.7  难度:11)
Q:It can be inferred from the paragraph that
  [A] some NBAC members hesitate to ban human cloning completely.
  a law banning human cloning is to be passed in no time.
  [C] privately funded researchers will respond positively to NBAC’s appeal.
  [D] the issue of human cloning will soon be settled.
解析:推断题。③“Shapiro和大多数成员赞成请求这项立法。”推断出少数人反对,所以选项[A]“有些NBAC成员对全面禁止克隆人持犹豫的态度”正确。这是一个“反着考”的典范,考研题实际上一半有“反着考”的味道。
  例2
  As is true of any developed society, in America a complex set of cultural signals, assumptions, and conventions underlies all social interrelationships. And, of course, speaking a language does not necessarily mean that someone understands social and cultural patterns. Visitors who fail to “translate” cultural meanings properly often draw wrong conclusion. For example, when an American uses the word“friend”, the cultural implications of the word may be quite different from those it has in the visitor’s language and culture. It takes more than a brief encounter on a bus to distinguish between courteous convention and individual interest. Yet, being friendly is a virtue that many American value highly and expect from both neighbors and strangers.(可读性:36.2  难度:12级)
Q:It could be inferred from the paragraph that
[A] culture exercises an influence over social interrelationships.
courteous convention and individual interest are interrelated.
[C] various virtues manifest themselves exclusively among friends.
[D] social interrelationships equal the complex set of cultural conventions.
解析:此题虽名为“推断题”,但仔细看看,推理的味道却不多。①“与任何发达国家一样,在美国一整套复杂的文化特征、观念和习俗构成了社会关系的基础。”对其换一种说法,则成为[A]“文化影响着社会上的人际关系”。Underlie:构成……的基础。其他选项均不合文意。
4.态度题
常见形式:The author’s attitude towards…can be best summarized as…
         The tone of this passage is best described as…
有可能作解的词汇:
1.积极类:objective(客观的);concerned(关注的);confident(有信心的);interested(感兴趣的);optimistic(乐观的);positive(正面的);impressive(给人以深刻印象的)
2.中立/折衷类:impartial(公正的);neutral(中立的);impersonal(非个人的);factual(根据事实的);detached(不含个人偏见的)
3.否定:doubtful(不相信的);critical(批判的);suspicious(可疑的);compromising(妥协的);worried(焦虑的)
不太可能是解的词汇:主要是指消极类词汇,如negative(作“否定”讲可以是答案;作“消极”讲不作答案);indifferent(漠不关心的);depressed(消沉的); subjective(主观的); pessimistic(悲观的);unconcerned(不关心的);contemptuous(轻慢的);hostile(敌对的);biased(片面的)。
从解题方法来看,一方面要重视具有感情色彩的词汇,更重要的是整体判断优于局部的原则。不要根据某一段来判断,这是考生常犯的错误,也是出题者做手脚的地方。一定要根据文章各段的大意,去归纳分析。
例1
For Lloyd Nickson , a 54-year-old Darwin resident suffering from lung cancer, the NT Rights of Terminally Ill law means he can get on with living without the haunting fear of  his suffering: a terrifying death from his breathing condition.“I’m not afraid of dying from a spiritual point of view, but what I was afraid of was how I’d go, because I’ve watched people die in the hospital fighting for oxygen and cawing at their masks,” he says.
Q: The author’s attitude towards euthanasia seems to be that of
[A] opposition.     suspicion.    [C] approval.  [D]indifference.
解析:主要通过①中感情色彩词“without the haunting fear of his suffering”表现出作者对安乐死的态度是[C]“同意的”。
例2
The world is going through the biggest wave of mergers and acquisitions ever witnessed. The process sweeps from hyperactive America to Europe and reaches the emerging countries with unsurpassed might. Many in these countries are looking at this process and worrying:“won’t the wave of business concentration turn into an uncontrollable anti-competitive force?”
There’s no question that the big are getting bigger and more powerful. Multinational corporations accounted for less than 20% of international trade in 1982. Today the figure is more that 25% and growing rapidly. International affiliates account for a fast-growing segment of production in economies that open up and welcome foreign investment. In Argentina, for instance, after the reforms of the early 1990s, multinationals went from 43% to almost 70% of the industrial production of the 200 largest firms. This phenomenon has created serious concerns over the role of smaller economic firms, of national businessmen and over the ultimate stability of the world economy.
I believe that the most important forces behind the massive M&A wave are the same that underlie the globalization process: falling transportation and communication costs, lower trade and investment barriers and enlarged markets that require enlarged operations capable of meeting customer’s demands. All these are beneficial, not detrimental, to consumers. As productivity grows, the world’s wealth increases.
Examples of benefits of costs of the current concentration wave are scanty. Yet it is hard to imagine that the merger of a few oil firms today could re-create the same threats to competition that were feared nearly a century ago in the U.S., when the Standard Oil trust was broken up. The mergers of telecom companies, such as WorldCom, hardly seem to bring higher prices for consumers or a reduction in the pace of technical progress. On the contrary, the price of communications is coming down fast. In cars, too, concentration is increasing-witness Daimler and Chrysler. Renault and Nissan-but it does not appear that consumers are being hurt.
Yet the fact remains that the merger movement must be watched. A few weeks ago, Alan Greenspan warned against the megamergers in the banking industry. Who is going to supervise, regulate and operate as lender of last resort with the gigantic banks that are being created? Won’t multinationals shift production from one place to another when a nation gets too strict about infringements to fair competition? And should one country take upon itself the role of “defending competition”on issues that affect many other nations, as in the U.S. vs. Microsoft case?(可读性:39  难度:12级)
Q:Toward the new business ware, the writer’s attitude can be said to be
[A] optimistic.       objective.     [C] pessimistic.    [D] biased.
  解析:作者态度题。总结各段大意: <1 >指出公司合并这个现象(只是文章的引言,判断态度为时尚早)。 <2 >这种现象引起了人们对小型经济实体和国内企业的作用,以及对世界经济的最终稳定等问题的强烈关注。(不要根据此段匆忙下结论去选择[C]“悲观的”)。 <3 >促成企业合并的几个因素(不要根据此段怱忙下结论去选择[D]“偏袒的”)。 <4 >作者认为合并有好处(不要根据此段匆忙下结论去选择[A]“乐观的”)。 <5 >作者却说要关注某些合并。(不要根据此段匆忙下结论去选择[C]“悲观的”)。综合各段大意,可见作者的基本态度是“客观地描述和评价”,所以“客观的”正确。[A]“乐观的”,[C]“悲观的”,[D]“偏袒的”均与文意不符。

5.词汇题
词汇题对词语的考查有两种:一是超纲词的含义推断,二是熟词僻义、熟词场合义。词汇题不是考词汇量有多大,而是主要考查是否读懂了上下文和结合上下文推测词义的能力。词汇题之所以归在阅读理解中,目的是考查对文章的理解。因此,做词汇题的方法就是通过认真阅读被考词周围的句子,尤其是该词的前后的句子以及该词出现的句子的意义去推断。
例1
With as many as 120 varieties in existence, discovering how cancer works is not easy. The researchers made treat progress in the early 1970s, when they discovered that oncogenes, which are cancer-causing genes, are inactive in normal cells. Anything from cosmic rays to radiation to diet may activate a dormant oncogene, but how remains unknown. If several oncogenes are driven into action, the cell, unable to turn them off, becomes cancerous。 (可读性:40.5   难度度:12级)
Q: The word “dormant” in the paragraph most probably means
  [A] dead.         ever-present.      [C] inactive.     [D] potential.
  解析:dormant出现在③。①“由于存在120多种癌症,所以找到癌症起因并不容易。”对于猜词帮助不大。②“研究人员发现在正常细胞中,致癌基因并不活跃(inactive)。”③“任何东西都可能激活的致癌基因,但其内在机制仍不为人知。”④“如果几个致癌基因被激活,而细胞又不能将它们消灭,就会成为癌细胞。”据②、③推断“dormant”的意思与“不活跃”相近,故[C] 正确。
例2
We live in a society in which the medicinal and social use of substances (drugs) is pervasive: an aspirin to quiet a headache, some wine to be sociable, coffee to get going in the morning, a cigarette for the nerves. When do these socially acceptable and apparently constructive uses of a substance become misuses? First of all, most substances taken in excess will produce negative effects such as poisoning or intense perceptual distortions. Repeated use of a substance can also lead to physical addiction or substance dependence. Dependence is marked first by an increased tolerance, with more and more of the substance required to produce the desired effect, and then by the appearance of unpleasant withdrawal symptoms when the substance is discontinued. (可读性:42.4  难度:12级)
Q: The word “pervasive” (Line 1) might mean
[A] widespread.                 overwhelming.
[C] piercing.                    [D] fashionable.
解析:pervasive 出现在①。考查考生对下面文字的理解。“:”的使用表示下文是解释、举例、说明的文字。所以该题的线索应为自下而上猜词。后半句,作者列举了“物品”在日常生活各个方面的使用:服用阿斯匹林来缓解头痛,喝点酒为了交际,早晨喝咖啡提神,吸烟以消除紧张。说明前半句,作者说药物在医疗和社交方面的使用很普遍。因此[A]“普遍的”正确。“势不可挡的”;[C]“有穿透力的”;[D]“时髦的”。
例3
Right now, the notion that conventional computers and software are fundamentally incapable of matching the processes that take place in the brain remains controversial. But if it proves true, then the efforts of Conrad and his fellow AI rebels could turn out to be the only game in town. (可读性:54.2  难度:11.7级)
Q: Which of the following is closes in meaning to the phrase “the only game in town”?
[A] The only approach to building an artificially intelligent computer.
The only way for them to win a prize in artificial intelligence research.
[C] The only area worth studying in computer science.
[D] The only game they would like to play in town.
解析:解析:首先排除[D],因为[D]是表面意思,绝对不是解,考研绝对不会出这样简单的题。①:人们认为传统的计算机和软件根本不能和发生在人脑中这些过程相比,这一观念仍然有争议。②:但是:如果这证明是正确的话,那么Conrad等人的努力可能最后证明是“the only game in town”, 即开发人工智能计算机的唯一途径。所以[A]正确。
第二节  潜在命题点的挖掘

这部分试题研究分为两个部分:一是原题的研究,二是潜在命题点的挖掘。原题选自考研实考试题,是历年实考的精华部分,也是考研命题组多年心血的结晶。潜在命题点的挖掘(新试题的命制)耗费了毕老师两个月的宝贵时间。
说起考研实考试题的价值,不要说每套题,就是每道题人都下了很深的功夫。很多同学、考试的时候都感到考研题的选项不是很长,文字上也不是特别难,但干扰性很强,经常会觉得两个,甚至三个选项都对。这一点很像下围棋,很多高手下出的看似平常一步的背后都隐藏着复杂而又激烈的变化。考研每道题、每个选项的设置都是命题人员深入、曲折、周密、准确思维的体现。平均每道题的分析用了笔者3-4个小时的时间。由于分析详细,论述不免繁琐。很多同学都有一种不好的习惯,答对题后就不想再深究了,我奉劝大家研究考试一定要严谨,不可生浮躁之心。要有足够的耐心和细心琢磨透这蕴含着考研命题智慧的十篇文章,不负作者的一片苦心。
第一篇(1995年Passage 3)
一、原题的研究
In such a changing, complex society formerly simple solutions to informational needs become complicated. Mary of life’s problems which were solved by asking family members, friends or colleagues are beyond the capability of the extended family to resolve. Where to turn fox expert information and how to determine which expert advice to accept are questions facing many people today.
In addition to this, there is the growing mobility of people since World War Ⅱ. As families move away from their stable community, their friends of many years, their extended family relationships, the informal flow of information is cut off, and with is the confidence that information will be available when needed and will be trustworthy and reliable. The almost unconscious flow of information about the simplest aspects of living can be cut off. Thus, things once learned subconsciously through the casual communications of the extended family must be consciously learned.
Adding to societal changes today is an enormous stockpile of information. The individual now has more information available than any generation, and the task of finding that one piece of information relevant to his or her specific problem is complicated, time-consuming and sometimes even overwhelming.
Coupled with the growing quantity of information is the development of technologies which enable the storage and delivery of more information with greater speed to more locations that has ever been possible before. Computer technology makes it possible to store vast amounts of date in machine-readable files, and to program computers to locate specific information. Telecommunications developments enable the sending of messages via television, radio, and very shortly, electronic mail to bombard people with multitudes of messages. Satellites have extended the power of communications to report events at the instant of occurrence. Expertise can be shared world wide through teleconferencing, and problems in dispute can be settled without the participants leaving their homes and/or jobs to travel to a distant conference site. Technology has facilitated the sharing of information and the storage and delivery of information, thus making more information available to more people.
In this world of change and complexity, the need for information is of greatest importance. Those people who have accurate, reliable up-to-date information to solve the day-to-day problems, the critical problems of their business, social and family life, will survive and succeed.“Knowledge is power”may well be the truest saying and access to information may be the most critical requirement of all people.(404 words 可读性:25.4   难度:12级)
1.The word“it”(Line 3, para.2) most probably refers to
[A] the lack of stable communities.
the breakdown of informal information channels.
[C] the increased mobility of families.
[D] the growing number of people moving from place to place.
2.The main problem people may encounter today arises from the fact that
[A] they have to learn new things consciously.
they lack the confidence of securing reliable and trustworthy information.
[C]they have difficulty obtaining the needed information readily.
[D]they can hardly carry out casual communications with an extended family.
3.From the passage we can infer that
[A] electronic mail will soon play a dominant role in transmitting messages.
it will become more difficult for people to keep secrets in an information era.
[C] people will spend less time holding meetings or conferences.
[D] events will be reported on the spot mainly through satellites.
4.We can learn from the last paragraph that
[A] it is necessary to obtain as much knowledge as possible.
people should make the best use of the information accessible.
[C]we should realize the importance of accumulating information.
[D] it is of vital importance to acquire needed information efficiently.
二、潜在命题点的挖掘
请参考后面的“三、原题的讲解”彻底弄懂原题后,再将文章读两遍,思考一个什么地方还可出题,出什么样的题。请继续做下面的试题:
5.The author indicates that in former times
[A] the information people needed was basic and plain.
the problems of life could be solved with limited information.
[C] the families were less expanded than they are today.
[D] the ways of getting needed information were relatively simple.
6.Many modern families are confronted with the trouble that
[A]their members enjoy far greater mobility than ever before.
their relatives and friends have left them for good.
[C]they are confused by the outbreak of information explosion.
[D]they wonder whether the expert advice is acceptable.
7.The expanding information stockpile results in the
[A] complication of searching for needed pieces of information.
acceleration of the progress of social reformations.
[C]confusion of approaches to the delivery of any amount of data.
[D] destruction of the regular flow of trustworthy information.
8.To achieve something significant in life, one has to
[A] be coupled with the fast growth of information technology.
resort to the latest true information of his main concern.
[C] keep track of the current developments of telecommunications.
[D]work out effective solutions to the urgent problems of his business.
三、原题的讲解
第一题:
题眼研读:细节题。本题[The word“it”(Line 3, Para.2)most probably refers to]问代词it指代什么。表面上考指代,实质是对 <2 >前两句的理解。如前所述,做细节题的关键是研读句子。
① In addition to this, there is the growing mobility of people since World War Ⅱ.②As families move away from their stable community, their friends of many years,their extended family relationships, the infor. mal flow of information is cut off, and with it the confidence that information will be available when needed and will be trustworthy and reliable.
对②的理解是关键。As引导的状语从句的意思是“随着家庭离开了稳定的社区,离开了多年的朋友们,离开了亲戚”。主句可分为由and连接的两个并列分句:第一分句在and之前,意思是“非正式的信息流通被切断了”;第二分句从and开始到结尾,是难点也是考点。仔细观察发现第二分句只有核心词the confidence及其同位语从句。 同位语从句有两个谓语(will be available和will be trustworthy and reliable);when needed=when it is needed,相当于省略了主谓的状语从句。第二分句现存结构的意思是:“能获得需要的可靠信息的信心。”显然省略了谓语,按照并列句的省略原则谓语在and前面必然出现过,可见被省略的谓语就是is cut off。补全:and with it the confidence that information will be available when needed and will be trustworthy and reliable is cutoff.with it作状语,放在句首使得文章流畅。大意:“随着‘it’,能够获得需要的可靠信息的信心也失去了”。显然,it指的是第一分句中the informal flow of information is cut off(非正式信息流通被切断),故[B]the breakdown of informal information channels.(不正规的信息渠道被切断了)正确。要注意命题者采取了同义词替换和结构转换的方法:breakdown=cut off, channels=flow;原文是句子,答案用名词词组。
干扰项分析:[A]the lack of stable communities(缺乏稳定的社区)取材于并篡改了②中move away from their stable community(家庭离开了稳定的社区);[C]the increased mobility of families.(家庭流动性的增加)取材于①中growing mobility of people,两选项均非it 所指。[D]the growing number of people moving from place to place(四处移动的人的数量越来越多)是[c]的翻版,只不过把families换成people,growing换成growing number, mobility换成moving from place to place。
体会:做题的时候文章中概念的逻辑关系一定要搞清楚,不能根据似曾相识的个别词汇选择答案,选项完全照抄原文的一般不是解。但正确选项中也可出现文章中词汇,如本题中information,对于命题者而言这也是无可奈何之事。
九阴真经解注:指代常考,中心思想是解。
第二题:[C]
题眼研读:变形的主旨题。本题(The main problem people may encounter today arises from the fact that)问人们现今碰到的主要问题源于一个什么事实。该题从字面上看包括两个方面:1)什么是所谓的人们遇到的“主要问题”;2)问题产生的事实(客观原因)是什么。但看看四个选项,其实四个选项回答的都是第一个问题,无异于The main problem people may encounter today is that,可简化为The main problem is that。此题看似细节题,实际上考查的是全文的topic,问题之所以成为主题是因为人们因存在问题才写文章,考研相当一部分文章是问题解决型的。
文章的主题实际上是各段主题的综合。了解topic的方法是阅读 <1 >首句和末句,并兼顾每段的主题句。 <1 >:由于社会变得复杂了,所以得到信息难。 <2 >:人口的高流动性加剧了信息获得难的程度。 <3 >:信息量的剧增使获得与自己有关的信息更加复杂了,更费时间了( <3 >②后半句:and the task of finding that one piece of information relevant to his or her specific problem is complicated,time-consuming and sometimes even overwhelming.)。这三段加在一起或者说作为文章主题句的 <3 >②说明“人们不能容易地获得所需信息”是人们当今遇到的主要问题。所以[C]they have difficulty obtaining the nedded information readily.既是 <3 >②后半句的改写,又是全文主题的概括。 <4 >:技术的发展对于信息的好处。实际上是说解决信息问题的出路。末段重申主题,笼统地说获得信息的重要性。
干扰项分析:对于主题题,以“偏概全”是干扰项的重要特征。[A]They have to learn new things consciously.(他们必须有意识地学习新东西)和They lack the confidence of securing reliable and trustworthy information.(他们缺乏获得可靠信息的信心)在 <2 >中提到,但属细节或次要问题,以偏概全。[D]They can hardly carry out casual communications with an extended family.(人们不能和大家庭成员进行非正式的交流)有两个问题:一是hardly,这个词是出题者强加上去的。根据 <2 >末句。原文强调信息交流的减少,而[D]说不能交流,过于绝对。此外即使没有hardly这个词也不对,[D]无论如何也只是文中细节。另外请大家注意观察三个干扰项与 <2 >文字的雷同之处。

  In addition to this, there is the growing mobility of people since World War Ⅱ. As families move away from their stable community, their friends of many years, their extended family relationships, the informal flow of information is cut off, and with it the confidence that information will be available when needed and will be trustworthy and reliable. The almost unconscious flow of information about the simplest aspects of living can be cut off. Thus, things once learned subconsciously through the casual communications of the extended family must be consciously learned.
体会:对于主旨题,干扰项通常是片面的、有关细节的、照抄原文的。
九阴真经解注:段中心常考,合理项不是解,绝对的不是解,含义深刻是解,中心思想是解,difficulty是解。

第三题:[A]
题眼研读:推理题。本题(From the passage we can infer that)问从文中可以推导出什么结论。推理题如前所述可能是针对一句话,也可能是一段,也可能是整篇文章。从题干看不出考点,只能求助于四个选项。通过提炼选项的关键词,如 electronic mail, satellites等,很容易判断考点在 <4 >。
①Coupled with the growing quantity of information is the development of technologies which enable the storage and delivery of more information with greater speed to more locations than has ever been possible before. ②Computer technology makes it possible to store vast amounts of date in machine-readable files, and to program computers to locate specific information. ③Telecommunications developments enable the sending of messages via television, radio, and very shortly, electronic mail to bombard people with multitudes of messages. ④Satellites have extended the power of communications to report events at the instant of occurrence. ⑤Expertise can be shared world wide through teleconferencing, and problems in dispute can be settled without the participants leaving their homes and/or jobs to travel to a distant conference site. ⑥Technology has facilitated the sharing of information and the storage and delivery of information, thus making more information available to more people.
③:电信技术的发展使人们可以通过电视、无线电发送消息,电子邮件很快就会给人们传送大量的信息。[A]Electronic mail will soon play a dominant role in transmitting messages.(电子邮件不久可以在信息传递中起主导作用)是从该句演变而来的,但说电子邮件起主导作用,有点过分,尽管文中bombard和multitudes of二词有一定的力度。实事求是地讲,此题决定选[A]应该是在排除其他选项之后。
干扰分析:It will become more difficult for people to keep secrets in an information era.(在信息时代人们很难保守秘密); <4 >说信息传递速度快,但推进太过。[C]people will spend less time holding meetings or conferences.(人们开会所用的时间将来越越少);源于⑤后半句,“不必离家或去远的会场就可以解决很多问题”,由此不能推断开会时间的减少。[D]Events will be reported on the spot mainly through satellites.(事件的现场报道将主要通过卫星):④“人造卫星扩展了通信能力,可以对事件进行现场报道。”没有体现出事件的现场报道将主要通过卫星。如果将[D]改成Events can be reported on the spot through satellites.则正确。
九阴真经解注:重要是解。
第四题:[D]
题眼研读:段落大意判断题。本题问“we can learn from the last paragraph that”,实际上问的是末段的中心大意。
In this world of change and complexity, the need for information is of greatest importance. Those people who have accurate, reliable up-to-date information to solve the day-to-day problems, the critical problems of their business, social and family life, will survive and succeed. “Knowledge is power” may well be the truest saying and access to information may be the most critical requirement of all people.
末段的中心大意是获取知识的重要性。在第二题的讨论中我们已经说过更准确、贴切的文章中心是:获得所需知识的重要性。此二者的范围上不同,在这种情况下,要有整体观念,局部要服从整体,结尾段的中心要符合文章的中心,即纵观全文后应深刻地将末段大意理解成“获取所需知识的重要性。”这样只有[D] It is of vital importance to acquire needed information efficiently. 正确。从感觉上说,[D]精雕细刻、字斟句酌,像答案。干扰项分析:[A] It is necessary to obtain as much knowledge as possible. (有必要获取尽可能多的知识):直接取材于“Knowledge is power”, 并且偷换了概念,用knowledge代替information。即使认为knowledge可以和information替换,[A]也仅仅是对末段中心的肤浅理解。[C]We should realize the importance of accumulating information. (我们应该认识到积累信息的重要性)和[A]是一个问题。 People should make the best use of the information accessible. (人们应该充分利用可以获取的知识)偏离主题。
体会:如何精确地、恰如其分地理解中心思想是一个关键的问题。
九阴真经解注:段中心常考,最高级常考,重要是解,中心思想是解。
四、新题的讲解
第五题:[D]
题眼研读:推理题。题干(The author indicates that in former times)中indicate的意思是“暗示着”,表明该题文章没有明说,考生末根据文章的内容去分析、推理。其中“in former times ”很关键,给我们提供了线索,从文章 <1 >①中找到与之相响应的formerly:

①In such a changing, complex society formerly simple solutions to informational needs become complicated. ②May of life’s problems which were solved by asking family members, friends or colleagues are beyond the capability of the extended family to resolve.

①:在这样一个不断变化的复杂社会中,从前对信息需求的简单解决方法变得复杂了。need是名词,很多同学误把needs当成谓语动词,从而影响对句子的理解。而且容易忽略副词formerly,或误将formerly看成formally,从而抓不住阅读重点。[D] The ways getting needed information were relatively simple.(过去获取需要信息的方式相对简单)正确。此题是考研逆向思维的体现,大家往往会把阅读重点放在句子的主干——主、谓、宾上面,而忽视了主语比较“隐蔽”的定语simple,这就是我总结的“隐蔽处常考”的典型例子。同时此题又蕴含着“反着考”的味道。
干扰项分析:[A] The information people needed was basic and plain.(过去人们需要的信息简单):偷换概念,将ways of obtaining information改成了information。 The problems of life could be solved with limited information.(生活的问题可以通过有限的信息解决):the problems of life could be solved源于②,大意是“现在人们解决生活问题靠问家庭成员已经不可能了”,与相差甚远。[C] The families were less expanded than they are today.(过去的家庭没有现在的大):与文意正好相反。expand与extend形似。文中extended family是合成词,与nuclear family(小家庭)相对,意思是“大家庭”,家中可能包括父母、祖父母、叔、婶等,老一辈四世同堂的中国人对大家的感受都很深。
第六题:[C]
题眼研读:题干(Many modern families are confronted with the trouble that)意思是“现代很多家庭面临的一个困难是什么?”此题是第二题的翻版,实际上问的是主题。[C] They are confused by the outbreak of information explosion.(人们在信息爆炸面前感到困惑)符合要求。
干扰项分析:[A] Their members enjoy far greater mobility than ever before.(它们的成员拥有比过去更大的移动性): <2 >①②大意是人口流动性的增加(原因)加剧了现代家庭的一个问题,即获取信息难(结果)。本题问的是果,不是因。 Their relatives and friends have left them for good:注意小动作,错在for good(永远), <2 > ②说离开,但没说永远离开。[D]They wonder whether the expert advice is acceptable.(他们怀疑是否接受专家建议)来源于 <1 >末句“去哪儿寻找专家信息,如何决定接受哪个专家的建议是我们遇到的问题。”更改了原文的意思。
体会:注意小动作,注意细微含义的分辨。
第七题:[A]
题眼研读:细节题。题干“The expanding information stockpile results in the”(不断增加的信息导致了什么结果?)stockpile(积蓄、储备)是关键词,该词位于原文 <3 >①:
①Adding to societal changes today is an enormous stockpile of information. ② The individual now has more information available than any generation, and the task of finding that one piece of information relevant to his or her specific problem is complicated, time-consuming and sometimes even overwhelming.
①和②之间存在着内在的逻辑关系,①和②前半句是②后半句的原因,题干里的result in 与之呼应。解必然是②的改写,实际上[A]complication of searching for needed pieces of information.(寻找所需信息的复杂化)正是②后半句的改写,与第二题异曲同工,问来问去都是在问主题。解中complication是complicated的名词,search for是find的同义词,needed information是information relevant to his/her specific problem的概括。干扰项分析:acceleration of the progress of social reformations.(社会变革的加速):是凭空想象的合理项,文中没提到。[C]confusion of approaches to the delivery of any amount of date.(对发送任何数量的数据方法的困惑):使用 <4 >①②中只言片语(delivery,date,amount)拼凑而成,而且any这个词过于绝对,不是解。[D]destruction of the regular flow of trustworthy information.I(可信赖的信息流通的终止):是 <2 >中人口流动性增加所导致的结果,不是 <3 >中信息量增加的结果。
体会:张冠李戴是干扰项常用的欺骗手段;绝对地不是解,中心思想是解。
第八题:
题眼研读:细节题。题干(To achieve something significant in life, one has to)和末段②相似。To achieve something significant in life=survive and succeed,由此判断考点在末段②。
Those people who have accurate, reliably up-to-date information to solve the day-to-day problems, the critical problems of their business, social and family life, will survive and succed.
大意:那些利用准确、可靠和最新的信息解决日常问题、企业关键问题及社会与家庭问题的人就能生存下去并获得成功。正确选项 resort to the latest true information of his main concern.(人们必须使用主要关心的事情的最新的真实信息)是原文的改写。the latest=up-to-date, true=reliable。
干扰分析:[A] be coupled with the fast growth of information technology.(伴随着信息技术的快速发展):采自 <4 >①,放在这里逻辑不通,人不能伴随发展而发展。本题实际上考查考生对coupled with的理解。[C] keep track of the current developments of telecommunications.(紧跟电信的发展):取材于 <4 >③,过于具体,不是解。考词组keep track of(关注……的发展)。[D]work out effective solutions to the urgent problems of his business.(找出公司迫切问题的有效解决办法):取材于末段②,本题问手段,[D]是结果。这几个干扰项大量使用文中原词或同义词,如work out=solve,urgent=critical, problems of business.
注释:1)extended family大家庭;2)mobility流动性;3)subconsciously下意识地;4)casual communications非正式的交流;5)stockpile储存,积聚;6)overwhelming不知所措;7)telecommunication电信;8)teleconference电视会议;9)facilitate使……变得容易

第二篇(1995年Passage 5)
That experiences influence subsequent behaviour is evidence of an obvious but nevertheless remarkable activity called remembering. Learning could not occur without the function popularly named memory. Constant practice has such an effect on memory as to lead to skilful performance on the piano, to recitation of a poem, and even to reading and understanding these words. So-called intelligent behavior demands memory, remembering being a primary requirement for reasoning. The ability to solve any problem or even to recognize that a problem exists depends on memory. Typically, the decision to cross a street is based on remembering many earlier experiences.
Practice(or review) tends to build and maintain memory for a task or for any leaned material. Over a period of no practice what has been learned tends to be forgotten; and the adaptive consequences may not seem obvious. Yet, dramatic instances of sudden forgetting can be seen to be adaptive. In this sense, the ability to forget can be interpreted to have survived through a process of natural selection in animals. Indeed , when one’s memory of an emotionally painful experience lead to serious anxiety, forgetting may produce relief. Nevertheless, an evolutionary interpretation might make it difficult to understand how the commonly gradual process of forgetting survived natural selection.
In thinking about the evolution of memory together with all its possible aspects, it is helpful to consider what would happen if memories failed to fade. Forgetting clearly aids orientation in time, since old memories weaken and the new tend to stand out, providing clues for inferring duration. Without forgetting, adaptive ability would suffer, for example, learned behaviour that might have been correct a decade ago may no longer be. Cases are recorded of people who(by ordinary standards)forgot so little that their everyday activities were full of confusion. This forgetting seems to serve that survival of the individual and the species.
Another line of thought assumes a memory storage system of limited capacity that provides adaptive flexibility specifically through forgetting. In this view, continual adjustments are made between or memory storage(input) and forgetting(output). Indeed, there is evidence that the rate at which individuals forget is directly related to how much they have learned. Such data offers gross support of contemporary models of memory that assume an in-put-output balance.(381 words 可读性:32.6  难度:12级)
一、原题的研究
1.From the evolutionary point of view
[A] forgetting for lack of practice tends to be obviously inadaptive.
if a person gets very forgetful all of a sudden he must be very adaptive.
[C] the gradual process of forgetting is an indication of an individual’s adaptability.
[D] sudden forgetting may bring about adaptive consequences.
2.According to the passage, if a person never forgot
[A] he would survive best.
he would have a lot of trouble.
[C] his ability to learn would be enhanced.
[D] the evolution of memory would stop.
3.From the last paragraph we know that
[A] forgetfulness is a response to learning.
the memory storage system is an exactly balanced input-output system.
[C] memory is a compensation for forgetting.
[D] the capacity of memory storage system is limited because forgetting occurs.
4.In this article, the author tries to interpret the function of
[A]remembering.   forgetting.    [C]adapting.     [D]experiencing.
二、潜在命题点的挖掘
请参考后面的“三、原题的讲解”彻底弄懂原题后,再将文章读两遍,思考一下什么地方还可以出题,出什么样的题。请继续做下面的试题:
5.The author believes that learning capacity
[A] gives clear evidence of remarkable process of experiences.
affects one’s subsequent achievements in life.
[C] roots in the knowledge and skill gained in practice.
[D] results from the influence of intelligent behavior.
6.It would be safe to say that memory
[A] appears to be the unique constituent of learning and reasoning.
is indispensable to the capability to tackle practical problems.
[C] proves to be the wonderful instinct of the human heart.
[D] is established and improved by persistent learning.
7.The preservation of past experience in the conscious mind
[A] is retained through frequent review.
is threatened by unexpected forgetting.
[C] relies on the inference of earlier practice.
[D] consists in the adaptive consequences.
8.The author argues that
[A] natural selection can be avoided through forgetting.
memory fading helps determine its existing duration.
[C] the adaptability of memory is liable to constant adjustments.
[D] forgetting ability contributes to the survival of the fitttes.
三、原题的讲解
第一题:[D]
题眼研读:细节题。题干From the evolutionary point of view(从进化论的观点来看)中关键词为evolutionary,该词位于 <2 >末句:
①Practice(or review) tends to build and maintain memory for a task or for any learned material.②Over a period of no practice what ahs been learned tends to be forgotten; and the adaptive consequences may not seem obvious.③Yet, dramatic instances of sudden forgetting can be seen to be adaptive.④In this sense, the ability to forget can be interpreted to have survived through a process of natural selection in animals. ⑤Indeed, when one’s memory of an emotionally painful experience lead to serious anxiety, forgetting may produce relief. ⑥ Nevertheless, an evolutionary interpretation might make it difficult to understand how the commonly gradual process of forgetting survived natural selection.

此段代表进化论的观点,结构非常复杂,一波三折。如果你能把这三折弄清楚,你就能答对。①和②是“第一折”,意思是实践(或复习)建立并保持对某一任务或所学材料的记忆。有一段时间不实践,学到的东西往往就会忘记(渐进性的遗忘);其适应性结果也就显得不太明显。③④是“第二折”,大意是说突然遗忘可以看作是适应性的。⑤为“第三折”,重申第一折的观点,渐进性的遗忘不是适应性的。所以[D]sudden forgetting may bring about adaptive consequences.(突然遗忘可能产生适应性的结果)正确。注意:原文和解之间的结构转换,原文can be seen to be adaptive被动,[D]为主动语态。may含义相对,很容易作解。
干扰项分析:[A]forgetting for lack of practice tends to be obviously inadaptive.(由于缺乏练习所致的遗忘趋于是明显非适应性的):错在obviously这个词上,一定要注意这种小动作,如果去掉obviously则正确。②说缺乏练习导致遗忘,其适应性不明显。if a person gets very forgetful all of a sudden he must be very adaptive.(如果一个人突然变得非常健忘,他一定非常具有适应性):本来意思是对的,但语气太强烈了,绝对地不是解,如果这个选项去去掉一个must和两个very就对的。[C] the gradual process of forgetting is an indication of an individual’s adaptability.(逐渐的遗忘过程是一个人适应力的表现):和文意正好相反。
九阴真经解注:转折常考,绝对不是解,含义相对是解。
第二题:

题眼研读:推理题。从题干(According to the passage, if a person never forgot)找线索,在  <3 >①有if memories failed to fade(假设记忆不衰退),两者意思相同,并且都是虚拟语气。
①In thinking about the evolution of memory together with all its possible aspects, it is helpful to consider what would happen if memories failed to fade.②Forgetting clearly aids orientation in time, since old memories weaken and the new tend to stand out, providing clues for inferring duration.③Without forgetting, adaptive ability would suffer, for example, learned behaviour hat might have been correct a decade ago may nolonger be.④Cases are recorded of people who(by ordinary standards)forgot so little that their everyday activities were full of confusion. ⑤This forgetting seems to serve that survival of the individual and the species.
④“有记录表明,几乎没有遗忘的那些人,其日常活动充满了混乱”=he would have a lot of trouble。(他会有许多麻烦。)
干扰项分析:[A]he would survive best.(他将活得最好)和[C]his ability to learn would be enhanced.(他的学习能力将会提高)文中未提到,是符合常识的合理项。[D]the evolution of memory would stop:取材于①,和文意相差很远。
体会:合理项不是解。
九阴真经解注:反着考,取非求解,命题模式:A至B,所以非A则非B,这种不严格的逻辑在考试中常用
第三题:[A]
题眼研读:大意推理题。题干(From the last paragraph we know that)明示考末段。
Another line of thought assumes a memory storage system of limited capacity that provides adaptive flexibility specifically through forgetting. In this view, continual adjustments are made between learning or memory storage (input) and forgetting (output). Indeed, there is evidence that the rate at which individuals forget is directly related to how much they have learned. Such data offers gross support of contemporary models of memory that assume an input-output balance.
该段说的是另外一个关于记忆和遗忘的观点。根据这一观点,人的记忆空间是有限的,遗忘为学习记忆提供了适应性和灵活性。更明白一点讲,一个人的记忆储存空间有限,所以只能通过忘记一些头脑中的信息的方式为记忆新知识提供空间。[A]forgetfulness is a response to learning.(遗忘是对学习的一种反应)符合这一观点,为这一段的中心。
干扰项分析: the memory storage system is an exactly balanced input-output system.(记忆储存系统是一个完全平衡的进出系统):错就错在exactly这个词上,过于绝对,不是解。[C]memory is compensation for forgetting.(记忆是对遗忘的补偿):是符合生活常识的合理项,但和文意正好相反,如果改成 forgetting is a compensation for memory就对了。[D] the capacity of a memory storage system is limited because forgetting occurs.(因为遗忘的发生所以记忆储存系统的容量是有限的);也是命题者编造的符合常识的合理项。而且大量使用了①中词语如:capacity, memory storage system, forgetting, limited.而且还远距离地用了 <1 >②中occur。
九阴真经解注:段中心常考,绝对不是解,合理项不是解,含义深刻是解,相互作用的是解。
第四题:
题眼研读:主旨题。从题干(In this article, the author tries to interpret the function of )可以看出考的是本文的主题词。前文讲过,这类题的解题思路有两个,一是看各段大意和内容,二是找文章中出现频率高的词。本文共4段,从 <2 >开始讨论forgetting,一直讨论到 <4 >。另外forgetting在文中反复出现。据此判断forgetting对。
干扰项分析:[A]remembering(记忆):只是出现在 <1 >,片面。[C] adaptive在文中出现不少,但也只是一个细节。[D]experiencing(经历):更具体。
体会:对付主旨题要防止“具体和片面”。
九阴真经解注:中心思想是解,反复出现的词,往往是中心思想。
四、新题的讲解
第五题:[C]
题眼研读:细节题。本题(The author believes that learning capacity…)极难!从关键词learning判断,考点位置在 <1 >。
①That experiences influence subsequent behaviour is evidence of an obvious but nevertheless remarkable activity called remembering. ②Learning could not occur without the function popularly named memory.③Constant practice has such an effect on memory as to lead to skilful performance on the piano, to recitation of a poem, and even to reading and understanding these words. ④So-called intelligent behavior demands memory, remembering being a primary requirement for reasoning. ⑤The ability to solve any problem or even to recognize that a problem exists depends on memory. ⑥Typically, the decision to cross a street is bases on remembering many earlier experiences.
大意:①经历会影响日后的行为, 这就表明存在着一种明显但却非凡的脑力活动——书记。②如果没有大家称之为记忆的功能,学习便不可能发生。③反复练习对记忆有很大影响,可以使人们熟练地演奏钢琴、背诵诗歌乃至阅读和理解这些词句。简化:①“以经历为例引出记忆。” ②“学习源于记忆。” ③“反复实践影响记忆,可以使人能够弹钢琴、诵诗等。”推理:弹琴能力是从实践中获得的知识和技能。进一步推理:学习能力源于实践中获得的知识和技能=[C] roots in the knowledge and skill gained in practice。
干扰项分析:[A] give clear evidence of the remarkable process of experiences.(为非凡经历过程提供明显证据):取材于①。照抄文中的词汇有:evidence, remarkable, experiences;与原文类似的词汇有clear(obvious), process(activity)。①大意为“经历会影响日后的行为,这就表明存在着一种明显的但却非凡的脑力活动——记忆。”[A]断章取义。affects one’s subsequent achievements in life.(影响人一生中后来的成就):是编造的符合生活常识的合理项,文中没提到。[D] results from the influence of intelligent behavior.(起源于智力行为的影响):断章取义,逻辑荒谬,但有一定的诱惑性,因为好几个词都出自文章中中:intelligent behavior出自④,results from 从④中demands同义替换而来。
第六题:
题眼研读:推理题。题干(It would be safe to say that memory…)中关键词是memory。 <1 >⑤The ability to solve any problem or even to recognize that a problem exists depends on memory.(解决一个问题,甚至认识到一个问题的存在都需要记忆)所以 is indispensable to the capability to tackle practical problems.(记忆对于解决实际问题的能力是不可或缺的)是原文的改写,正确。
干扰项分析:[A] appears to be the unique constituent of learning and reasoning.(似乎是学习和推理独一无二的要素: <1 >④说记忆是推理和智力活动的基础,而本选项则显得言过其实了,尽管从形式上看语气并不太绝对。[C] proves to be the wonderful instinct of the human heart.(证明是人的心脏美妙的本能):凭空编造的合理项,太不符合常识了。[D] is established and improved by persistent learning.(是通过持续学习建立和提高的):源于 <2 >①“实践(或复习)建立并保持对某一任务或所学材料的记忆。”[D]错在persistent learning上,因为建立某种记忆不见得需要持续学习。
体会:符合常识的选项一般叫合理项,合理项是经常作为干扰项而出现的,不是正确答案。但明显不符合基本常识甚至于达到荒谬程度的选项也不可能是解,正所谓“物极必反”。

第七题:[A]
题眼研读:细节题。题干(The preservation of past experience in the conscious mind)关键词是preservation of past experience,实际上指的是memory的保持。在 <1 >和 <2 >都提到不断的练习可使人保持某种记忆。[A] is retained through frequent review.(通过不断复习保持),符合文意。
干扰项分析: is threatened by unexpected forgetting.(受到难以预料的遗忘的威胁):很多同学都误选。原文 <2 >关于遗忘的观点是:突然性的遗忘应被认为是人适应性的表现,根本谈不到什么“威胁”,遗忘是好事,关于这个问题我专门请教了毕老师,毕老师说:“threaten这个词放在这里比较含糊。”其实考研许多题是很难用语言讲清楚的,我给大家讲的很多都是一种感觉。[C] relies on the inference of earlier practice.(依赖于先前实践的推导):记忆的保持依赖于反复的练习和实践,用不着去推理。[D] consists in the adaptive consequences(在于其适应性结果):生硬地使用了 <2 >②中the adaptive consequences,造成该句逻辑不通。
体会:所谓题感就是对题目的感觉,题感是从研究历年实考试题中得来的。

第八题:[D]
题眼研读:态度题。本题(The author argues that)考作者的观点。一篇文章中甚至在一段中作者的观点可能有好几个,导致对此类题的界定比较难,必须通篇逐个寻找,一边验证正确的观点,一边排除错误的,未提到的或非作者的观点。考生需有足够的耐心!本题主要考 <3 >末句(This forgetting seems to serve that survival of the individual and the species.)大意:这种遗忘似乎有助于个体及物种的存活。[D]forgetting ability contributes to the survival of the fittest.(遗忘能力有助于适者生存)是这句话的改写。
干扰项分析:[A] natural selection can be avoided through forgetting.(通过遗忘,自然选择可以避免):来自于 <2 >末句“然而进化论很难解释渐进遗忘过程是在自然选择中幸存下来的。”言外之意是渐进性的遗忘不是适应性的。[A]与文意不符。照抄原文的词汇有:natural selection, forgetting; avoided间接来自difficult to understand. memory fading helps determine its existing duration.(记忆消退有助于判断记忆存在的时间):该句的欺骗性很强,取材于来自 <3 >①②。借词很巧妙也很隐蔽:memory fading,来自①的memories failed to fade; helps和②的aids同义;determine和providing clues for 类似;existing duration对应inferring duration。 <3 >②说:遗忘有助于时间的定位,(不是遗忘的定位),因为旧的记忆淡忘,新的记忆往往就突出,从而为推算持续的时间提供线索。在duration前加上定语its和existing,造成了假信息。[C]the adaptability of memory is liable to constant adjustments.(记忆的适应能力有不断调整的倾向):末段②“根据这个观点,在学习或记忆储存(输入)和遗忘(输出)之间要不断进行调整。”不是指记忆适应性的调整,故不对。
注释:1)recitation of a poem朗诵诗歌;2)typically通常;3)adaptive consequence适应性结果;4)evolutionary interpretation进化论的解释;5)fade消失,退色;6)orientation in time时间定位;7)line of thought 思路
第三篇(1996年Passage 3)
一、原题的研究
In the last half of the nineteenth century “capital”and “labour”were enlarging and perfecting their rival organizations on modern lines. Many an old firm was replaced by a limited liability company with a bureaucracy of salaried managers. The change met the technical requirements of the new age by engaging a large professional element and prevented the decline in efficiency that so commonly spoiled the fortunes of family firms in the second and third generation after the energetic founders. It was moreover a step away from individual initiative, towards collectivism and municipal and state-owned business. The railway companies, though still private business managed for the benefit of shareholders, were very unlike old family business. At the same time the great municipalities went into business to supply lighting, trams and other services to the taxpayers.
The growth of the limited liability company and municipal business had important consequences. Such large, impersonal manipulation of capital and industry greatly increased the numbers and importance of shareholders as a class, an element in national life representing irresponsible wealth detached from the land and the duties of the landowners; and almost equally detached from the responsible management of business. All through the nineteenth century, America, India Australia and parts of  Europe were being developed by British capital, and British shareholders were thus enriched by the world’s movement towards industrialization. Towns like Bournemouth and East-bourne sprang up to house large“comfortable”classes who had retired on their incomes, and who had to relation to the rest of the community except that of drawing dividends and occasionally attending a shareholders meeting to dictate their orders to the management. On the other hand“shareholding”meant leisure and freedom which was used by many of the later Victorians for the highest purpose of a great civilization.
The “shareholders”as such had no knowledge of the lives, thoughts or needs of the workmen employed by the company in which he held shares, and his influence on the relations of capital and labour was not geed. The paid manager acting for the company was in more direct relation with the men and their demands, but even he had seldom that familiar personal knowledge of the workmen which the employer had often had under the more patriarchal system of the old family business now passing away. Indeed the mere size of operations and the numbers of workmen involved rendered such personal relations impossible. Fortunately, however, the increasing power and organization of the trade unions, at least in all skilled trades, enabled the workmen to meet on equal terms the managers of the companies who employed them. The cruel discipline of the strike and lockout taught the two parties to respect each other’s strength and understand the value of fair negotiation.(460 words 可读性:32  难度:12级)
1.It’s true of the old family firms that
[A] they were spoiled by the younger generations.
they failed for lack of individual initiative.
[C] they lacked efficiency compared with modern companies.
[D] they could supply adequate services to the taxpayers.
2.The growth of limited liability companies resulted in
[A] the separation of capital from management.
the ownership of capital by managers.
[C] the emergence of capital and labour as two classes.
[D] the participation of shareholders in municipal business.
3.According to the passage, all of the following are true except that
[A] the shareholders were unaware of the needs of the workers.
the old firm owners had a better understanding of their workers.
[C] the limited liability companies were too large to run smoothly.
[D] the trade unions seemed to play a positive role.
4.The author is most critical of
[A] family firm owners.   landowners.
[C] managers.           [D] shareholders.
二、潜在命题点的挖掘
请参考后面的“三、原题的讲解”彻底弄懂原题后,再将文章读两遍,思考一个什么地方还可出题,出什么样的题。请继续做下面的试题:
5.The phrase“on modern lines”in the opening sentence of the text may be best interpreted as
[A] going along with social changes.
following up-to-date methods.
[C] keeping to advanced courses.
[D] moving by the newest routes.
6.The root cause of the old furns’ collapse lies in their
[A] inadaptability to satisfy the demands for high efficiency.
failure to engage capable managers to run their business.
[C] inability to foresee the rapid development of technology.
[D] deficiency of highly qualified managing professionals.
7.The author points out that
[A] limited liability companies brought modernizations to many countries.
great contributions were made by shareholders to British economy.
[C] British shareholders profited most from the tide of modem industrialization.
[D] the capital and management of corporations were impaired by shareholders.
8. In the 19th century British shareholding classes
[A] enjoyed their remaining years comfortably on their share profits.
enriched their colonies in five continents through investments.
[C] broke off all their connections with the rest of the community.
[D] bore responsibility for the detachment of their riches from company manipulation.

三、原题的讲解
第一题:[C]
题眼研读:推理题。本题(It’s true of the old family firms that)问对于老的家族式商号下面哪个论述是对的。题干中关键词是old family firms 。根据第一题通常考 <1 >的特点,在该段寻找关键词,发现②③都出现family firm 或old firm,确定这两句为重点。

①In the last half of the nineteenth century “capital”and “labour”were enlarging and perfecting their rival organizations on modern lines. ②Many an old firm was replaced by a limited liability company with a bureaucracy of salaried managers. ③The change met the technical requirements of the new age by engaging a large professional element and prevented the decline in efficiency that so commonly spoiled the fortunes of family firms in the second and third generation after the energetic founders. ④It was moreover a step away from individual initiative, towards collectivism and municipal and state-owned business. ⑤The railway companies, though still private business managed for the benefit of shareholders ,were very unlike old family business.⑥At the same time the great municipalities went into business to supply lighting, trams and other services to the taxpayers.

②简单,意思是“很多老商号被有限责任公司所取代,这种公司具有由领薪经理所组成的官  机构。” ③主语是change,谓语是由and连接的met和prevented,decline是prevented的宾语,that引导的定语从句修饰decline,大意是“这种变化通过雇 大量专业人员来满足新时代的技术要求,并阻止了效率下滑。由于效率的低下,家族式商号通常在那些精力充沛的创始人之后的第二、三代就倒闭了。”由此判断家族商号比现代公司缺乏效率。所以[C] they lacked efficiency compared with modern companies.(他们比现在的公司缺乏效率)正确。本题的考法是“反着考”,防止了效益的下滑,反过来说就是缺乏效率。而且考的地方是定语从句,比较隐蔽,所以可以总结成“隐蔽处常考”。
干扰项分析:[A] they were spoiled by the younger generations .(它们被晚辈们毁掉了):干扰性极强。为什么[A]不对?因为[A]可使人理解为晚辈们有意地毁掉那些企业,而③用的是in the second and third generation,不是by the second and third generation,实际情况是晚辈由于缺乏科学高效的管理而使企业衰落。另外要注意③中“so commonly”这个词说明并不是所有的晚辈都无能,[A]比较绝对,使人感到所有的企业都是被晚辈毁掉的。they failed for lack of individual initiative.(他们因缺乏个人积极性而失败);与③的语意正好相反,老企业不缺乏进取心。[D]they could supply adequate services to the taxpayers.(它们能对纳税人提供充足的服务);与文意完全不符,是对⑥断章取义的结果。
体会:如果从正面不得其解,就反着找。当你认识到考研常常是“反着考”时,你答题的境界就提高了。
九阴真经解注:隐蔽处(定语从句)有解,对比是解,复杂是解。

第二题:[A]
题眼研读:细节题。该题(The growth of limited liability companies resulted in)问有限责任公司的发展导致什么后果。从关键词the growth of limited liability companies出发,在 <2 >①找到线索。该句就是这一段的主题句。①中“had important consequences”相当于题干中resulted in,接下来谈论的是结果。②又长又难,共47words,考点在此。
①The growth of the limited liability company and municipal business had important consequences. ②Such large, impersonal manipulation of capital and industry greatly increased the numbers and importance of shareholders as a class, an element in national life representing irresponsible wealth detached from the land and the duties of the landowners; and almost equally detached from the responsible management of business. ③All through the nineteenth century, America, Africa, India, Australia and parts of Europe were being developed by British capital ,and British shareholders were thus enriched by the world’s movement towards industrialization. ④ Towns like Bournemouth and Eastbourne sprang up to house large“comfortable”classes who had retired on their incomes, and who had no relation to the rest of the community except that of drawing dividends and occasionally attending a shareholders meeting to dictate their orders to the management.⑤On the other hand “shareholding”meant leisure and freedom which was used by many of the later Victorians for the highest purpose of a great civilization.
②分析:主语是Such large, impersonal manipulation of capital and industry,谓语是increased,宾语是the numbers and importance of shareholders, as a class介词短语作定语修饰shareholders, an element in national life是shareholders的同位语,representing irresponsible wealth作定语修饰clement, detached from the land and the duties of the landowners; and almost equally detached from the responsible management of business是wealth的定语。大意:这样巨大的非个人的对资金和企业的运作增加了作为一个社会阶层的股东的重要性和人数;股东阶层的出现表现财富与土地、土地所有者应尽义务相分离,与管理相分离。这与[A] the separation of capital from management.(资本与管理的分离)吻合。注意capital 与wealth的替换。本题从语法来看,考的是难句中宾语的同位语的定语的定语,充分反映了考研的特色。
干扰项分析: the ownership of capital by managers.(管理人员对资本的拥有权):②说明股东拥有资本,管理人员只负责管理,与文意相反。[C]the emergence of capital and labour as two classes(出现了资方和劳方两个阶层): <1 >①“19世纪下半期,”‘资方’和‘劳方’按现代化方式不断扩大和完善各自相对立的组织。②“很多老商号被有限责任公司取代”。①和②之间是因果关系,劳资双方的扩大导致了有限责任公司的产生,和[C]的意思不同。[D]the participation of shareholders in municipal business.(股东参与市政企业):断章取义于 <1 >末句,文中说股东参与市政企业。
九阴真经解注:隐蔽处常考,矛盾处常考,矛盾是解,含义深刻是解,肤浅不是解
第三题:[C]
题眼研读:Except题。该题(According to the passage, all of the following are true except that)关键词是except,问下面哪一项是错误的。从题干看不出考哪一段。这类题可能考一段,也可能考好几段,一般以一段居多。浏览选项,发现[A]、、[C]三项的共同点是workers(trade union是workers的集合),是末段谈到的内容。
①The “shareholders”as such had no knowledge of the lives, thoughts or needs of the lives, thoughts or needs of the workmen employed by the company in which the held shares, and his influence on the relations of capital and labour was not good. [A]②The paid manager acting for the company was in more direct relation with the men and their demands, but even he had seldom that familiar personal knowledge of the workmen which the employer had often had under the more patriarchal system of the old family business now passing away. ③Indeed the mere size of operations and the numbers of workmen involved rendered such personal relations impossible. ④Fortunately, workmen to meet on equal terms the managers of the companies who employed them. [D] ⑤The cruel discipline of the strike and lockout taught the two parties to respect each other’s strength and understand the value of fair negotiation.
正确选项分析:[A] the shareholders were unaware of the needs of the workers.(股票持有人没有认识到工人的需要):注意were unaware of与文中had no knowledge of同义替换。 the old firm owners had a better understanding of their workers)(老商号的所有者更好地了解他们的工人):隐藏于原句的从句之中。[D] the trade unions seemed to play a positive role.(工会看起来起到正面的作用):注意文中感情色彩词(fortunately提示positive。Seem含义相对,不容易犯错误。
错误选项分析:[C] the limited liability companies were too large to run smoothly.(有限责任公司太庞大而难以顺利地运转):符合生活常识的合理项,在Except题型中是解。
体会:是非判断题一般考点范围广泛,本题等于把末段的每句话都考到了。所以,考生应句句看懂,不要轻易下结论。
九阴真经解注:根据题干或选项中的关键词回返原文,找到相关句与选项相比较就能得到答案。如果定位困难,可以考虑运用排除法,不符合解的特征及“含有原文中未提到的新概念”等选项可以排除,这样找到正确答案的可能性就大。Fortunately表正评价。

第四题:[D]
题眼研读:态度题。本题(The author is most critical of )问作者对谁最持批评态度,属于主旨题范畴,要求考生纵览全篇,体会字里行间流露的语气,并特别注意most。 <1 >大意是19世纪后半叶有限公司出现,代替了家族企业。 <2 >大意是有限公司的发展带来的结果:出现了股东阶层。股东们由于经济的全球扩张变得异常富有,和外界没有联系,靠吃利息过着悠闲自在的生活。 <3 >说股东对工人情况一无所知,对劳资双方关系的影响也不好,领取薪金的公司经理很少了解工人,工会作用渐起。作者对股东阶层的描述中三处用了引号,分别是“comfortable”classes, “shareholding”,“shareholders”,表示出强烈不满。所以[D]shareholders(股东们)应是作者批得最狠的对象。
干扰项分析:[A] family firm owners(家族式商号的所有者):虽然在 <1 >提到家族式商号所有者的后代管理不善,但在末段说他们和工人的关系良好。作者的态度是贬中有褒,对他们不是只持批评态度的。 landowners(土地所有者):只在 <2 >态度“暖昧”地提到了,不是只有批评。[C]managers(经理们):在 <1 >,作者对经理们持赞扬态度,说他们提高了企业的效率。在末段说经理们对工人的了解不如家族商号所有者,只是稍加批评。
体会:态度题要注意感情色彩词:引号表示批评。
九阴真经解注:引号表反义。

四、新题的讲解
第五题:
题眼研读:词汇题。本题(The phrase“on modern lines”in the opening sentence of the text may be best interpreted as)问“on modern lines”什么意思?大家一看,三个词都认识,属高中词汇,判断考的是熟词僻义,这种题比“难词猜意思”更容易错。解这种题要根据周围语境去分析,本题实际上考的是对①的完整理解。
In the last half of the nineteenth century “capital”and “labour”were enlarging and perfecting their rival organizations on modern lines.
“capital”and“labour”常见意思分别是资本和劳动力,但这里加了引号,引申为“资方”和“劳方”。大意:在19世纪后半叶,“资方”和“劳方”——不断扩大和完善各自相对立的组织。“猜出 following up-to-date methods.(采用最新方式)正确,lines意思是“方式”。注意modern和up-to-date的同义替换。
干扰项分析:[A] going along with social changes(随着社会的发展):有很强的干扰性,因将它带入该句可以自圆其说,但[A]与原文“on modern lines”语义相差太远。[C]keeping to advanced courses.(遵循先进的路线)和[D]moving by the newest routes.(沿着最新的路线移动)都是根据line的字面意思“线”编制的选项。
体会:字面意思不解。既要根据周围语境决定语义,又要考虑文字本身,不过是中国人的中庸之道而已。
第六题:[A]
题眼研读:推理题。题干(The root cause of the old firms’ collapse lies in their)意思是“旧式商号衰败的根本原因是”,是第一题的翻版。[A] inadaptability to satisfy the demands for high efficiency.(无法满足高效率的要求)正确,注意命题者用“大词” inadaptability增加难度。
干扰项分析: failure to engage capable managers to run their business.(未能聘用有能力的经理来管理企业)和[D]deficiency of highly qualified managing professionals(缺乏合格的高级管理人才)谈到的都是管理问题。文中说有限责任公司的产生,提高了效率,但不是根本(root)原因。[C]inability to foresee the rapid development of technology.(未能预见技术的快速发展)为凭空捏造的合理项。
体会:当你发现很多题都是一道题的时候你的水平就提高了。
第七题:[C]
题眼研读:细节判断/观点题。题干(The author points out that)=Which of the following is true?浏览选项,、[C]、[D]有一个共同点:shareholders。本题是第四题的翻版, <2 >③
All through the nineteenth century, America, Africa, India, Australia and parts of Europe were being developed by British capital, and British shareholders were thus enriched by the world’s movement towards industrialization.
大意:整个19世纪,美洲、非洲、印度、澳洲及欧洲的部分地区都靠英国的资本发展起来,而英国股东则因世界性的工业化而大发其财。[C] British shareholders profited most from the tide of modern industrialization.(英国股东们从现代工业化浪潮中获利最多)符合文意。
干扰项分析:[A] limited liability companies brought modernizations to many countries(有限责任公司给许多国家带来了现代化):程度太过,③说其他国家由于英国的资本而发展,没说达到了“先进的”现代化。 great contributions were made by shareholders to British economy.(股东为英国经济做出了重大贡献):作者对股东持批评态度,所以不对。[D] The capital and management of corporations were impaired by shareholders.(股东损害了公司的资金和管理):属过头话,虽然作者对股东持批评态度,但股东也没那么大的危害。
体会:分寸很重要。
第八题:[D]
题眼研读:细节题。本题(In the 19th century British shareholding classes)是第二题的翻版。[D]bore responsibility for the detachment of their riches from company manipulation.(股东阶层对他们的财富与公司运作的分离负有责任)是答案。本题的命制采用了同义替换和结构转换两种方法,词汇手段发挥得淋漓尽致,比较:
原文:irresponsible      wealth    management    detached from
[D]: bore responsibility    riches    manipulation    detachment of…  from…
干扰项分析:[A] enjoyed their remaining years comfortably on their share profits(股东阶层依靠股票利润安度晚年): <2 >④说股东retired on their incomes,该选题把incomes改为share profits,是非常明显的掉包。 enriched their colonies in five continents through investements.(股东通过投资使五大洲的殖民地富有了): <2 >③只是说殖民地发展了,谈不上富了,真正富的人是股东自己。[C]broke off all their connections with the rest of the community(股东断绝了和其他所有人的关系和来往):源于 <2 >④,但把原句中except后面的文字去掉了。
注释:1)on modern lines按现代方式;2)be replaced by被……所取代;3)a limited liability company有限责任公司;4)for the benefit of 为……的利益;5)go into business从商;6)detach from和……分离;7)render… impossible使……成为不可能;8)on equal terms以平等条件

第四篇(1998年Passage 2)
Well, no gain without pain, they say. But what about pain without gain? Everywhere you go in America, you hear tales of corporate revival. What is harder to establish is whether the productivity revolution that businessmen assume they are presiding over is for real.
The official statistics are mildly discouraging. They show that, if you lump manufacturing and services together, productivity has grown on average by 1.2% since 1987. That is somewhat faster than the average during the previous decade. And since 1991, productivity has increased by about 2% a year, which is more than twice the 1978~1987 average. The trouble is that part of the recent acceleration is due to the usual rebound that occurs at this point in a business cycle, and so is not conclusive evidence of a revival in the underlying trend .There is, as Robert Rubin, the treasury secretary, says, a “disjunction”between the mass of business anecdote that points to a leap in productivity and the picture reflected by the statistics.
Some of this can be easily explained. New ways of organizing the workplace-all that re-engineering and downsizing-are only one contribution to the overall productivity of an economy, which is driven by many other factors such as joint investment in equipment and machinery, new technology, and investment in education and training. Moreover, most of the changes that companies make are intended to keep them profitable, and this need not always mean increasing productivity: switching to new markets or improving quality can matter just as much.
Two other explanations are more speculative. First, some of the business restructuring of recent years may have been ineptly done. Second, even if it was well done, it may have spread much less widely than people suppose.
Leonard Schlesinger, a Harvard academic and former chief executive of Au Bong Pain, a rapidly growing chain of bakery cafes, says that much“re-engineering”has been crude. In many cases, he loss of revenue has been greater than the reductions in cost. His colleague, Michael Beer, says that far too many companies have applied re-engineering in a mechanistic fashion, chopping out costs without giving sufficient thought to long-term profitability. BBDO’s Al Rosenshine is blunter. He dismisses a lot of the work of re-engineering consultants as mere rubbish—— “the worst sort of ambulance-chasing.”(386 words 可读性:41.8  难度:11.9级)
一、原题的研究
1.According to the author, the American economic situation is
[A] not as good as it seems.
at its turning point.
[C] much better than it seems.
[D] near to complete recovery.
2.The official statistics on productivity growth
[A] exclude the usual rebound in a business cycle.
fall short of businessmen’s anticipation.
[C] meet the expectation of business people.
[D] fail to reflect the true state of economy.
3.The author raises the question“what about pain without gain?”because
[A] he questions the truth of “no gain without pain”.
he does not think the productivity revolution works.
[C] he wonders if the official statistics are misleading.
[D] he has conclusive evidence for the revival of businesses.
4.Which of the following statements is NOT mentioned in the passage?
[A] Radical reforms are essential for the increase of productivity.
New ways of organizing workplaces may help to increase productivity.
[C] The reduction of costs is not a sure way to gain long-term profitability.
[D] The consultants are a bunch of good for nothings.
二、潜在命题点的挖掘
请参考后面的“三、原题的讲解”彻底弄懂原题后,再将文章读两遍,思考一个什么地方还可出题,出什么样的题。请继续做下面的试题:
5.From the first paragraph we know that
[A] no efforts are made without successful results.
American corporations are going to thrive again.
[C] the stories about productivity revolution are questionable.
[D] businessmen are manipulating their production improvement.
6.Official statistics show that in the late 1980’s, productivity
[A] developed at a far greater speed than that of the previous decade.
grew at a remarkable rate twice greater than the average in 1970’s
[C] increased by over 2% per year as against 1% in the earlier decade.
[D] rose by more than 1% per year as compared with the average during 1970’s.
7. In Robert Rubin’s opinion
[A] the recovery of national economy lacks convincing proof.
the statistics predict the approaching of productivity revolution.
[C] the story about a sudden rise in production is just imaginary.
[D] the scene of a leap in productivity is depicted by the statistics.
8. Business restructuring and workplace re-engineering and downsizing are aimed at
[A] drawing more investments in equipment and job training.
accelerating the application of new technology to production.
[C] shifting to new markets and improving the quality of products.
[D] augmenting revenues and reducing costs of production.
三、原题的讲解
第一题:[A]
题眼研读:变形的主旨题。本题(According to the author, the American economic situation is)问作者对美国经济形势的观点。 <1 > <2 >段落大意是:美国经济形式不如想象的乐观。因此[A]not as good as it seems.(不如表面看上去的那么好)是作者的观点,正确。
干扰项分析: at its turning point.(处在转折点上):从 <2 >⑤at this point 断章取义而来,文不对题。[C] much better than it seems.(比表面上好得多):为商家错误观点。[D] near to complete recovery.(几乎完全恢复):更不对。
九阴真经解注:段中心常考,含义深刻是解,中心思想是解。且AC项反义解在其中。
第二题:
题眼研读:细节题。本题(The official statistics on productivity growth)问官方关于生产率提高的统计数字说明了什么问题。根据题干中关键词statistics容易判断考点大致位置在 <2 >,只有 <2 >出现了official statistics。该段中比较难的句子是末句,根据难句、长句必考的原则,判断考点具体位置在该句。倒数②也长,但难度不够。
There is, as Robert Rubin, the treasury secretary, says, a“disjunction”between the mass of business anecdote that points to a leap in productivity and the picture reflected by the statistics.
a disjunction分离,分岐;mass of business anecdote大量的企业界传言;picture应理解成“情景”。文意:商界大量传说的生产率大幅度提高的神话与官方统计数字所显示的情景并不吻合。fall short of businessmen’s anticipation.(官方有关生产率提高的调查统计并未达到商业家们的预期目标)是本句的改写。请观察与原文的词汇比较
原文 a disjunction between     business…       a leap        picture reflected
fall short of               businessmen’s    anticipation    题干
干扰项分析:[A] exclude the usual rebound in a business cycle.(不包括商业循环中通常会出现的反弹)、[C] meet the expectation of business people.(达到了商业家们的预期目标)、[D] fail to reflect the true state of economy(没有反映出国家经济的真实情况)均与原文的意思相反。注意[A]和[C]大量采用了文中词汇以加强干扰度。
九阴真经解注:矛盾常考,中心思想是解,BC项反义解在其中。
第三题:
题眼研读:从表面上看这是一道细节题,本质上是在考主旨。本题(The author raises the question“what about pain without gain?” because)问引号部分的意思。请注意本题的宏大构思:虽然出处是文章的第一段,但必须通读全文才能正确解答。本篇文章是社会现象分析型文章, <1 >和 <2 >提出一个社会现象“美国经济不如商家认为的那样乐观。” <3 >至末段分析原因,认为主要原因是生产效率革命不见效。从字面上讲“pain without gain”的意思是“劳而无获。”“劳”暗指企业的改革,“无获”比喻企业的改革措施没有效果,“劳而无获”实际上是指 <1 >和 <2 >提出的社会现象。题干中的because提示本题问的是原因,也就是 <3 >至末段分析原因的总结。 He does not think the productivity revolution works.(他认为这场生产率革命未见成效)正是原因,也是文章的中心所在。
干扰项分析:题干中含有一个问题,所以[A] he questions the truth of “no gain without pain.”(他怀疑“不劳无获”这句话是否真实)、[C] he wonders if the official statistics are misleading.(他怀疑官方的数据是否有误导作用)均为字面意思,不是解。[D] he has conclusive evidence for the revival of businesses.(他对经济复苏有结论性的证据),使用了不少原文的词语如conclusive, evidence, revival of businesses,但意思和文意相反。
体会:字面意思不是解。
九阴真经解注:文章中心常考,含义深刻是解,中心思想是解。字面不是解,具体不是解,绝对不是解。
第四题:[A]
题眼研读:Except题型。题干(Which of the following statements is NOT mentioned in the passage?)关键词是NOT。
正确选项分析:本题正确选项涉及 <3 > <5 >两段:
<3 >②In many cases, he believes, the loss of revenue has been greater than the reductions in cost.③His colleague, Michael Beer, says that far too many companies have applied re-engineering in a mechanistic fashion, chopping out costs without giving sufficient thought to long-term profitability.④BBDO’s Al Rosenshine is blunter. ⑤He dismisses a lot of the work of re-engineering consultants as mere rubbish——“the worst sort of ambulance-chasing.”
②~③大意:收入的损失大于成本的减少,削减成本没有考虑长远的利益,成本的降低并没有带来收入的增加。所以[C] The reduction of costs is not a sure way to gain long-term profitability.(成本降低不一定能获得长远利益)正确。⑤大意:他对重组顾问们做的大量工作不屑一顾,因为那些完全是垃圾——典型的劳而无获。[D] The consultants are a bunch of good for nothings(顾问是一群饭桶)是⑤的改写,比较:
原文      a lot of           mere rubbish
[D]      a bunch of        good for nothings
错误选项分析:[A] Radical reforms are essential for the increase of productivity.(彻底变革是提高生产率的必要条件):文章只是分析了企业的种种reform措施并不意味着the increase of productivity, [A]推论太过头。
体会:真理和谬误仅仅一步之遥。
九阴真经解注:复杂句常考,含义深刻常考,强烈感情色彩常考,EXCEPT题型合理项是解。
四、新题的讲解
第五题:[C]
题眼研读:段中心题。题干(From the first paragraph we know that)明示考 <1 >:
①Well, no gain without pain, they say. ②But what about pain without gain?③Everywhere you go in America, you hear tales of corporate revival.④What is harder to establish is whether the productivity revolution that businessmen assume they are presiding over is for real.
①“有人说不劳无获。” ②转折,“但劳而无获呢?” ③“在美国无论你走到哪你都会听到有关公司复苏的传言。” ④转折,“难以确定的商家假设的他们主持的生产率革命是不是真的。”本段采用的欲扬先抑的写作方法。①、③分别被②、④所否定。[C]the stories about productivity revolution are questionable.(关于生产率革命的神话值得质疑)正确。
干扰项分析:[A]no efforts are made without successful results.( 不劳无获):①变体,和中心意思相反。American corporations are going to thrive again.(美国公司再次兴旺发达):和③相反。[D] businessmen are manipulating their production improvement.(商家在进行生产的改进):断章取义于③定语从句中的preside over(掌管)而成,与文意相差太远。
体会:中心思想永远是解题的关键。
第六题:[D]
题眼研读:细节题。根据题干(Official statistics show that in the late 1980’s, productivity)中关键词official statistics判断考点在 <2 >,粗看四个选项考点是数字问题。 <2 >②③④都涉及数字:
②They show that, if you lump manufacturing and services together, productivity has grown on average by 1.2% since 1987.③That is somewhat faster than the average during the previous decade.④And since 1991, productivity has increased by about 2% a year, which is more than twice the 1978~1987 average.
大意:数据表明,如果把制造业和服务业放在一起,自1987年以来,生产率平均每年上升1.2%,这比前十年的平均增长指数要快。自1991年以来,生产率每年上升大约2%,这是1978~1987年度平均增长指数的两倍。[D](in the late 1980’s, productivity)rose by more than 1% per year as compared with the average during 1970’s.(80年代末期的生产率比70年代每年平均增长1%多一点)是②③的合并,既正确,又有概括性。并注意词汇替换:rise=grow, late 80’s=1987, during 1970’s=the previous decade, more than 1%=1.2%。
干扰项分析:[A] developed at a far greater speed than that of the previous decade.(比上一年十年发展速度快得多):把文中somewhat换成了a far greater speed,程度太过。grew at a remarkable rate twice greater than the average in 1970’s(以比70年代发展速度快1倍的速度增长)和[C] increased by over 2% per year as against 1% in the earlier decade.(与上一个十年1%的增长率相比超过2%)都源于④,但④说的是1991年以来的事,和[C]把它当成80年代末。

第七题:[C]
题眼研读:细节题。根据题干(In Robert Rubin’s opinion)中关键词——人名找到考点在 <2 >末句。本题是第二题的翻版。[C] the story about a sudden rise in production is just imaginary.(关于生产突然增加的说法只是假想的)正确。注意同义替换和结构转换:stories=anecdote, sudden rise =leap, imaginary是对disjunction的彻底理解后的近似解释。原句是存在结构,[C]为主系表结构。
干扰荐分析:[A] the recovery of national economy lacks convincing proof.( 国民经济的复兴缺乏令人信服的证据):干扰性能极佳。错在national economy这两个字上,如果调换成leap in productivity则正确,可见偷梁换柱的利害。The statistics predict the approaching of productivity revolution.(统计数字预示了生产率革命将要来临):借用statistics, points to等词编造而成,完全错误。[D]the scene of a leap in productivity is depicted by the statistics.(统计数字描述了生产率跃进的画面);借用后半句个别词汇编制,不合文意。

第八题:[D]
题眼研读:关于对复杂句理解的细节题。根据题干(Business restructuring and workplace re-engineering and downsizing are aimed at)问改革的目的是什么。根据关键词——三个动名词找到 <3 >②③。
②New ways of organizing the workplace—all that re-engineering and downsizing— are only one contribution to the overall productivity of an economy, which is driven by many other factors such as joint investment in equipment and machinery, new technology, and investment in education and training. ③Moreover, most of the changes that companies make are intended to keep them profitable, and this need not always mean increasing productivity: switching to new markets or improving quality can matter just as much.

目的(intended to)是使公司更加有利可图,所以[D]augmenting revenues and reducing costs of production.(增加收入,降低成本)=profitable,正确。
干扰项分析:[A] drawing more investment in equipment and job training.(吸引更多设备和职业培训投资):取材于② such as后, accelerating the application of new technology to production.(加快新技术在生产中的应用)借用了new technology, production,编造了accelerating the application, [C] shifting to new markets and improving the quality of products.(开辟新市场和改进产品的质量)借用③后半部分的主语。这几个干扰项大量借用文中词汇,但与文意出入很大。
体会:要培养一种见到照搬原文章中的词汇就不舒服的感觉。
注释:1)corporate公司的;2)revival复苏,复兴,复活;3)preside over主持,管理;4)statistics统计,统计数字;5)lump…together合在一起;6)disjunction分离,分裂,7)re-engineering重新策划,重新建造,重组;8)downsizing裁员;9)speculative推测的,猜测性的,思辩的,纯理论的;10)ineptly不合适地,不恰当地,笨拙地;11)revenue收入;12)chop out大幅度削减

第五篇 (1999年Passage 2)
In the first year or so of Web business, most of the action has revolved around efforts to tap the consumer market. More recently ,as the Web proved to be more that a fashion, companies have started to buy and sell products and services with one another. Such business-to-business sales make sense because business people typically know what product they’re looking for.
Nonetheless, many companies still hesitate to use the Web because of doubts about is reliability.“Businesses need to feel they can trust the pathway between them and the supplier,”says senior analyst Blane Erwin of Forrester Research. Some companies are limiting the risk by conducting online transactions only with established business parthers who are given access to the company’s private intranet.
Another major shift in the model for Internet commerce concerns the technology available for marketing. Until recently, Internet marketing activities have focused on strategies to “pull”customers into sites. In the past year, however, software companies have developed tools that allow companies to “push” information directly our to consumers, transmitting marketing messages directly to targeted customers. Most notably, the Pointcast Network uses a screen save to deliver a continually updated stream of news and advertisements to subscribers’ computer monitors. Subscribers can customize the information they want to receive and advertisements to a company’s Web site. Companies such as Virtual Vineyards are already starting to use similar technologies to push messages to customers about special sales, product offerings ,or other events. But push technology has earned the contempt of many Web users. Online culture thinks highly of the notion that the information flowing onto the screen comes there by specific request. Once commercial promotion begins to fill the screen uninvited, the distinction between the Web and television fades. That’s prospect that horrifies Net purists.
But it is hardly inevitable that companies on the Web will need to resort to push strategies to make money. The examples of Vineyards, Amazon. com, and other pioneers show that a Web site selling the right kind of products with the right mix of interactivity, hospitality, and security will attract online customers. And the cots of computing power continues to free fall,which is a good sign for any enterprise setting up shop in silicon. People looking back 5 or 10 years from now may well wonder why so few companies took the online plunge.(396 words 可读性:38.6   难度:12级)
一、原题的研究
1. We learn from the beginning of the passage that Web business
[A] had been striving to expand its market.
intended to follow a fanciful fashion.
[C] tried but in vain to control the market.
[D] has been booming for one year or so.
2. Speaking of the online technology available for marketing, the author implies that
[A] the technology is popular with many Web users.
businesses have faith in the reliability of online transactions.
[C] there is a radical change in strategy.
[D] it is accessible limitedly to established parthers.
3.In the view of Net purists
[A] there should be no marketing messages in online culture.
money making should be given priority to on the Web.
[C] the Web should be able to function as the television set.
[D] there should be online commercial information without requests.
4. We learn from the last paragraph that
[A] pushing information on the Web is essential to Internet commerce.
interactivity, hospitality and security are important to online customers.
[C] leading companies began to take the online plunge decades ago.
[D] setting up shops in silicon is independent of the cost of computing power.
二、潜在命题点的挖掘
请参考后面的“三、原题的讲解”彻底弄懂原题后,再将文章读两遍,思考一个什么地方还可出题,出什么样的题。请继续做下面的试题:
5. Many companies’ hesitation in using the Web business
[A] caused doubts about its reliability.
hindered it from quick popularization.
[C] aroused suspicion of its being trustworthy.
[D] minimized the risk of failure in online transactions.
6.The expression“established business partners”(Lines 3~4, para.2)refers to businesses that have
[A] gained full recognition and acceptance in commerce.
been set up on firm financial foundations.
[C] got unlimited credit in online transactions.
[D] placed themselves in steady positions of profitability.
7.According to the text, “push”strategy has
[A] promoted the advance of Internet commerce.
helped the progress of message transmission.
[C] won high praise on the part of online culture.
[D] incurred despise of a host of Web users.
8.In order to be profitable, online companies
[A] need to set up silicon shops to make computer parts.
have to customize information for their customers.
[C] can adopt some better policies than push strategies.
[D] should plunge themselves into online marketing.

三、原题的讲解

第一题:[A]

题眼研读:段落大意题。题干(We learn from the beginning of the passage that Web business)关键词是Web business,明示考点在 <1 >。
① In the first year or so of Web business, most of the action has revolved around efforts to tap the consumer market. ②More recently, as the Web proved to be more than a fashion, companies have started to buy and sell products and services with one another.③Such business-to-business sales make sense because business people typically know what product they’re looking for.
大意:Web business 在第一年主要开发消费者市场,最近开始做公司的生意。可见[A] had been striving to expand is market.(一直努力开拓市场)正确。注意:strive to=revolved around efforts to, expand is market=tap the market。本题的特色是结合段落大意考对难短语“tap the market”(开拓市场)的理解。
干扰项分析: intended to follow a fanciful fashion.(追赶新奇、时髦):取材于②,语义相反。[C]tried but in vain to control the market.(网络业务试图控制市场,但未奏效):是将①中 tap the market 错误理解成“控制市场”的结果。[D] has been booming for one year or so.(已经蓬勃发展了一年左右的时间):程度太过了。
九阴真经解注:段落句常考。

第二题:[C]

题眼研读:通过细节考段中心。题干(Speaking of the online technology available for marketing, the author implies that)的关键词是online technology available for marketing(为网上营销提供的技术)。实际上本题是通过该关键词考 <3 >中心,即由于新技术的出现网络营销策略发生了巨大变化,从吸引变成推销。所以[C] there is a radical change in strategy.(在策略上有根本的改变)正确。
干扰项分析:[A] the technology is popular with many Web user.(这项技术受到了许多网络用户的喜爱):根据生活常识编造的合理项。 businesses have faith in the reliability of online transactions.(公司对在线交易的可靠性充满信心):和 <2 >文意正好相反。[D] it is accessible limitedly to established parthers. (它仅限于对固定合作伙伴使用); <2 >③说网上交易仅对固定合作伙伴使用,[D] 张冠李戴,将网上交易偷换成网上技术。
体会:合理项不是解。
九阴真经解注:变化是解,until表转变。
第三题:[D]

题眼研读:细节题。本题(In the vies of Net purists)问及网络净化者的态度。根据关键词Net purists找到 <3 >最后三句。
Online culture thinks highly of the notion that the information flowing onto the screen comes there by specific request. Once commercial promotion begins to fill the screen uninvited, the distinction between the Web and television fades. That’s a prospect that horrifies Net purists.
大意:网络文化主张信息应根据用户的要求而提供,而一旦商业促销不请自来,网络与电视就没多大差异了。这可是网络纯净主义者所惧怕的前景。可见[D] There should be no online commercial information without requeste.(不经要求就不应提供须线信息)正确。难点是代词that的理解。Net purists害怕that,也就是反对that,主张that的反面,that代替的是“信息不请自来”。[D] 是原文的否定之否定,比较:
原文:The information comes there by specific request.
[D]:No online commercial information without requests.
干扰项分析:[A] There should be no marketing messages in online culture.(网络文化不应该有营销信息):程度太过。 Money making should be given priority to on the Web.(上网的首要目的应该是为了赚钱):根据生活常识编造的合理项,再说钱这种庸俗的东西轻易不要选。[C] The Web should be able to function as the television set.(网络的功能应该像电视一样):是对倒数第二句的字面理解,和文意相反。
体会:双重否定是解,钱是常用的合理项。
九阴真经解注:反着考,倒着考,相反支持,双重否定是解,复杂是解。本文通过感情词汇horrify表明Net purist反对商业信息不请自来,支持商业信息在特殊要求下才能送达。
第四题:

题眼研读:细节题。题干(We learn from the last paragraph that)确定考点在末段。
①But it is hardly inevitable that companies on the Web will need to resort to push strategies to make money. ②The examples of Virtual Vineyards, Amazon. com, and other pioneers show that a Web site selling the right kind of products with the right mix of interactivity, hospitality, and security will attract online customers. ③And the cost of computing power continues to free fall, which is a good sign for any enterprise setting up shop in silicon. ④People looking back 5 or 10 years from now may well wonder why so few companies took the online plunge.

我们还要反复提到长难句是考点,①难点在于hardly inevitable的意思是“不是不可以避免的”,即“是可以避免的”。②最长:The examples…show为主谓语,从that 到句尾是宾语从句;宾语从句的主语是a Web site,selling… security是Web site定语,will…customers为宾语从句的谓宾部分。大意是Virtual Vineyards和Amazon.com(两个网站的名称)以及其他开拓者的例子表明:销售对路产品的网站,加上相互合作、礼貌周到、安全可靠这几方面恰到好处地结合起来,将同样能吸引网上客户。由此推断 interactivity, hospitality and security are important to online customers.(互动、友好及安全对网络顾客很重要)正确。注意同义转换+结构转换:important=attract,原句的介词宾语换成主语。并且考到了隐蔽处(宾语从句主语的定评。)
干扰项分析:[A]pushing information on the Web is essential to Internet commerce.(在网上‘推销’信息对网上商业来说至关重要):与①相反。[C] Leading companies began to take the online plunge decades ago.(一些大公司在几十年前就开始积极尝试网络服务):细微之处见真功,[C]错在decades ago上,④说5~10年。[D] setting up shops in silicon is independent of the cost of computing power.(建立网络销售点不受计算机功率成本的制约):与③正好相反。
九阴真经解注:重要是解,同义替代是解。注意attrcct与inportant的替代

四、新题的讲解

第五题:

题眼研读:细节题。题干(Many companies’ hesitation in using the Web business)是 <2 >①的名词化处理。
①Nonetheless, many companies still hesitate to use the Web because of doubts about its reliability.②“Businesses need to fell they can trust the pathway between them and the supplier,”says senior analyst Blane Erwin of Forester Research. ③Some companies are limiting the risk by conducting online transactions only with established business partners who are given access to the company’s private intranet.
命题者将末句“有些公司只和固定企业做生意”概括成 hindered it from quick popularization.(阻碍了网上交易的快速普及)
干扰项分析:[A] caused doubts about its reliability.(导致了对其可靠性的怀疑):基本使用了原文的词汇来编造,因果关系倒置。[C] aroused suspicion of its being trustworthy.(产生了对Web business可信任的怀疑):因果关系倒置。[D] minimized the risk of failure in online transaction.(将网上交易失败的风险减到最小):程度太过。③说一些公司对使用网络比较犹豫,试图减少风险,但没有说它的做法使失败的风险减到最小。

第六题:[A]

题眼研读:词汇题。本题[The expression“established business partners”(Lines 3-4, Para,2) refers to businesses that have]考established的场合意。Established的常见意思是“确定的、固定的”,但考研不可能考这个意思,我们要根据周围语言环境—— <2 >选择词义解题:
①Nonetheless, many companies still hesitate to use the Web because of doubts about its reliability.②“Businesses need to feel they can trust the pathway between them and the supplier,”says senior analyst Blane Erwin of Forester Research. ③Some companies are limiting the risk by conducting online transactions only with established business partners who are given access to the company’s private intranet.
③意思:有些公司为了减少风险,只和    交易伙伴进行网络交易,这些贸易伙伴可以进入公司内部的局域网。结合本段中心即“信任”,可以推断[A] gained full recognition and acceptance in commerce.(赢得了商界的充分认可和接受)正确。
干扰项分析: been set up on firm financial foundations.(建立在牢固的资金基础之上)和[D] placed thernselves in steady positions of profitability.(使他们自己的赢利稳定)都是根据established的常见意思编造的选项,且与上下文脱节,不能自圆其说。[C] got unlimited credit in online transactions.(网络交易中获得无限信誉的企业):unlimited太绝对了。
体会:对于熟词,常见意义不是解,字面意义不是解,与周围语境相吻合、体现段(文章)中心的是解。

第七题:[D]

题眼研读:细节题。题干(According to the text,“push”strategy has)中关键词push的意思要在文中 <3 >倒数③④体现。
But push technology has earned the contempt of many. Web users. Online culture thinks highly of the notion that the information flowing onto the screen comes there by specific request.
命题者把原文改写成[D] incurred despise of a host of Web users.(招致了一群网民的鄙视)。
干扰项分析:[A] promoted the advance of Internet commerce.(促进了网络商业的进步)和 helped the progress of message transmission.(有助于信息传递的进步)均为根据常识编制的合理项。[C] won high praise on the part of online culture.(赢得了网络文化的高度赞扬):断章取义,与文意相反。online culture think highly of=won high praise。

第八题:[C]

题眼研读:这是一道大意归纳题。题干(In order to be profitable, inline companies)中to be profitable和末段①to make money同义,本题实际上是第四题的翻版,不过将答案虚化成[C] can adopt some better policies than push strategies. (能采用一些比推销更好的方法)
干扰项分析:[A] need to set up silicon shops to make computer parts. (需要建立计算机商店来制造计算机配件);取材于倒数②,逻辑不能。 have to customize information for their customers. (不得不为他们的顾客定制信息):取材于 <3 >⑤,把主语“顾客”篡改成“公司”。[D] should plunge themselves into online marketing. (应该冒险投身于网络营销):取材于末段末句,逻辑不通。
注释:1)revolve around围绕着;2)tap开拓;3)make sense有道理,有意义;4) pathway [计算机用语]路径,通道;5)online transaction在线交易,即批网上交易;6)intranet指内部局域网,与internet对应;7)subscriber指上网者,即网络用户;8)take the plunge采取断然措施。
于:2003-05-27 18:30:53         


第六篇(1999年Passage 3)
An invisible border divides those arguing for computers in the classroom on behalf of students, career prospects and those arguing for computers in the classroom for broader reasons of radical educational reform. Very few writers on the subject have explored this distinction — indeed, contradiction — which goes to the heart of what is wrong with the campaign to put computers in the classroom.
An education that aims at getting a student a certain kind of job is a technical education, justified for reasons radically different from why education is universally required by law. It is not simply to raise everyone’s job prospects that all children are legally required to attend school into their teens. Rather, we have a certain conception of the American citizen, a character who is incomplete if he cannot competently assess how his livelihood and happiness are affected by things outside of himself. But this was not always the case; before it was legally required for all children to attend school until a certain age, it was widely accepted that some were just not equipped by nature to pursue this kind of education. With optimism characteristic of all industrialized countries, we came to accept that everyone in fit to be educated. Computer education advocates forsake this optimistic notion for a pessimism that betrays their otherwise cheery outlook. Banking on the confusion between educational and vocational reasons for bringing computers into schools, computer-ed advocates often emphasize the job prospects of graduates over their educational achievement.
There are some good arguments for a technical education given the right kind of student. Many European schools introduce the concept of professional training early on in order to make sure children are properly equipped for the professions they want to join. It is, however, presumptuous to insist that there will inly be so many jobs for so many scientists, so many businessmen, so many accountants. Besides, this is unlikely to produce the needed number of every kind of professional in a country as large as ours and where the economy is spread over so many states and involves so many international corporations.
But, for a small group of students, professional training might be the way to go since well-developed skills, all other factors being equal, can be the difference between having a job and not. Of course, the basics of using any computer these days are very simple. it does not take a lifelong acquaintance to pick up various software programs. If one wanted to become a computer engineer, that is, of course, an entirely different story. Basic computer skills take — at the very longest —a couple of months to learn. In any case, basic computer skills are only complementary to the host of real skills that are necessary to becoming any kind of professional. It should be observed, of course, that no school, vocational or not, is helped by a confusion over its purpose. (487 words  可读性:38  难度:12级)

一、原题的研究
1.  The author thinks the present rush to put computers in the classroom is
[A] far reaching.                       dubiously oriented.
[C] self-contradictory.                   [D] radically reformatory.
2.  The belief that education is indispensable to all children
[A] is indicative of a pessimism in disguise.
came into being along with the arrival of computers.
[C] is deeply rooted in the minds of computer-ed advocates.
[D] originated from the optimistic attitude of industrialized countries.
3.  It could be inferred from the passage that in the author’s country the European model of professional training is
[A] dependent upon the starting age of candidates.
worth trying in various social sections.
[C] of little practical value.
[D]attractive to every kind of professional.
4.  According to the author, basic computer skills should be
[A] included as auxiliary course in school.
highlighted in acquisition of professional qualifications.
[C] mastered through a life-long course.
[D] equally emphasized by any school, vocational or otherwise.

二、潜在命题的挖掘
请参考后面的“三、原题的讲解”彻底弄懂原题后,再将文章读两遍,思考一下什么地方还可出题,出什么样的题。请继续做下面的试题:
5.   Compulsory education requires that
[A] computers should be adopted in classroom teaching.
vocational study be absolutely necessary to students.
[C] all school-age children receive universal education.
[D] job training open up broad prospects for the youth.
6.  The concept that nobody is unsuitable for education
[A] is justified by universal education requirements.
is discarded by computer-education supporters.
[C] is carried out smoothly by all developed nations.
[D] is verified by graduates’ educational achievements.
7.  Computer-ed advocates
[A] depend on the confusion between irrefutable arguments.
give away their true intention of using computers in schools.
[C] take advantage of the different reasons for computer employment.
[D] make lots of profits from introducing computers into schools.
8.  The insistence that enough jobs will be available for various professionals is
[A] very bold and agitating.
very prompt and inspiring.
[C] too deliberate to be incredible.
[D] too arbitrary to be well-founded.

三、原题的讲解

第一题:
题眼研读:态度题。本题(The author thinks the present rush to put computers in the classroom is)问:对于目前存在的蜂拥而上地推行计算机教学的现象作者持何种看法?这正是本文谈论的主题。前面计过,做观点题要注意两点,一从局部上要注意感情色彩词汇;二要有大局观,归纳各段大意。本文 <1 >提出计算机教学出于职业教育需要和革命性教育改革需要的矛盾,认为这个矛盾是计算机错误地进入课堂的关键性问题; <2 >末句说: Banking on the confusion between educational and vocational reasons for bringing computers into schools…文章的结论——末段末句说:It should be observed .of course, that no school, vocational or not, is helped by a confusion over its purpose. 所以dubiously oriented.(其目的值得怀疑)=confusion over its purpose是作者的观点。
干扰项分析:[A] far reaching.(意义深远的):出题人根据我国“国情”编出来的合理项。[C] self-contradictory.(自相矛盾的):注意关于计算机进入课堂的两种观点是相互矛盾的,不是计算机进入课堂本身相互矛盾,或者说跟谁矛盾。[D]radically reformatory.(根本改革性的):根据 <1 >①中radical, reform编造,是作者提到的两种观点中的一种,不是作者的观点。
九阴真经解注:隐蔽处(介词短语)有解,含义不肯定是解,同义替代是解。
第二题:[D]

题眼研读:细节题。本题(The belief that education is indispensable to all children)问所儿童都需要教育的观点是什么。在 <2 >找到all children, we came to accept, education等关键词:
  ①An education that aims at getting a student a certain kind of job is a technical education, justified for reasons radically different from why education is universally required by law. ②It is not simply to raise everyone’s job prospects that all children are legally required to attend school into their teens. ③Rather, we have a certain conception of the American citizen, a character who is incomplete if he cannot competently assess how his livelihood and happiness are affected by things outside of himself. ④But this was not always the case; before it was legally required for all children to attend school until a certain age, it was widely accepted that some were just not equipped by nature to pursue this kind of education. ⑤With optimism characteristic of all industrialized countries, we came to accept that everyone is fit to be educated. ⑥Computer education advocates forsake this optimistic notion for a pessimism that betrays their otherwise cheery outlook. ⑦Banking on the confusion between educational and vocational reasons for bringing computers into schools, computer-ed advocates often emphasize the job prospects of graduates over their educational achievement.
⑤“随着所有工业化国家都存在的乐观情绪的出现,人们开始接受每个人都适合受教育的观念”,可见[D]originated from the optimistic attitude of industrialized countries. (起源于工业化国家的乐观主义态度)是答案。本题的关键是题干中children与⑤中everyone的替换。
干扰项分析:[A] is indicative of a pessimism in disguise. (预示伪装下的悲观主义态度):取材于⑥。 came into being along with the arrival of computers. (随着计算机的诞生而存在):取材于⑤。[C] is deeply rooted in the minds of computer-ed advocates. (深深的植根于主张计算机教育的人们的思想中)。这三项的出题方法很类似,均错误地把题干的education理解为计算机教育、偷梁换柱。
体会:研究生的特征之一是思维清楚。
九阴真经解注:倒着考,起源是解,含义深刻是解,oriented是解。
第三题:[C]

题眼研读:段落大意推理题。本题(It could be inferred from the passage that in the author’s country the European model of professional training is)问欧洲职业培训模式在作者的国家里如何。 <3 >②出现了European这个词。
  ①There are some good arguments for a technical education given the right kind of student. ②Many European schools introduce the concept of professional training early on in order to make sure children are properly equipped for the professions they want to join. ③It is, however, presumptuous to insist that there will only be so many jobs for so many scientists, so many businessmen, so many accountants. ④Besides, this is unlikely to produce the needed number of every kind of every kind of professional in a country as large as ours and where the economy in spread over so many states and involves so many international corporations.
   ②提出欧洲职业训练模式,③和④否之。[C] of little practical value. (几乎没有什么实际价值)正确。[C]从词汇使用上找不到原文的任何痕迹,是原文的概括。
这里需要提出的是关于难词的处理。很多考生不认识③中presumptuous, 其实认不认识这个词对解题没有多大的关系,有时候不认识更好,免得因自己熟知词义而掉入出题者布下的陷阱。这个时候要从句子的整体含义出发:“然而,硬功夫说有如此多的工作等着如此多的科学家、商人、会计师去做,这种观点是              。”答案必然是否定的,有很多可能,如:错误的,荒谬的,没有根据的,主观的。有可能我们所想出来的意思没有一个和词典上的意思相同,但它是符合逻辑的,不会影响我们的理解。请看该词在字黄中的释义:不客气的,放肆的,冒昧的,自以为是的,专横的,傲慢的。
干扰项分析:[A] dependent upon the starting age of candidates. (取决于学员开始接受培训的年龄):是凭空捏造的合理项,不合文意。 worth trying in various social sections. (值得在社会各界尝试):和文意相反。[D] attractive to every kind of professional. (对各种专业人员来说都具有吸引力):更像合理项,取材于④
九阴真经解注:段中心常考,中心思想是解。
第四题:[A]
题眼研读:细节题。题干(According to the author, basic computer skills should be)关键词basic computer skills出现在末段⑥:
  ③ It does not take a lifelong acquaintance to pick up various software programs. ④ If one wanted to become a computer engineer, that is, of course, an entirely different story. ⑤ Basic computer skills take — at the very longest —— a couple of months to learn. ⑥ In any case, basic computer skills are only complementary to the host of real skills that are necessary to becoming any kind of professional. ⑦ It should be observed, of course, that no school, vocational or not, is helped by a confusion over its purpose.
  ⑥“计算机基本技能只是任何一种专业人员所需要的各种职业技能的补充。”因此[A] included as an auxiliary course in school. (应作为学校的辅助课程)正确。Complementary to=auxiliary。
干扰项分析: highlighted in acquisition of professional qualifications. (在专业资质认证中加以强调):取材于⑥定语从句部分,与原文不符。[C] mastered through a life-long course. (花毕生的时间去掌握):取材于③,但去掉了文中not,与文意相反。[D] equally emphasized by any school, vocational or otherwise. (应该受到任何学校,无论是职业学校还是一般学校的重视):取材于⑦,不符合文意。
九阴真经解注:绝对语气常考,含义深刻是解,替代是解。合理项不是解。
四、新题的讲解

第五题:[C]

题眼研读:细节题。题干(Compulsory education requires that)的意思是:义务教育要求什么。根据“义务”这个词找线索。文章中没有出现该词,但在 <2 >①②我们发现其近义词by law, legally:
  ① An education that aims at getting a student a certain kind of job is a technical education, justified for reasons radically different from why education is universally required by law. ② It is not simply to raise everyone’s job prospects that all children are legally required to attend school into their teens.
②是强调句,强调的是表目的的不定式短语,it指代的是“that all…teens.”也是本题的考点。注意本题考点不是作者讲话强调的重点,这种命题思维是考研的特色,可以归纳为“隐蔽处常考”。如果你不理解,很多题做起来会很难受。①②大意:技术教育的目的是给学生找到某种工作,其存在的理由是和法律一般规定的教育存在的理由不同。所有儿童都按法律规定上学直到十七、八岁,这不仅仅使人们的就业前景看好。所以[C] all school-age children receive universal education. (所有学龄儿童都必须接受普遍的教育)正确,另外需要指出的是本题不用看原文,通过一般常识就可判断选项[C] all school-age children receive universal education. (所有学龄儿童都必须接受普及教育)是答案。因为根据我国和外国的义务教育法,儿童到上学年龄就必须上学。不过这种出题的情况在考研英语中是少见的。
干扰项分析:[A] computers should be adopted in classroom teaching. (计算机在课堂教学中应该使用):取材于 <1 >末句。 vocational study be absolutely necessary to students. (职业学习对于学生来说绝对必要):取材于 <2 >①中justified for, 出题人把后面的内容省去了。[D] job training open up broad prospects for the youth. (职业培训为年轻人开辟了广阔的前景):取材于 <2 >②。以上三项均不符合文意。
体会:琢磨一下考隐蔽处的感觉。

第六题:

题眼研读:细节题。题干(The concept that nobody is unsuitable for education)中有个否定之否定结构,变为肯定结构后意思是“人人都适合受教育的观念”和第二题题干(education is indispensable to all children)是一回事,第二题考的是 <2 >⑤,本题考的是 <2 >⑥“计算机教育的是提倡者们抛弃了这一乐观的认识,代之以一种背离了他们原本应有的快乐态度的悲观论调。”所以 is discard by computer-education supporters. (被计算机教育支持者所抛弃)正确。注意替换:题干本身就是原文的否定之否定;discard=forsake; supporters=advocates。
干扰项分析:[A] is justified by universal education requirements. (被普及教育要求证明是有道理的):取材于 <2 >①,如第二题所述,人人都该受教育来自工业国家的态度,不合文意。[C] is carried out smoothly by all developed nations.(在所有发达国家执行很顺利):取材于 <2 >⑤,语义明显不舍文意,而且过于绝对。[D] is verified by graduates’ educational achievements. (被毕业生的教育成绩所证实):取材于 <2 >⑦,曲解原意。

第七题:[A]

题眼研读:细节题。题干(Computer-ed advocates)出现在 <2 >末句:
  Banking on the confusion between educational and vocational reasons for bringing computers into schools, computer-ed advocates often emphasize the job prospects of graduates over their educational achievement.

bank on 作动词,意思是“依赖,指望”。全句大意是:基于这种将计算机引入学校的“教育论”和“职业论”论点之间的混淆不清,计算机教育的提倡者常常只强调毕业生的就业前景,而忽略了他们的教育成就。[A]depend on the confusion between irrefutable arguments. (出于毋庸置疑的论点之间的困惑):难就难在毋庸置疑的论点上面,vocational reasons和educational reasons都是毋庸置疑的,banking on the confusion between educational and vocational reasons=[A]。故[A]正确。
干扰项分析: give away their true intention of using computers in schools. (泄漏了他们在课堂上使用计算机的真正意图):根据末句后半部分编造的,原文是作者的分析,不是计算机教育倡导人的独白。[C] take advantage of the different reasons for computer employment. (利用了在计算机行业就业的不同理由):如果将computer employment换成education则正确。[D] make lots of profits from introducing computers into schools. (通过把计算机带入课堂盈利颇丰):钱是常见的合理项。

第八题:[D]

题眼研读:细节题。题干(The insistence that enough jobs will be available for various professionals is)中that引导的同位语从句的意思是“各种各样的专业人员将有许多工作”,根据 <3 >③[D] too arbitrary to be well-found-ed. (太主观臆断以致不能自圆其说)正确。本题实质是考presumptuous的意思,请参见第三题。
干扰项分析:[A] very bold and agitating. (傲慢的,煽动的):是presumptuous辞典上的一个意思,但这里考的是这个词在这个语境的意思,即选项[D]的意思。 very prompt and inspiring. (果断而令人鼓舞的)、[C] too deliberate to be incredible. (谨慎到难以置信的地步)错得太邪乎了!
体会:勇于并惯于向不认识的词挑战,是考研胜者必备的素质。
注释:1)on behalf of为了……的利益;2)broader reasons of radical educational reform指除了职业教育以外的更广泛的(如教育的抽象目的)对教育进行根本改革的理由;3)go to the heart of涉及到……的核心问题;4)put computers in the classroom不是指将计算机放到教室里,而是指进行计算机课堂教学;5)into their teens指受教育到十七八岁;6)the way to go原意是“应该走的道路”,这里指正确的做法;7)an entirely different story另外一回事,story在这里意为“情况”。

第七篇 (1999年Passage 5)

Science, in practice, depends far less on the experiments it prepares than on the preparedness of the minds of the men who watch the experiments. Sir Isaac Newton supposedly discovered gravity through the fall of an apple, apples had been falling in many places for centuries and thousands of people had seen them fall. But Newton for years had been curious about the cause of the orbital motion of the moon and planets. What kept them in place? Why didn’t they fall out of the sky? The fact that the apple fell down toward the earth and not up into the tree answered the question he had been asking himself about those larger fruits of the heavens, the moon and the planets.
How many men would have considered the possibility of an apple falling up into the tree? Newton did because he was not trying to predict anything. He was just wondering. His mind was ready for the unpredictable. Unpredictability is part of the essential nature of research. If you don’t have unpredictable things, you don’t have research. Scientists tend to forget this when writing their cut and dried reports for the technical journals, but history is filled with examples of it.
In talking to some scientists, particularly younger ones, you might gather the impression that they find the “scientific method” a substitute for imaginative thought. I’ve attended research conferences where a scientist has been asked what he thinks about the advisability of continuing a certain experiment. The scientist has frowned, looked at the graphs, and said “the data are still inconclusive.” “We know that,” the men from the budget office have said, “but what do you think” “Is it worthwhile going on” “What do you think we might expect?” The scientist has been shocked at having even been asked to speculate.
What this amounts to, of course, is that the scientist has become the victim of his own writings. He has put forward unquestioned claims so consistently that he not only believes them himself, but has convinced industrial and business management that they are true. If experiments are planned and carried out according to plan as faithfully as the reports in the science journals indicate, then it is perfectly logical for management to expect research to produce results measurable in dollars and cents. It is entirely reasonable for auditors to believe that scientists who know exactly where they are going and how they will get there should not be distracted by the necessity of keeping one eye on the cash register while the other eye is on the microscope. Nor, if regularity and conformity to a standard pattern are as desirable to the scientist as the writing of his papers would appear to reflect, is management to be blamed for discriminating against the “odd balls” among researchers in favor of more conventional thinkers who “work well with the team.”(480 words   可读性:57.3   难度:10.2级)

一、原题的研究
1.   The author wants to prove with the example of Isaac Newton that
[A] inquiring minds are more important than scientific experiments.
science advances when fruitful researches are conducted.
[C] scientists seldom forget the essential nature of research.
[D] unpredictability weighs less than prediction in scientific research.
2.  The author asserts that scientists
[A] shouldn’t replace “scientific method” with imaginative thought.
shouldn’t neglect to speculate on unpredictable things.
[C] should write more concise reports for technical journals.
[D] should be confident about their research findings.
3.   It seems that some young scientists
[A] have a keen interest in prediction.   often speculate on the future.
[C] think highly of creative thinking.   [D] stick to “scientific method’.
4.   The author implies that the results of scientific research
[A] may not be as profitable as they are expected.   
can be measured in dollars and cents.
[C] rely on conformity to a standard pattern.
[D] are mostly underestimated by management.

二、潜在命题点的挖掘
请参考后面的“三、原题的讲解”彻底弄懂原题后,再将文章读两遍,思考一下什么地方还可出题,出什么样的题。请继续做下面的试题:
5.  The word “supposedly” (Line 2, Para.1)suggests that Newton’s discovery of gravity through the fall of an apple is
[A] only a beautiful legend.
merely based on hypothesis.
[C] generally believed rather than proved.
[D] commonly imagined rather than testified.
6.  In paragraph 2, “cut and dried reports” refers to reports that are
[A] brief and easily comprehensible.
reduced to plain facts and data.
[C] routine in from and stale in content.
[D] prepared according to scientific criteria.
7.  The author scientific writings has
[A] been irritated at the demand for speculation.
been deprived of the essential nature of research.
[C] verified the perfect credibility of his claims.
[D] won the full trust of industrial and business management.
8.  In the eyes of business management. “odd balls”
[A] usually behave in a strange or abnormal way.
always think and work individually and irregularly.
[C] are occasional researchers who cannot work well with team.
[D] are far less profitable than researchers following accepted.

三、原题的讲解

第一题:[A]
题眼研读:例子题。本题(The author wants to prove with the example of Isaac Newton that)问作者文中用牛顿的例子来证明什么。考点是 <1 >的大意。
  ①Science, in practice, depends far less on the experiments it prepares than on the preparedness of the minds of the men who watch the experiments. ②Sir Isaac Newton supposedly discovered gravity through the fall of an apple. ③Apples had been falling in many places for centuries and thousands of people had seen them fall. ④But Newton for years had been curious about the cause of the orbital motion of the moon and planets. ⑤What kept them in place? ⑥Why didn’t they fall out of the sky? ⑦The fact that the apple fell down toward the earth and not up into the tree answered the question he had been asking himself about those larger fruits of the heavens, the moon and the planets.

  ①为观点,大意是“在实践中,科学与其说依靠事先准备的实验不如说依靠实验观察者有准备的头脑。” ②~⑦:用牛顿的例子说明观点,所以问牛顿就是问①的观点。阅读这一段可以简化成只读①和②开头的牛顿,如粗体所示,这是阅读的一个重要技巧。[A] inquiring minds are more important than scientific experiments.(喜欢探寻的大脑比实验本身更重要)是 <1 >的概括,①的改写。
干扰项分析: science advances when fruitful researches are conducted. (当进行有成就的研究时,科学就会进步):根据大科学家牛顿编造的合理项。[C] scientists seldom forget the essential nature of research. (科学家很少忘记研究的本质):来自 <2 >末句scientists tend to forget this。但加了seldom, 并且把this错误理解为牛顿的例子。原句的this代替unpredictability。再看句子的含义,也是一个合理项。[D]unpredictability weighs less than prediction in scientific research. (在科学研究中,不可预测性不如预测重要):玩的是一个正反颠倒的游戏,该选项和文意正好相反。
体会:例子题实际上是大意题,重点是抓主题句。
九阴真经解注:文章中心常考,抽象是解,比校结构是解,中心思想是解。绝对不是解,合理项不是解,和中心思想相反的不是解。例子是中心思想的体现。
第二题:

题眼研读:细节题。本题(The author asserts that scientists)问作者关于科学家的观点。题干中scientist和science是作者本篇谈论的对象,出现在各段中,所以凭借该词无法断定考点,这就需要根据四个选项判断。请注意下面的粗体标记的词汇:
[A] shouldn’t replace “scientific method” with imaginative thought.
shouldn’t neglect to speculate on unpredictable things.
[C] should write more concise reports for technical journals.
[D] should be confident about their research findings.
四个选项中关键词在文中集中出现在 <2 >和 <3 >①:
   ①How many men would have considered the possibility of an apple falling up into the tree? ②Newton did because he was not trying to predict anything. ③He was just wondering. His mind was ready for the unpredictable.④ Unpredictability is part of the essential nature of research. ⑤If you don’t have unpredictable things, you don’t have research. ⑥Scientists tend to forget this when writing their cut and dried reports for the technical journals, but history is filled with examples of it.

①In talking to some scientists, particularly younger ones, you might gather the impression that they find the “scientific method” a substitute for imaginative thought.
作者的观点是:科学研究的一个重要特征是不可预见性。词语“不可预见(unpredictable, unpredictability)”多次出现(共三次,加上predict共四次),是本段的中心词,也是谈论的中心。“不应该忽视对不可预测现象的推测”正确,注意出题者使用了否定了之否定结构增加难度。
干扰项分析:[A]“不应该用想象思维代替科学方法”:张冠李戴,批一些科学家的观点说成是作者的观点。 <3 >①说明一些科学家的观点是:所谓“科学的方法”可以取代想像思维。引号表明作者的观点与之相反,而本题问的是作者的观点。[C]“应该为科学杂志撰写更简明的报告”:来自 <2 >⑥。大意:当科学家为专业杂志撰写固定模式的报告时,他们往往忘记这一点(指不可预见性),但历史上充满了这样的例子。Cut and dried是个固定短语,意思是“按照固定模式的”。该句强调的是科学家写报告时忽略了研究课题的不可预见因素,与[C]相关太远。[C]除了使用该句的原词write reports for technical journals外,还把cut从cut and dried中分离出来,使用其同义词concise来增加欺骗性。[D]“应该对自己的研究成果有信心”:凭空编造的合理项。
体会:反复出现的关键词是文章(段)中心的体现;否定之否定是解。
九阴真经解注:反复强调处常考,三重否定是解,复杂的是解,中心思想是解,和中心思想相反不是解,合理项不是解。
第三题:[D]

题眼研读:细节题。本题(It seems that some young scientists)是第二题的翻版,不同之处在于第二题问的是作者的观点,本题问的是一些年轻科学家的观点,即作者反对的观点。[D]stick to “scientific method”.(对所谓“科学方法”深信不疑)正是这些科学家的观点,是解。
干扰项分析:[A] have a keen interest in prediction. (对预测很感兴趣)、 often speculate on the future. (经常预测未来)和[C] think highly of creative thinking. (崇尚创造性思维)是根据年轻人的一般看法编造的合理项,文章未提到。
九阴真经解注:隐蔽处常考,合理项不是解,不合理项是解。
第四题:[A]

题眼研读:推断题。从题干(The author implies that the results of scientific research)的关键词“the results”可以在末段③找到:
  ③If experiments are planned and carried out according to plan as faithfully as the reports in the science journals indicate, then it is perfectly logical for management to expect research to produce results measurable in dollars and cents. ④It is entirely reasonable for auditors to believe that scientists who know exactly where they are going and how they will get there should not be distracted by the necessity of keeping one eye on the cash register while the other eye is on the microscope. ⑤Nor, if regularity and conformity to a standard pattern are as desirable to the scientist as the writing of his papers would appear to reflect, is management to be blamed for discriminating against the “ odd balls ” among researchers in favor of more conventional thinkers who “work well with the team.”

③是个假设条件句,正话反说:“如实验完全按科学杂志报告中所陈述的那样按事先的计划去设计完成,那么管理部门可以完全合逻辑地期待研究成果用美元、美分来衡量。”意思是科学研究的结果可能并不像所预期的那样有效益,因此[A] may not be as profitable as they are expected. (可能不像预料的那样有利可图)正确。注意[A]中may含义相对,一般含义对相的选项不容易错。而且[A]反映了中心思想,即科研的不可预见性,更增加了正确的把握。
④是深达四层的复杂句。第一层为It is entirely reasonable for auditors to believe…,其中it指代for auditors to believe, auditors是不定式to believe的逻辑主语。第二层为believe的宾语从句:scientists…should not be distracted…while the other eye is on the microscope。第三层是scientists的定语从句:who know exactly…。第四层是know的两个并列的宾语从句:where they are going and how they will get there。大意:审计员们也完全有理由相信:确切知道自己的目标并知道如何实现这一目标的科学家应该没有必要分心,一只眼睛盯着现金收款机,另一只眼睛盯着显微镜。
⑤Nor, if regularity and conformity to a standard pattern are as desirable to the scientist as the writing of his papers would appear to reflect, is management to be blamed for discriminating against the “odd balls” among researchers in favor of more conventional thinkers who “work well with the team.” = if regularity and conformity to a standard pattern are as desirable to the scientist as the writing of his papers would appear to reflect, nor is management to be blamed for discriminating against the “old balls” among researchers in favor of more conventional thinkers who “work well with the team.” 大意:假如像他们的论文所反映的那样,科学家们也喜欢看到规律性和与某种标准模式的一致性,那么,如果管理者们歧视研究者中的“怪人”,而喜欢“善于合作”的具有传统思维模式的人,这也是无可指责的。
干扰项分析: can be measured in dollars and cents. (可以用美元和美分衡量):③结尾的字面意思,不是解。[C] rely on conformity to a standard pattern.(依赖于与标准模式的一致性):conformity to a standard pattern取材于⑤,但与文意相反。[D] are mostly underestimated by management. (通常被管理者低估了):也是来自⑤中nor is management to be blamed, 但和文意相并甚远。
体会:正话反说、反话正说是命题最常用的东西。
思考题:从钱的角度分析③④⑤三句,你能看出什么问题?
九阴真经解注:正话反说,取非求解,含义不肯定是解,出乎意料是解。含义肯定不是解,具体不是解。
四、新题的讲解

第五题:[C]

题眼研读:词汇题。此题(The word “supposedly” (Line 2, Para. 1) suggests that Newton’s discovery of gravity through the fall of an apple is)的实质是问supposedly的意思。此题关键是要处理好程度的问题。[A] only a beautiful legend. (仅仅是一个美丽的传说):意思过虚,尤其是加上个only。 merely based on hypothesis. (只是基于假设):是suppose的“假设,假定”字面词义,意思也过虚,尤其是有merely的存在。[D] commonly imagined rather than testified. (人们想象出来的,不能证实的):是[A]的翻版,其中imagined比较实。[C] generally believed rather than proved. (一般人通常认为,但难以证实的):虚中透出一些实,不偏不倚,符合文意,正确。
体会:中国人的中庸之道真伟大!

第六题:[C]
题眼研读:词汇题。本题(In paragraph 2, “cut and dried reports” refers to reports that are)考查cut and dried的意思。可以绝对地说,考题中出现的短语中的词如果都是简单词,结论只有一个:该短语的意思和字面意思相差甚远,一定要符合上下文语境 <2 >。
  ④Unpredictability is part of the essential nature of research. ⑤If you don’t have unpredictable things, you don’t have research. ⑥Scientists tend to forget this when writing their cut and dried reports for the technical journals, but history is filled with examples of it.

大意:一些科学家经常忽视不可预见性对于科研的重要性,所以cut and dry在文中意思必须是不可预见性的反义词。[C] routine in form and stale in content. (文章写作形式老一套,内容陈腐)意思最接近。
干扰项分析:[A] brief and easily comprehensible. (简洁,易懂):根据cut的字面意思而来,把cut理解为文字简单,简单就意味着易懂。 reduced to plain facts and data. (减少到只有明显的事实和数据):根据dried而来,把dried理解为干枯文字,没有其他的说明,只数据,也是字面意思。[D] prepared according to scientific criteria. (根据科学标准而写):取材于 <3 >①的scientific method, 但文中scientific method带具有讽刺意味的引号,所以[D]也是字面意思。
体会:字面意思不是解,注意根据语境选择词义。
cut and dried还有一个意思:已成定局的、确定的、不容更改的。如果出现类似这个意思的选项,也是对的。要把这个短语透彻理解,请参考我摘录的6本国外著名辞典的定义和解释,特别注意美语和英语的不同解释,说明该词在美语和英语中用法略有差异。
1. 美国出版的辞典的定义:
cut and dried also cut-and-dry (adj.) (1710): being or done according to a plan, set procedure, or formula: routine
  ——from Webster’s Ninth New Collegiate Dictionary (1710)表示该词最早出现的年代。
Cut-and-dried also cut-and-dry(adj.)(1) settled in advance; not needing much discussion; clear-cut: This issue seems fairly cut-and dried; let’s vote on it now. (2) routine: cut-and-dried operations.
  ——from Random House Webster’s Dictionary of American English
cut-and-dried (adj.) routine; boring; not requiring much thought or effort. Your essay seems totally cut-and –dried and will not get a very high grade. / This is a simple question that requires only a cut-and-dried answer.
  ——from NTC’s American English Learner’s Dictionary
2. 英国出版的辞典的解释:
cut-and-dried, (adj.) If something is cut-and-dried, it is decided and unlikely to be changed: We need a cut-and-dried decision by the end of the week.
A cut-and-dried solution to a problem is clear and simple, and removes any further difficulty or confusion: There is no cut-and-dried answer to this problem.
  ——from Cambridge International Dictionary of English
cut and dried (adj.) a decision or result that is cut and dried cannot be changed: I think we can say that the result of the election is now cut and dried.
  ——from Longman Dictionary of Contemporary English
cut-and-dried. A cut-and-dried answer or solution is clear and obvious. There is no cut-and-dried formula which can answer these questions.
  ——from Collins Cobuild Essential English Dictionary

第七题:

题眼研读:例子题。题干(The author of scientific writings has)中the最关键,那位作者,显然不是本文作者,因为本文作者一般用the author of this passage。文章提到的具体某一个人在 <3 >,其中有人物对白,非常生动形象。可在 <4 >也提到了该科学家,也是阅读内容。考什么?对例子的出题方式通常是这个例子说明了什么,作者举例的目的何在。其实很简单,例子大部分是说明前文提到的中心。本文中 <2 >的大意是:科研的本质部分是不可预见性。例子有正面的,有反面的。 <3 >的例子是反例,由于他的研究没有不可预见性,说明been deprived of the essential nature of research. (那位作者也就丧失了科研的本性)正确。本题考查面很宽,涉及2~4共三段。
干扰项分析:[A] been irritated at the demand for speculation. (被推测的要求激怒了): <3 >④被偷梁换柱了,把shocked换成了irritated。再说即使是换成shocked也是表面意思,不可能是解。[C] verified the perfect credibility of his claims. (证实了他的观点完全可靠):claim取材于 <4 >②,但和文意相反。尤其是有perfect这么绝对的词汇就更不对了。[D] won the full trust of industrial and business management. (赢得了实业界和企业管理人员的完全信任):是对 <4 >②的绝对化理解。原文说这位科学家使实业界和企业管理人员相信他的观点是正确的。但不能推断是“完全”信任,full过于绝对。

第八题:[D]

题眼研读:细节题。本题(In the eyes of business management, “odd balls”)的解正是第四题思考题的答案。[D] are far less profitable than researchers following accepted. (那些比下面提到的研究者们带来的利润少得多的人)正确。大家注意观察“钱”是怎样跨过三句,爬山涉及水,和不合群的odd balls (怪人)联系在一起的。忽然想起看看考研大纲,大纲对阅读要求的第六条说:“考生应能理解文章的总体结构公及单句之间、段落之间的关系”,这就是考研。本题从形象上看是细节题或者是词汇题,但实际上是推理题。这就验证了我们前面的观点,考研题都是推理题,从形式上划分题型是错误的。
干扰项分析:“odd ball”常见意思是怪人,在本文中指不合传统思维的人。[A] usually behave in a strange or abnormal way. (通常行为举止古怪或变态)和 always think and work individually and irregularly. (思维和工作时,总是一个人干且没有规律)及[C] are occasional researchers who cannot work well with team. (是临时研究人员,不能和他人一起工作)都是它的普通意思,或者字面意思,不是解。
体会:蔑视局部,追求整体是考研思维应遵循的准则。
注释:1)depend far less on…与其说依靠……,不如说依靠……;2)the orbit motion of the moon and the planets月球和行星沿轨道的运行;3)cut and dried 早已准备好的,刻板的;4) budget office 预算部门;5)management此处指“管理者”。

第八篇(2000年Passage 5)

If ambition is to be well regarded, the rewards of ambition — wealth, distinction, control over one’s destiny — must be deemed worthy of the sacrifices made on ambition’s behalf. If the tradition of ambition is to have vitality, it must be widely shared; and it especially must be highly regarded by people who are themselves admired, the educated not least among them. In an odd way, however, it is the educated who have claimed to have given up an ambition as an ideal. What is odd is that they have perhaps most benefited from ambition — if not always their own then that of their parents and grandparents. There is a heavy note of hypocrisy in this, a case of closing the barn door after the horses have escaped — with the educated themselves riding on them.
Certainly people don’t seem less interested in success and its signs now than formerly. Summer homer, European travel, BMWs — the locations, place names and name brands may change, but such items don’t seem less in demand today than a decade or two years ago. What has happened is that people can not confess fully to their dreams, as easily and openly as once they could, lest they be thought pushing, acquisitive and vulgar, instead, we are treated to fine hypocritical spectacles, which now more than ever seem in ample supply: the critic of American materialism with a Southampton summer home; the publisher of radical books who takes his meals in three-star restaurants; the journalist advocating participatory democracy in all phases of life, whose own children are enrolled in private schools. For such people and many more perhaps not so exceptional, the proper formulation is, “Succeed at all costs but avoid appearing ambitious.”
The attacks on ambition are many and come from various angles; its public defenders are few and unimpressive, where they are not extremely unattractive. As a result, the support for ambition as a healthy impulse, a quality to be admired and fixed in the mind of the young, is probably lower than it has ever been in the United States. This doesn’t mean that ambition is at an end, that people no longer feel its stirrings and promptings, but only that, no longer openly honored, it is less openly professed. Consequences follow from this, of course, some of which are that ambition is driven underground, or made sly. Such, then ,is the way things stand: on the left angry critics, on the right stupid supporters, and in the middle, as usual, the majority of earnest people trying to get on in life. (429 words  可读性:51.3  难度:11.7级)

一、原题的研究
1. It is generally believed that ambition may be well regarded if
[A] its returns well compensate for the sacrifices.
it is rewarded with money, fame and power.
[C] its goals are spiritual rather than material.
[D] it is shared by the rich and the famous.
2. The last sentence of the first paragraph most probably implies that it is
[A] customary of the educated to discard ambition in words.
too late to check ambition once it has been let out.
[C] dishonest to deny ambition after the fulfillment of the goal.
[D] impractical for the educated to enjoy benefits from ambition.
3. Some people don’t openly admit they have ambition because
[A] they think of it as immoral.
their pursuits are not fame or wealth.
[C] ambition is not closely related to material benefits.
[D] they don’t want to appear greedy and contemptible.
4. From the last paragraph the conclusion can be drawn that ambition should be maintained
[A] secretly and vigorously.
openly and enthusiastically.
[C] easily and momentarily.
[D] verbally and spiritually.

二、潜在命题点的挖掘

请参考后面的“三、原题的讲解”彻底弄懂原题后,再将文章读两遍,思考一下什么地方还可出题,出什么样的题。请继续做下面的试题:
5. In reality, educated people have
[A] denied the vitality of ambition with their sincere claims.
constituted the minority among those who regard ambition highly.
[C] substituted their high ideals for traditional ambitions.
[D] profited most by their own or, as often as not, their parental ambitions.
6. The fact of continuous demands for luxuries is used to prove
[A] their lasting and strong attraction for ordinary people.
their being remarkable symbols of personal success in life.
[C] the unfailing popularity of ambition among social members.
[D] the pushing and acquisitive qualities of being ambitious.
7. The examples of the critic ,the publisher, and the journalist are intended to show that
[A] many people are not ambitious in word while they are in deed.
ambition is not a mere presentation of beautiful scenes.
[C] obstacles to being ambitious are in unprecedented multitude.
[D] seeking for fortune and comfort is natural and normal to humans.
8. In the U.S. society, open defenders of ambition are few because
[A] they are much less impressive and attractive than its attackers.
most people pursue their ambitions privately to avoid seeming vulgar.
[C] ambition has ceased to have its impulsive and exciting properties.
[D] they are deemed to be hypocritical rather than honorable.

三、原题的讲解

第一题:[A]

题眼研读:细节题。题干(It is generally believed that ambition may be will regarded if)中that从句和 <1 >①几乎是一样的:
  If ambition is to be well regarded, the rewards of ambition — wealth, distinction, control over one’s destiny — must be deemed worth of the sacrifices made on ambition’s behalf.

考生容易发现原句和题干的意思是一致的,但是语序正好相反。原句的if条件句变成了题干的主句,原句的主句是四个选项,答案必是对主句的正确理解。大意:如果雄心壮志受人羡慕,那么,它所带来的回报——财富,荣誉,对命运的主宰——定被认为是值得为之做出牺牲的。本句强调了付出与回报相匹配的重要性,所以[A] its returns well compensate for the sacrifices. (它所带来的回报能很好地补偿为之做出的牺牲)正确。本题采用了“同义替换+结构转换”的设计方法。比较:

干扰项分析: it is rewarded with money, fame and power. (它以金钱,名誉和权力作为回报):词汇来自原文词汇的同义词rewarded是rewards的动词,money(wealth), fame(distinction), power(control), 该选项的迷惑性最强。文章说的是回报与牺牲两者相匹配,但谈到了回报,却没谈匹配的问题,所以错。[C] its goals are spiritual rather than material. (它的目的是精神的,而非物质的):来自对本句sacrifices(牺牲)的错误理解。[D] it is shared by the rich and the famous. (富人及名人具有雄心壮志):来自对“wealth, distinction”的错误理解。
体会:矛盾(对立统一)是解。
九阴真经解注:段落句常考,倒着考,相互作用是解。
第二题:[C]
题眼研读:例子题。本题(The last sentence of the first paragraph most probably implies that it is)表面上问的是 <1 >末句的含义,但这句话实际上是文章中用以说明问题的例子,所以从根本上讲是例子型题。
  ② If the tradition of ambition is to have vitality, it must be widely shared; and it especially must be highly regarded by people who are themselves admired, the educated not least among them. ③ In an odd way, however, it is the educated who have claimed to have given up an ambition as an ideal. ④What is odd is that they have perhaps most benefited from ambition — if not always their own then that of their parents and grandparents. ⑤ (a) There is a heavy note of hypocrisy in this, (b)a case of closing the barn door after the horses have escaped — with the educated themselves riding on them.

  例子是为观点服务。②~⑤(a)是观点。大意是:“雄心壮志的传统若能激发活力,那么,人们就应当广泛地拥有这种传统;尤其那些自身受人羡慕之士更应给予高度重视。当然,受过良好教育的人也尤应包括在内。然而,有点奇怪的是,宣称已经放弃理想抱负的人却是受教育的人。真正奇怪的是,受过教育的人也许就是抱负的最大受益者——如果抱负不是自己的便是父辈和祖辈给的。” ⑤(b)是说明观点的例子。大意是“马逃走后就关上马厩的门,骑在马上的正是这些受过教育的人。”这里,把得到马比作理想,在英语和汉语里马都有“成功”的意思,把马逃走比作理想的实现,把骑在马上经作正在享受成功。马已经跑了再关上马厩的门,肯定来不及了,所以说这种做法是虚伪的。[C] dishonest to deny ambition after the fulfillment of the goal.(达到目的后对雄心壮志的否认是不诚实的。)正确。
干扰项分析:[A] customary of the educated to discard ambition in words. (受过良好教育的人习惯口头摒弃雄心壮志是符合惯例的):不合文意,是编造的合理项。 too late to check ambition once it has been let out. (雄心壮志一旦放弃便难以制止):是对“马”这个比喻的字面上的理解,将escape曲解为let out,将close the door曲解为too late to check ambition. [D] impractical for the educated to enjoy benefits from ambition. (受过良好教育的人享受雄心壮志带来的好处是现实的):和文意相反。
体会:例子是观点和中心的体现。
九阴真经解注:替代是解,中心思想是解,矛盾是解,复杂性是解。
第三题:[D]

题眼研读:细节题。本题(Some people don’t openly admit they have ambition because)问一些人不公开承认他们有雄心壮志的原因。题干的关键词don’t openly admit出现在 <2 >③,其中admit和confess to同义。
   ①Certainly people don’t seem less interested in success and its signs now than formerly. ②Summer homes, European travel, BMWs — the locations, place names and name brands may change, but such items don’t seem less in demand today than a decade or two years ago. ③What has happened is that people can not confess fully to their dreams, as easily and openly as once they could, lest they be thought pushing, acquisitive and vulgar.

lest意思是“惟恐”,它引导的从句中动词应用原形。大意:人们不像以前那样轻松地、公开地彻底承认自己有梦想,是因为害怕自己被认为爱出风头、贪婪、庸俗。可见[D] they don’t want to appear greedy and contemptible. (他们不想显露出贪婪和可鄙)正确。比较:

干扰项分析:[A] They think of it as immoral. (他们认为雄心壮志不道德):程度太过,而且明显违反了基本的常识。 Their pursuits are not fame or wealth. (他们追求的既不是名誉也不是财富):将 <2 >①的双重否定理解成一重否定,与文意相反。[C] Ambition is not closely related to material benefits. (雄心壮志与物质利益没有紧密联系): 将 <2 >②的双重否定理解成一重否定,与文意相反。
体会:带有偏难词的选项是解。
九阴真经解注:隐蔽处(状语从句)有解,表面与事实相反是解,矛盾是解。
第四题:
题眼研读:有关中心思想、写作目的和作者态度的推理题。本题(From the last paragraph the conclusion can be drawn that ambition should be maintained)明示考末段。
   The attacks on ambition are many and come from various angles; its public defenders are few and unimpressive, where they are not extremely unattractive. As a result, the support for ambition as a healthy impulse, a quality to be admired and fixed in the mind of the young, is probably lower than it has ever been in the United States. This doesn’t mean that ambition is at an end, that people no longer feel its stirrings and promptings, but only that, no longer openly honored, it is less openly professed. Consequences follow from this ,of course, some of which are that ambition is driven underground, or made sly. Such, then, is the way things stand: on the left angry critics, on the right stupid supporters, and in the middle, as usual, the majority of earnest people trying to get on in life.

前面说过一篇文章的中心思想通常出现在首段或末段,本篇属于后者。本段大意:由于受到攻击,雄心壮志隐藏起来,就如同[A] secretly and vigorously. (秘密地而有活力地)所描述的那样,但作者认为审不对的。特别要注意题干中关键词“should”,表示问的是“应该”,而不是目前的情况。所以判断openly and enthusiastically.(公开而热情地)是解。该段多外出现了选项中 openly的反义词语,如less openly, no longer openly, underground, sly, 大家需要认真体会反着考的威力了。
干扰项分析:[A] secretly and vigorously:不少考生因没有看清楚问题误选[A]。[C] easily and momentarily.(轻松地、暂时地):意思和文章相差甚远。如果你选了这个选项,你的水平有待提高。[D] verbally and spiritually(口头上和精神上地):断章取义于③中“at an end”(穷途末路),认为ambition已经不复存在了,但去掉了前面的否定结构,导致和原文相反。
体会:审题很关键,如果你养成提炼关键词的习惯,会避免或少丢的分(主要针对文章看懂了,但题没答对的情况)。
九阴真经解注:段末常考,简单不是解,肤浅不是解。并关注本问题问的是should be。
四、新题的讲解

第五题:[D]

题眼研读:细节题。题干(In reality, educated people have)关键词educated people在 <1 >②③④出现:
  ②If the tradition of ambition is to have vitality, it must be widely shared; and it especially must be highly regarded by people who are themselves admired, the educated not least among them. ③In an odd way, however it is the educated who have claimed to have given up an ambition as an ideal. ④What is odd is that they have perhaps most benefited from ambition — if not always their own then that of their parents and grandparents.

④“真正奇怪的是,受过教育的人也许就是抱负的最大受益者——如果抱负不是自己的便是父辈祖辈给的”。选项[D] profited most by their own or, as often as not ,their parental ambitions. (通过自己本身的抱负获益比从父母的抱负中获益更普遍),正确,考了句中的“隐蔽之处”:破折号后。
干扰项分析:[A] denied the vitality of ambition with their sincere claims. (真切地否认立志的生命力):断章取义于②③中“ambition, vitality, claim”与文章相反。 constituted the minority among those who regard ambition highly. (在赞赏抱负的人中占少数):拼凑②中“regard, ambition, highly”,并且根据对“not least among them”的错误理解编造出“constituted the minority”。注意:“not least”的意思是“尤其”,不是“不是最少”。[C]substituted their high ideals for traditional ambitions. (用传统的抱负代替了崇高理想):由②中“tradition of ambition, highly”个别是词汇拼凑而成,而原文说传统的抱负依然有生命力。
体会:第二题和本题都考第一段。但第二题是例子题,反映的是段中心。本题是细节题,考的是细枝末节,与中心没有直接关系。需要指出的是考研相当一部分细节题是反映中心的。

第六题:[C]

题眼研读:例子题。题干(The fact of continuous demands for luxuries is used to prove)中关键词continuous demands for luxuries(人们对奢侈品的不断要求)来自 <2 >,该段提到了诸如summer homes, European travel, BMWs等奢侈品:
  ①Certainly people don’t seem less interested in success and its signs now than formerly. ②Summer homes, European travel, BMWs — the locations, place names and name brands may change, but such items don’t seem less in demand today than a decade or two years ago. ③What has happened is that people can not confess fully to their dreams, as easily and openly as once they could, lest they be thought pushing, acquisitive and vulgar. ④Instead, we are treated to fine hypocritical spectacles, which now more than ever seem in ample supply: the critic of American materialism with a Southampton summer home; the Publisher of radical books who takes his meals in three star restaurants; the journalist advocating participatory democracy in all phases of life, whose own children are enrolled in private schools. ⑤For such people and many more perhaps not so exceptional, the proper formulation is , “ Succeed at all costs but avoid appearing ambitious.”

①②:人们对成功所带来的物质的追求和以前一样强烈。③④:现实是人们将雄心壮志隐藏起来。⑤是段主题“不择一切手段成功,但却避免让人看上去太野心勃勃。”[C] the unfailing popularity of ambition among social members. (社会成员对抱负和理想不衰的热情)是②的一部分,反映了段中心的前半部分,在没有更好的完整中心的情况下正确,一定要仔细研究完其他选项后再作决断。
干扰项分析:[A] their lasting and strong attraction for ordinary people. (对老百姓永久强烈的吸引力)和 their being remarkable symbols of personal success in life. (他们是个人成功的典型代表)及[D] the pushing and acquisitive qualities of being ambitious. (带有抱负、爱出风头以及贪婪的属性)都是根据常人观点编造的合理项。
体会:例子是中心思想的体现。

第七题:[A]

题眼研读:例子题。本题(The examples of the critic, the publisher, and the journalist are intended to show that)考的是 <2 >另一个例子的含义,与第六题相似。[A] many people are not ambitious in word while they are in deed. (很多人口头上不说自己雄心勃勃,但是行动上却如此)反映了段中心的全部,是解。(第六题反映的是段中心的一半。)
干扰项分析: ambition is not a mere presentation of beautiful scenes. (抱负给我们呈现的不仅仅是美丽的画面):是根据④中spectacles(景观)断章取义的结果。[C] obstacles to being ambitious are in unprecedented multitude. (有雄心面临着前所未有的大量困难):是根据④中in ample supply断章取义而来。[D] seeking for fortune and comfort is natural and normal to humans. (追求幸福和安逸是人们的天性):是合理项。
体会:再说一遍,例子是中心思想的体现。

第八题:

题眼研读:细节题。题干(In the U.S. society, open defenders of ambition are few because)的open defenders of ambition are few来自末段①:
①The attacks on ambition are many and come from various angles; its public defenders are few and unimpressive, where they are not extremely unattractive. ②As a result, the support for ambition as a healthy impulse. A quality to be admired and fixed in the mind of the young, is probably lower than it has ever been in the United States. ③This doesn’t mean that ambition is at an end, that people no longer feel its stirrings and promptings, but only that, no longer openly honored, it is less openly professed. ④Consequences follow from this, of course, some of which are that ambition is driven underground, or made sly. ⑤Such, then, is the way things stand: on the left angry critics, on the right stupid supporters, and in the middle, as usual, the majority of earnest people trying to get on in life.

①是原因“对抱负的攻击多,公开的支持者少”。②~⑤是①的结果“抱负隐蔽起来”。如果将题干中的because换成result in that则本题没有任何难度, Most people pursue their ambitions privately to avoid seeming vulgar. (大多数人默默地追逐着自己的理想,以避免看起来庸俗)正确。但题干是because,难度就比较大,给人一种因果倒置的感觉。但在确定其他选项都不正确的前题下只能选。共正确性可用互为因果关系来解释,更重要的原因是,是段中心的总结。段大意分析参见第四题。
干扰项分析:[A] They are much less impressive and attractive than its attackers. (他们比攻击者给人的印象和吸引力都差多了):①中where引导的从句不是常见的地点状语从句,特点为具有让步状语功能,意思是“虽然并非特别的没有吸引力”。也就是说有一定的吸引力,这样[A]中much less语气就显得太重了。[C] Ambition has ceased to have its impulsive and exciting properties. (雄心已经失去了其令人冲动和兴奋的特征了):取材于②中impulse和③中no longer feel, stirring, 去掉了原文的否定词,造成意思和原文相反。[D]They are deemed to 中no longer openly honored,但省略了openly,将主语换成“抱负的公开支持者”,造成意思的改变。
体会:断章取义、张冠李戴,命题者习惯于斯。
注释:1)deem 认为;2)worthy of 值得;3)on sb’s behalf 为了/代表某人;4)not least 尤其是;5)confess to承认;6)treat sb. to款待,宴请(某人);7)in ample supply充足地供应;8) at all costs不惜一切代价

第九篇(2001年Passage 1)

Specialisation can be seen as a response to the problem of an increasing accumulation of scientific knowledge. By splitting up the subject matter into smaller units, one man could continue to handle the information and use it as the basis for further research. But specialization was only one of a series of related developments in science affecting the process of communication. Another was the growing professionalisation of scientific activity.
No clear-cut distinction can be drawn between professionals and amateurs in science: exceptions can be found to any rule. Nevertheless, the word “amateur” does carry a connotation that the person concerned is not fully integrated into the scientific community and, in particular, may not fully share its values. The growth of specialization in the nineteenth century, with its consequent requirement of a longer, more complex training, implied greater problems for amateur participation in science. The trend was naturally most obvious in those areas of science based especially on a mathematical or laboratory training, and can be illustrated in terms of the development of geology in the United Kingdom.
A comparison of British geological publications over the last century and a half reveals not simply an increasing emphasis on the primacy of research, but also a changing definition of what constitutes an acceptable research paper, Thus ,in the nineteenth century ,local geological studies represented worthwhile research in their own right; but , in the twentieth century, local studies have increasingly become acceptable to professionals only if they incorporate, and reflect on, the wider geological picture. Amateurs, on the other hand, have continued to pursue local studies in the old way. The overall result has been to make entrance to professional geological journals harder for amateurs, a result that has been reinforced by the widespread introduction of refereeing, first by national journals in the nineteenth century and then by several local geological journals in the twentieth century. As a logical consequence of this development, separate journals have now appeared aimed mainly towards either professional or amateur readership. A rather similar process of differentiation has led to professional geologists coming together nationally within one or two specific societies, whereas the amateurs have tended either to remain in local societies or to come together nationally in a different way.
Although the process of professionalisation and specialization was already well under way in British geology during the nineteenth century, its full consequences were thus delayed until the twentieth century. In science generally, however, the nineteenth century must be reckoned as the crucial period for this change in the structure of science. (424 words   可读性:19.3  难度:12级)

一、原题的研究
1. The growth of specialization in the 19th century might be more clearly seen in sciences such as
[A] sociology and chemistry.       physics and psychology.
[C] sociology and psychology.       [D] physics and chemistry.
2. We can infer from the passage that
[A] there is little distinction between specialization and professionalisation.
amateurs can compete with professionals in some areas of science.
[C] professionals tend to welcome amateurs into the scientific community.
[D] amateurs have national academic societies but no local ones.
3. The author writes of the development of geology to demonstrate
[A] the process of specialization and professionalisation.
the hardship of amateurs in scientific study.
[C] the change of policies in scientific publications.
[D] the discrimination of professionals against amateurs.
4. The direct reason for specialization is
[A] the development in communication.
the growth of professionalisation.
[C] the expansion of scientific knowledge.
[D] the splitting up of academic societies.

二、潜在命题点的挖掘

请参考后面的“三、原题的讲解”彻底弄懂原题后,再将文章读两遍,思考一下什么地方还可出题,出什么样的题。请继续做下面的试题:
5. The formation of specialization and professionalisation can be thought of as
[A] The result of further division of accumulating knowledge.
the evidence of the incapacity of individual scientists for research.
[C] the product of information explosion in respect of scientific growth.
[D] the consequence of the elaborate classification of scientific activities
6. What distinguishes amateurs from professionals in science may be that
[A] the former are partially involved in scientific research.
the latter are fully predominating over the scientific community.
[C] the former have got insufficient education needed for scientific work.
[D] the latter have benefited a lot from mathematical training.
7. In the 20th century, amateur’s studies have become acceptable to professionals on condition that they
[A] are carried out in cooperation with professionals.
embody comprehensive accounts of geological events.
[C] embrace a larger variety of geological pictures.
[D] pass the examination by national and local journals.
8. It is true of the 20th century that it
[A] appeared to be a critical time in geological history.
was regarded as the turning point in the advancement of science.
[C] signified the great progress made in geological studies in Britain.
[D] witnessed all the outcomes of scientific specialization.

三、原题的讲解

第一题:[D]

题眼研读:推理题。本题(The growth of specialization in the 19th century might be more clearly seen in sciences such as)问“19世纪的专业化趋势在下列哪些学科领域表现得更明显?”主语和 <2 >③的主语完全相同。
③The growth of specialization in the nineteenth century, with its consequent requirement of a longer, more complex training, implied greater problems for amateur participation in science. ④The trend was naturally most obvious in those areas of science based especially on a mathematical or laboratory training, and can be illustrated in terms of the development of geology in the United Kingdom.

④“这咱趋势明显表现在需要数学或实验训练的科学领域。”考生需要判断出哪个选项中的学科是符合数学或实验性特征的,有些GRE的人把这种题称为是应用题。显然[D] physics and chemistry. (物理学和化学)正确。
干扰项分析:[A] sociology and chemistry. (社会学和化学):错在社会学。physics and psychology.(物理学和心理学):  错在心理学。[C] sociology and psychology. (社会学和心理学):都错。
九阴真经解注:中心思想是解。例子是中心思想的体现。
第二题:

题眼研读:推理题。题干(We can infer from the passage that)什么都没说,无非是问哪个正确的。浏览选项:
[A] there is little distinction between specialization and professionalisation.
amateurs can compete with professionals in some areas of science.
[C] professionals tend to welcome amateurs into the scientific community.
[D] amateurs have national academic societies but no local ones.
[A]“专业化和职业化之间几乎没有差别”文章 <1 >末句谈论两者之间的关系:
But specialization was only one of a series of related developments in science affecting the process of communication, Another was the growing professionalisation of scientific activity.
从此句看出specialisation和professionalisation是两个不同的过程。所以[A]错。、[C]和[D]在谈论专业人士和业余爱好者之间的关系,定位是在 <3 >。
①A comparison of British geological publications over the last century and a half reveals not simply an increasing emphasis on the primacy of research, but also a changing definition of what constitutes an acceptable research paper. ②Thus, in the nineteenth century, local geological studies represented worthwhile research in their own right; but, in the twentieth century, local studies have increasingly become acceptable to professionals only if they incorporate, and reflect on, the wider geological picture. ③Amateurs, on the other hand, have continued to pursue local studies in the old way. ④The overall result has been to make entrance to professional geological journals harder for amateurs, a result that has been reinforced by the widespread introduction of refereeing, first by national journals in the nineteenth century and then by several local geological journals in the twentieth century. ⑤As a logical consequence of this development, separate journals have now appeared aimed mainly towards either professional or amateur readership. ⑥A rather similar process of differentiation has led to professional geologists coming together nationally within one or two specific societies, whereas the amateurs have tended either to remain in local societies or to come together nationally in a different way.

整段都在讲专业和业余人员之间的不相容,所以很少有人选[C]“专业人士比较欢迎业余爱好者进入科学界”。⑤⑥“这导致一个自然的结果是:不同的学刊面向不同的主体读者群,要么是专业人员,要么是业余人员。地质科研人员中也发生着相似的分化现象,这使得全国专业地质人员逐渐形成了一两个特定的科研协会,而业余科研人员往往要么呆在地方协会中,要么组成一个不同的全国性的协会。”说明[D]“业余人员有全国的学会,但没有地方学会”错,这很简单,关键说明“业余人员在某些学科领域可与专业人员相抗衡”正确,对很多人来说都是很难的,仔细想想什么叫考研英语的弦外之音。
体会:如果几个选英都在一段,比较好做,但如果分散于两段或两段以上,做一道题有的时候相当于读一篇文章,难!
九阴真经解注:SOME是解。
第三题:[A]

题眼研读:例子题。题干(The author writes of the development of geology to demonstrate)问作者描述地质学发展过程的目的是什么。这个例子从 <2 >末开始贯穿整个 <3 >。实际上是通过例子测试主题。 <3 >在全文的段落中最长,但不是文章主题句之所在,如前所述,文章的主题句一般是首末两段,所以应把阅读的重点放在这两段。 <1 >的后半部分的but引出了文章的topic: specialization and professionalisation; 末段围绕着两者的过程和变化。可见[A] the process of specialization and professionalisation. (专业化及职业化的发展过程)是本文的主题,是解。
干扰项分析: the hardship of amateurs in scientific study. (业余爱好者在科研中的艰难):非常有诱惑性,可从 <2 >、 <3 >中体现出来,但仅仅反映了主题的一个侧面,犯了“以偏概全,主次不分”的毛病。[C] the change of policies in scientific publications. (科学出版政策的变化):以偏概全,是 <3 >中一个细节内容,不是主题。[D] the discrimination of professionals against amateurs. (专业人士对业余人员的歧视):又具体又极端,不是解。
体会:再说一遍,例子是中心思想的体现。
九阴真经解注:概括性是解,中心思想是解。具体的不是解。

第四题:[C]

题眼研读:细节题。本题(The direct reason for specialization is)问导致专业化的直接原因是什么。文章论述specialisation的地方很多,所以要抓住direct reason这个关键词, <1 >①response to与之相呼应。
  ①Specialisation can be seen as a response to the problem of scientific knowledge. ②By splitting up the subject matter into smaller units, one man could continue to handle the information and use it as the basis for further research. ③But specialization was only one of a series of related developments in science affecting the process of communication. ④Another was the growing professionalisation of scientific activity.

①“科学知识的日积月累是专业化的反应”,故[C] the expansion of scientific knowledge. 正确,注意词语的同义替换:direct reason=response to; expansion=accumulation。
干扰项分析:[A] the development in communication. (通讯的发展):取材于③。①“知识的积累导致专业化”;③“专业化影响了交流的过程。”这是在混淆是非。 the growth of professionalisation(职业化的发展):取材于④the growing of professionalisation, 本段在叙述专业化和职业化时强调两者是同时进行的,彼此没有因果关系。[D] the splitting up of academic societies. (科学协会的分裂):注意在 <3 >末句中提到的[D],是作为结果出现的,不是直接原因。
体会:思维一定要清楚。
九阴真经解注:段落句常考,因果关系常考。

四、新题的讲解
第五题:[C]

题眼研读:大意题。本题(The formation of specialization and professionalisation can be thought of as)是第四题的翻版,be thought of as意思是“被认为是”考点仍在 <1 >。浏览四个选项前两个字,发现问的原因。注意 <1 >中逻辑:①“科学知识积累导致专业化”, ②“专业化等于或大体等于把学科内容细分成小的单位”。[C] the product of information explosion in respect of scientific growth. (科学的发展导致信息爆炸的产生)是专业分工的原因,正确。
干扰项分析:[A] the result of further division of accumulating knowledge. (不断增加的知识的分离的结果):further division of accumulating knowledge=专业化,如果[A]对,论证的逻辑就相当于专业化是专业化的结果。[D] the consequence of the elaborate classification of scientific activities. (科学活动详尽分类的结果):根据  <1 >末句词汇scientific activities以及②“研究课题的细分”编造的。如果将classification of scientific activities等同于splitting up the subject matter into smaller units,则本题的逻辑又是相当于[A]的逻辑“专业化是专业化的结果”。the evidence of the incapacity of individual scientists for research. (单个科学家不能搞研究这一事实):凭空编造的合理项。
体会:无论多么复杂的东西,深入分析,无非都是1+1=2。

第六题:[A]

题眼研读:细节题。本题(What distinguishes amateurs from professionals in science may be that)问科学上业余和专业人士的区别是什么?考 <2 >①②。
①No clear-cut distinction can be drawn between professionals and amateurs in science: exceptions can be found to any rule. ②Nevertheless, the word “amateur” does carry a connotation that the person concerned is not fully integrated into the scientific community and, in particular, may not fully share its values.

②:“业余”意味着不能充分地融入科学界,尤其是不能完全共享其价值观。可见[A] The former are partially involved in scientific research. (前者只是部分地从事科学研究)符合题意。注意同义替换:not fully = partially; integrated into = involved in。
干扰项分析: The latter are fully predominating over the scientific community. (后者在科学完全处于主导地位):错就错在fully这个词上,程度过于绝对。[C] The former have got insufficient education needed for scientific work. (对于科研工作,前者受到的教育不足):是根据一般常识编出的合理项,原文没有提到。[D]The latter have benefited a lot from mathematical training. (后者从数学训练中获益很多):根据 <2 >末句的mathematical training编成,推理稍嫌过火,按我现在的智力水平,我只能这么理解,如果不是因为[A]是正确答案,[D]就最佳。

第七题:

题眼研读:细节题。本题(In the 20th century, amateurs’ studies have become acceptable to professionals on condition that they)问在20世纪,业余科学者的研究成果想要被专业人士接纳,这些研究必须具备的条件是什么。题干的用词发现和 <3 >②后半部分相似,本题实际是考查对这句话的理解。
   Thus, in the nineteenth century, local geological studies represented worthwhile research in their own right; but, in the twentieth century, local studies have increasingly become acceptable to professionals only if they incorporate, and reflect on, the wider geological picture.

  incorporate意思是“体现”;the wider picture意思是“较大范围的”,不是“图画”。大意是“……但到了二十世纪,只有当这种局部研究并思考了更大范围的地质学问题时,这种局部性的地质学研究才能为专业人员所接受。”所以 embody comprehensive accounts of geological events. (包含了对地质学方面广泛的研究)正确。比较:

干扰项分析:[A] are carried out in cooperation with professionals. (与专业人员合作进行):把incorporate错误理解为in cooperation with后编造的信息。[C] embrace a larger variety of geological pictures. (包含大量地质图片):如果你选[C],把picture老老实实地理解为“图片”,那你就太过于纯朴了。[D] pass the examination by national and local journals. (通过全国和地方杂志的审查):凭空编造的合理项。
体会:不要作纯朴和天真的牺牲品。

第八题:[D]

题眼研读:题干(It is true of the 20th century that it)的关键词是the 20th century。本文两处涉及这个时间,一个在 <3 >②,另一个在末段①。浏览选项,发现考点在末段。
   Although the process of professionalisation and specialization was already well under way in British geology during the nineteenth century, its full consequences were thus delayed until the twentieth century. In science generally, however, the nineteenth century must be reckoned as the crucial period for this change in the structure of science.

大意:尽管职业化、专业化在19世纪的英国地质进行得相当深入,然而其充分的影响推迟美观20世纪才体现出来。但是,就总体科学研究而言,19世纪应被视为科研结构发生变化的关键时代。简洁地说,尽管专业化的充分发燕尾服是在20世纪,但19世纪是专业化的关键时期。[D] witnessed all the outcomes of scientific specialization. (目睹了科学的专业化的所有成果)是答案。注意同义替换,all the outcomes = full consequences。
干扰项分析:[A] appeared to be a critical time in geological history. (似乎是地质学历史上一个关键时期)和 was regarded as the turning point in the advancement of science. (被认为是科学发展的转折点)和[C] signified the great progress made in geological studies in Britain. (标志着英国在地质学研究上的巨大发展)都说的是19世纪,不是20世纪。
体会:本题对于“含义相对是解,含义绝对不是解”这一答题技巧而言,可以说是一个灾难,但我们要看到,要想让含义绝对的all正确,需要有文章中full对应,出题者出这样的题,要冒很大的风险。所以从多年的实践来看这是少数情况。
注释:1) in one’s own right 靠自己条件;2) reflect on 思考;3) under way 在进行之中。

第十篇(2001年Passage 5)

When I decided to quit my full time employment it never occurred to me that I might become a part of a new international trend. A lateral move that hurt my pride and blocked my professional progress prompted me to abandon my relatively high profile career although, in the manner of a disgraced government minister, I covered my exit by claiming “I wanted to spend more time with my family”.
Curiously, some two-and –a –half years and two novels later, my experiment in shat the Americans term “ downshifting” has turned my tired excuse into an absolute reality. I have been transformed from a passionate advocate of the philosophy of “having it all”, preached by Linda Kelsey for the past seven years in the pages of She magazine, into a woman who is happy to settle for a bit of everything.
I have discovered. As perhaps Kelsey will after her much-publicized resignation from the editorship of She after a build-up of stress, that abandoning the doctrine of “juggling your life”, and making the alternative move into “downshifting” brings with it far greater rewards than financial success and social status. Nothing could persuade me to return to the kind of life Kelsey used to advocate and I once enjoyed: 12-hour working days, pressured deadlines, the fearful strain of office politics and the limitations of being a parent on “quality time”.
In America, the move away from juggling to a simpler, less materialistic lifestyle is a well-established trend. Downshifting — also known in America as “voluntary simplicity” — has, ironically, even bred a new area of what might be termed anti-consumerism. There are a number of best-selling downshifting self-help books for people who want to simplify their lives; there are newsletters, such as The Tightwad Gazette, that give hundreds of thousands of Americans useful tips on anything from recycling their cling-film to making their own soap; there are even support groups for those who want to achieve the mid-‘90s equivalent of dropping out.
While in America the trend started as a reaction to the economic decline — after the mass redundancies caused by downsizing in the late’80s — and is still linked to the politics of thrift, in Britain, at least among the middle-class downshifters of my acquaintance, we have different reasons for seeking to simplify our lives.
For the women of my generation who were urged to keep juggling through the ’80s, downshifting in the mid- ’90s is not so much a search for the mythical good life — growing your own organic vegetables, and risking turning into one — as a personal recognition of your limitations. (428 words  可读性:39.4  难度:12级)

一、原题的研究
1. Which of the following is true according to paragraph 1?
[A] Full-time employment is a new international trend.
The writer was compelled by circumstances to leave her job.
[C] “A lateral move” means stepping out of full-time employment.
[D] The writer was only too eager to spend more time with her family.
2. The writer’s experiment shows that downshifting
[A] enables her to realize her dream.
helps her mold a new philosophy of life.
[C] prompts her to abandon her high social status.
[D] leads her to accept the doctrine of She magazine.
3. “Juggling one’s life” probably means living a life characterized by
[A] non-materialistic lifestyle.                             a bit of everything.
[C] extreme stress.                           [D] anti-consumerism.
4. According to the passage, downshifting emerged in the U.S. as a result of
[A] the quick pace of modern life.               man’s adventurous spirit.
[C] man’s search for mythical experiences.       [D] the economic situation.

二、潜在命题点的挖掘

请参考后面的“三、原题的讲解”彻底弄懂原题后,再将文章读两遍,思考一下什么地方还可出题,出什么样的题。请继续做下面的试题:
5. From the beginning of the text we know that the author
[A] got indignant at her transference without promotion.
was disposed to her involvement in a modern trend.
[C] was reluctant to give up her noble position as a minister.
[D] resigned from her job because of being put out of favor.
6. In about 2.5 years after the writer quitted her full-time job
[A] she intended to create and publish two novels.
her tired excuse for exit brought her back to reality.
[C] her philosophy was imposed on her by Linda Kelsey.
[D] she became adapted to a less ambitious philosophy.
7. High profile career requires that the employee
[A] keep working round the clock.
be obliged to work to his/her last breath.
[C] have no children during his/her term of office.
[D] be prohibited to talk politics during office hours.
8. In the mid-90’s
[A] many downshifters dropped out of the courses of higher education.
woman downshifters realized their own weaknesses.
[C] some downshifters sought for unrealistic thrifty life.
[D] most downshifters returned to self-sufficient lifestyles.

三、原题讲解

第一题:
题眼研读:细节是非判断题。题干(Which of the following is true according to paragraph 1?)明示考 <1 >,所以要点是抓住其中的难句大意。②不仅生词多而且结构复杂,所以是考点。
  ①When I decided to quit my full time employment it never occurred to me that I might become a part of a new international trend. ②A lateral move that hurt my pride and blocked my professional progress prompted me to abandon my relatively high profile career although, in the manner of a disgraced government minister, I covered my exit by claiming “I wanted to spend more time with my family”.

a lateral move横向调动;high profile career地位较高的工作。②:一次职位变动伤了我的自尊心,并阻断了我的事业发展,这促使我放弃自己的地位较高的职业,当然,就像面子扫尽的政府部长那样,我也掩饰说“我只想与家人更多地呆在一起”。可见作者辞职是由于对工作变化的不满,所以 The writer was compelled by circumstances to leave her job. (作者迫于环境而辞职)为答案。 The writer was compelled by circumstances to leave her job. (作者迫于环境而辞职)为答案。把原文长长句用了几个词简单做了归纳,而且把辞职的原因用circumstances做了抽象化处理。
干扰项分析:[A] Full-time employment is a new international trend. (全职工作是新的国际趋势):拼凑①“full time employment”和“a new international trend”而成。[C] “A lateral move” means stepping out of full-time employment. (“A lateral move”意思是脱离全职工作):拼凑“A lateral move”和“full-time employment”而成。[A]和 [C]的意思与文章相反。[D] The writer was only too eager to spend more time with her family. (作者渴望能和家人共度更多时间):根据②“I wanted to spend more time with my family”编写。这是作者的借口,是表面现象,不是辞职的真正原因。
体会:选项中文中词汇用得越多,看着越面熟,越像一棵救命稻草,越不能选。
九阴真经解注:照抄原文不是解,A与原文相反,C照抄原文,D表面现象均不是解。
第二题:]

题眼研读:这是一道大意题。题干(The writer’s experiment shows that downshifting)中experiment的意思是“尝试”。其中downshifting带引号是因为这是个美国人造的新词,一般辞典查不到,要求考生通过下下文和downshifting的构词方式去猜测。该词原意是指汽车行驶中换低档,在本文引早晨为从紧张的现代生活变成简单、纯朴的生活。考点是 <2 >。
  ①Curiously, some two-and-a-half years and two novels later, my experiment in what the Americans term “downshifting” has turned my tired excuse into an absolute reality. ② I have been transformed from a passionate advocate of the philosophy of “having it all”, preached by Linda Kelsey for the past seven years in the pages of She magazine, into a woman who is happy to settle for a bit of everything.

①:奇怪的是,大约两年半写了两本小说之后,美国人称之为“downshifting”的尝试,却使我厌烦的借口变成了现实。②句架:I have been transformed from…into a woman…“我已从一个‘拥有一切’哲学的狂热支持者,变成了一个乐于接受‘任何东西只要一点’的女人。”说明了作者的人生态度发生了变化,原来的我是美国社会的主流观点——努力工作,拥有一切物质享受;现在的我安于简单的生活。作者的尝试帮助她形成了一个全新的生活观念,所以 helps her mold a new philosophy of life.(帮助她建立了新的生活观念)正确。
干扰项分析:[A] enables her to realize her dream. (使她实现了自己的梦想):利用①②中个别词汇“turn, reality, transform, happy”编造成一个故事,不合文意。[C] prompts her to abandon her high social status. (促使她放弃了较高的社会地位):把 <1 >中辞了职的部长变成作者了。[D] leads her to accept the doctrine of She magazine. (导致她接受了女性杂志的观点):把文中advocate of…of She magazine(原来作者接受的观点)留下,去掉了into…everything, (作者现在看法),使得意思和原文相反。
九阴真经解注:新的是解,含义深刻是解,抽象是解。
第三题:[C]

题眼研读:词义题。题干(“Juggling one’s life” probably means living a life characterized by)明示考查 <3 >。
  ① I have discovered, as perhaps Kelsey will after her much-publicized resignation from the editorship of She after a build-up of stress, that abandoning the doctrine of “juggling your life”, and making the alternative move into “downshifting” brings with it far greater rewards than financial success and social status. ② Nothing could persuade me to return to the kind of life Kelsey used to advocate and I once enjoyed: 12-hour working days, pressured deadlines, the fearful strain of office politics and the limitations of being a parent on “quality time”.

①句架: I have discovered…that abandoning…and making…bring with it far greater rewards than…重点是that引导的宾语从句:主语是由and连接的两个动名词短语,谓语是bring, it指代downshifting,宾语是rewards。大意:我发现,放弃“经营人生”的信仰,转而过一种“downshifting”的生活所带来的回报,比经济成功和社会地位更有价值。我们可以看出juggling your life和downshifting是相反的意思。②中冒号起到了解释作用,冒号前面被解释的名词是life, life受到了两个定语的修饰:凯茜所宣扬的、我也曾自得其乐的。对这种生活的描述是“每天12小时的工作日,压得人喘不过气来的最后期限,可怕而紧张的办公室权术,以及对生一个孩子一起共度的‘宝贵时光’的种种限制。”这与juggling your life同义,所以[C] extreme stress. (极度的紧张)正确。
干扰项分析:[A] non-materialistic lifestyle. (非物质主义的生活方式)和[D] anti-consumerism. (反对消费主义)分别取材于 <4 >中①和②, a bit of everything. (物质东西有点儿就行)取材于 <2 >②,均不合文意。
体会:解词汇题,一般三句话足够了:上一句、本句、下一句。同义和反义关系是解难词型词汇题的关键。
九阴真经解注:概括性是解,单词的含义由周围语言环境所决定。
第四题:[D]

题眼研读:细节题。本题(According to the passage, downshifting emerged in the U.S. as a result of)问downshifting在美国出现的原因。As a result of 是关键词,和 <5 >①的as a reaction to同义。
  ① While in America the trend started as a reaction to the economic decline — after the mass redundancies caused by downsizing in the late ’80s — and is still linked to the politics of thrift, in Britain, at least among the middle-class downshifters of my acquaintance, we have different reasons for seeking to simplify our lives.

  ① 从句“在美国出现的生活简化的趋势是经济衰落引起的。”可见[D] the economic situation. (经济状况)正确。注意同义替换:as a result of = as a reaction to; economic situation是economic decline概括性的同义词。
干扰项分析: [A] the quick pace of modern life. (现代生活的快节奏)和 man’s adventurous spirit. (人类的冒险精神)是根据常识编造的合理项。[C] man’s search for mythical experiences. (人们对神秘体验的探寻):取材于 <6 >中search for the mythical good life,文中对此持相对否定态度(注意:not so much…as…与其说……不如说……)。
九阴真经解注:因果关系常考。
四、新题的讲解

第五题:[A]

题眼研读:大意题。本题(From the beginning of the text we know that the author)是第一题的翻版。[A] got indignant at her transference without promotion. (对于未被提升而调离感到非常气愤)正确。
干扰项分析:was disposed to her involvement in a modern trend. (愿意融入一种现代趋势):取材于 <1 >①后半部分become…trend,但砍掉了表否定的it never occurred to me, 导致与文意相反,原文说作者从没想到自己的辞职成了世界的一种新趋势。[C] was reluctant to give up her noble position as a minister. (不愿意放弃自己部长的高贵地位):是第二题[C]的翻版。[D] resigned from her job because of being put out of favor. (由于不受宠而辞职): being put out of favor歪曲 <1 >②中hurt my pride而成。

第六题:[D]
题眼研读:大意题。本题(In about 2.5 years after the writer quitted her full-time job)是第二题的翻版。本题答案[D] she became adapted to a less ambitious philosophy.(她习惯了一种对生活抱负不大的态度)也是第二题 helps her mold a new philosophy of life的具体化。
干扰项分析:[A] She intended to create and publish two novels.(她打算创作、出版两部长篇小说):与文意有细微的差别, <2 >①说作者大允两年半写完了两本小说。 Her tired excuse for exit brought her back to reality.(她那厌烦的辞职借口把她带回到了现实中):对 <2 >①后半部分has turned…to reality 编写,但把
原文的宾语my tired excuse变成主语,而宾语变成了her.原文说“我的尝试把我的辞职借口变成了一个绝对现实”和不同。[C] Her philosophy was imposed on her by Linda Kelsey.(她的生活态度是Linda Kelsey强加的);错
体会:大量使用原文词汇的选项不是解,解是归纳或同义替换出来的。

第七题:[C]
题眼研读:细节题。题干(High profile career requires that employee)中关键词是high profile career,该词出现在 <1 >,但四个选项似乎和 <1 >没有关系,四个选项都是关于 <3 >末句,本题考查大家能否发现 <3 >末句后半部分是high profile career的具体化解释,及对其能否准确理解。
Nothing could persuade me to return to the kind of life Kelsey used to advocate and I once enjoued:12-hour working days, pressured deadlines, the fearful strain of office politics and the limitations of being a parent on “quality time”.
  冒号后面有四个并列短语,描述这种所谓的较高职位的生活方式。未在外国公司工作过的中国女大学生不容易体会出最后一个短语the limitations of being a parent on“quality time”的意思。我的一个大学同学在IBM工作了十年,她告诉我说工作了那么多年都不敢轻易要孩子,因为生孩子意味着丧夫来之不易的职位。本文说的就是这种情形,所以[C] have no children during his/her term of office.(在任职期间不能生孩子)正确。
干扰项分析:[A] keep working round the clock.(24小时不间断地工作):把原文的“12-hour working days”夸大为24小时。 be obliged to work to his/her last breath.(不得不玩命工作):程度太过。根据 pressured deadlines编造,把最后期限(deadlines)理解为死亡线了。[D] be prohibited to talk politics during office hours.(在办公时间禁止谈论政治):文中“office politics”指的是办公室内的勾心斗角,即权术。
第八题:
题眼研读:细节题。题干(In the mid-90’s)中时间出现在末段。
For the women of my generation who were urged to keep juggling through the’80s, downshifting in the mid-90’s is not so much a search for the mythical good life-growing your own organic vegetables, and risking turning into one—as a personal recognition of your limitations.(48 words)
句架:…downshifting…is not so much…as…not so much…as与其说……不如说……。破折号之间的内容是例子,解释说明a search for good life。大意:对我们这一代女性来说,整个80年代我们都曾被迫忙碌地生活,90年代中期的“downshifting”与其说是寻求神话般的好生活倒不如说我们认识到了自身的局限。所以 woman downshifters realized their own weaknesses.(追求简朴生活的女性认识到了自己的弱点)=personal recognition of your limitations,正确。
干扰项分析:[A] many downshifters dropped out of the courses of higher education.(许多简朴生活者从高校退学了):drop out取材于 <4 >末句。drop out 在[A]中是其传统意思“退学”。在文中引申为“脱离传统社会方式。”[C] some downshifters sought for unrealistic thrifty life.(有些简朴生活者追求不切实际的节俭生活):根据末段破折号内的内容概括,并取材于 <5 >末句中thrift一词。错在unrealistic这个词,downshifters的生活是会合实际的。[D] most downshifters returned to self-sufficient lifestyles.(多数简朴生活者返回到自给自足的生活方式):most一词程度太过,self-sufficient lifestyle是封建社会的生活方式,和downshifters过的简化生活( <4 >)不完全一样。
注释:1)lateral横向的,侧面的;2) prompt促使,驱使;3)profile侧面(像);4)disgraced大势已去的;5)tem称为,取名为;6)downshift换低速档;7)tired陈旧的,陈腐的;8)preach宣扬,极力鼓吹;9)build-up积聚,增强10)juggle忙碌地应付;11)politics手腕,权术,策略;12)tip的指导,忠告,插图,插页;12)downsize缩小规模;13)redundancy裁员,解雇;14)thrift节俭;15)acquaintance熟人,朋友



第四部分考研阅读之射雕英雄篇
—————最新阅读练习精选

Unit 1

Passage 1

Man, even in the lower stages of development, possesses a faculty which, for want of a better name, I shall call Number Sense. This faculty permits him to recognize that something has changed in a small collection when without his direct knowledge, an object has been removed from or added to the collection.
Number sense should not e confused with counting, which is probably of a much later stage, and involves, as we shall see, a rather complicated mental process. Counting, so far as we know, is an attribute exclusively human, whereas some rough species seem to possess a rudimentary number sense similar to our own. At least, such is the opinion of competent observers of animal behavior, and the theory is supported by a weighty mass of evidence.
Many birds possess such a number sense. If a nest contains four eggs, one can safely be taken; but when two are removed from the nest, the bird generally perceives the change. In some unaccountable way the bird can distinguish be teen two and three. But this faculty is by no means confined to birds. In fact, the most striking instance we know is that of the insect called the “solitary wasp”. The mother wasp lays her eggs in individual cells and provides each egg with a number of live caterpillars on which the young feed when hatched. Now, the number of victims is remarkably constant for a given species of wasp. Some species provide 5, others, 12,and some as high as 24 caterpillars per cell. But most remarkable is the case of the Genus Eumenus, a variety in which the male is much smaller than the female. In some mysterious way the mother knows whether the egg will produce a male or a female grub and assigns the quantity of food into shares accordingly; she does not change the species or size of the prey, but if the egg is male, she supplies it with five victims; if female, with ten.
The regularity in the action of the wasp and the fact that this action is connected with a fundamental function in the life of the insect make this last case less convincing than the one which follows. Here the action of the bird seems to border on the conscious: A squire sought to shoot a crow which had made its nest in the watchtower of his estate. Though he repeatedly tried, it was in vain: at the approach of man the crow would leave its nest, and watchfully wait until the man had left the tower and then return to its nest. One day the squire hit upon a trick: two men entered the tower, one remained within, the other came out and went away. But the bird was not deceived: it kept away until the man within came out. The experiment was repeated on the following days with two, three, then four men, yet without success. Finally, five men were sent as before, the crow lost count. Unable to make a distinction it promptly returned to its nest. (510 words   可读性:66.1  难度:8.9级)
1.    The main idea of this passage is that
[A] man’s rudimentary number sense is found in lower species.
counting is not to be confused with number sense.
[C] birds have a limited number sense.
[D] number sense is a primitive form of counting.
2.    Counting is different from number sense in that counting is
[A] dependent on simpler mental activity.
farther up the evolutionary scale.
[C] found in animals as well as man.
[D] present in animals on the basis of much evidence.
3.  The author feels that the number sense displayed by Genus Eumenus is
[A] more remarkable than of a crow.
less convincing than that displayed by the crow.
[C] not an instance of what the author is discussing.
[D] restricted to the female of this species.
4.  The crow was finally deceived because
[A] it was not afraid of the men in the tower.
it did not have a number sense.
[C] it could not distinguish between four and five.
[D] it was evening and the crow was unable to count the hunters.
5.  Many birds possess the faculty to distinguish a certain number because
[A] they developed the number sense by their mother’s feeding.
their number sense is, to a certain extent, similar to that of humans.
[C] Genus Eumenus is typical only for different egg shapes of its own.
[D] wasps get the faculty by preying on a variety of victims.
词汇:rudimentary 基本的 wasp 黄蜂 caterpillars 毛虫 grub 蛆 squire乡绅

passage 2

There is widespread belief that the emergence of giant industries has been accomplished by an equivalent surge in industrial research. A recent study of important inventions made since the turn of the century reveals that more than half were the product of individual inventors working alone, independent of organized industrial research. While industrial laboratories contributed such important products as nylon and transistors, independent inventors developed air conditioning, the automatic transmission, the jet engine, the helicopter, insulin, and streptomycin. Still other inventions, such as stainless steel, television, silicon, and plexiglass were developed through the combined efforts of individuals and laboratory teams.
Despite these findings, we are urged to support monopoly power on the grounds that such power creates an environment supportive of innovation. We are told that the independent inventor, along with the small firm, cannot afford to undertake the important research needed to improve our standard of living while protecting our diminishing resources; that only the huge assets of the giant corporation or conglomerate can afford the kind of expenditures that can produce the technological advances vital to economic progress . But when we examine expenditures for research, we find that more than half of the government expenditure is funneled into military research and product development. There are those who consider it questionable that these defense-linked research projects will account for an improvement in the standard of living or, alternately, do much to protect our diminishing resources.
Recent history has demonstrated that we may have to have to change our long-standing conception of process driven forward by competition. The price variable once perceived as the dominant aspect of the competitive process is now subordinate to the competition of the neo product, the new business structure, and the new technology. While it can be assumed that in a highly competitive industry not dominated by a single corporation, investment in innovation—a risky and expensive budget item—might meet resistance from management and stockholders who might be more concerned with cost-cutting, efficient-organization, and large advertising budgets, it would be a shocking error to assume that the monopolistic producer should be equated with bountiful expenditures for research. Large-scale enterprises tend to operate more comfortably in stable and secure circumstances, and their managerial bureaucracies tend to promote status and resist the threat implies in change. Furthermore, the firm with a small share of the market will aggressively pursue new techniques and different products, since with little supplied interest in capital equipment or plant it is not it is not deterred from investment in innovation. In some cases, where inter-industry com petition is reduced or even entirely eliminated, the industrial giants may seek to avoid capital loss by deliberately preventing technological progress.
The conglomerates are not, however, completely exempt from strong competitive pressures; there are instances in which they, too, must compete, as against another industrial leviathan, and then their weapons may include large expenditures on innovation. (477 words  可读性:20.5  难度:12级)
6.    According to paragraph 1, important inventions of the 20th century
[A] sometimes reduced our standard of living and diminish industries.
came primarily from the huge laboratories of monopoly industries.
[C] were produced at least as frequently by independent inventors as by research teams.
[D] had greater impact on smaller firms than on conglomerates.
7.  From paragraph 2, we learn that
[A] it is reasonable to support independent inventors because they lack capital.
it is beyond all doubt that small enterprises alone contribute to promote the living standard.
[C] were produced at least as frequently by independent inventors as by research teams .
[D] had greater impact on smaller firms than on conglomerates.
8.  Management and stockholders might be deeply concerned with cost-cutting rather than innovation if
   [A] they are faced with strong competition in a field not dominated by one of the industrial giants.
they are very stable and secure and hold a monopoly in their industry.
[C] they have produced some of the important in inventions of this century.
[D] it still remains doubtful whether defense research is the cause of better living.
9.  Which of the following statements is implied in the passage?
[A] In the past, important inventions were produced by both individuals and corporate teams.
For a better living, the money spent on military research should be reduced.
[C] The development of the automatic transmission is not credited to organized industrial research.
[D] Industrial giants may deliberately suppress innovations to avoid capital loss resulting from obsolescence.
10.  The purpose of this writing is to
   [A] advocate an increase in governmental support of organized industrial research.
point out a misconception about the relationship between the industrial research and monopolistic power in industry.
[C] show that America’s strength depends upon individual ingenuity and resourcefulness.
[D] encourage free market competition.
词汇:insulin 胰岛素 plexiglass 有机玻璃 streptomycin 链霉素 assets 资产 conglomerate 企业集团 funnel 汇集 leviathan 《圣经》象征邪恶的海中怪兽

passage  3

This book is written expressly for students in an attempt to present the material that is most useful and interesting to them. Previous courses in chemistry are not necessary for the understanding of the material, although those students who have learned chemistry sections that follow.
The author has felt that in the past there was an improper selection of material from inorganic, organic, and biochemistry in the majority of the textbooks of chemistry for nurses. The tendency has been to develop the inorganic chemistry to such an extent that organic and biochemistry are covered too briefly. The recent advances in biochemistry and their widespread application to the practice of medicine and nursing have considerably altered the situation. Not only is biochemistry more closely allied to the practice of medicine and nursing but it is also of more interest to the student. In the author’s experience the response to biochemistry has always been more favorable than to the other sections. Within the brief period allotted to chemistry, therefore, the sections no inorganic, organic, and biochemistry should be so arranged that a good share of the time is spent in the study of biochemistry. This book presents mainly those fundamentals of inorganic and organic chemistry that are necessary for the understanding of the section on biochemistry.
The fundamental points suggested in the Curriculum Guide are included in the book, with some additions in the biochemistry section. The author feels that a study of urine, vitamins, nutrition, and hormones is so obviously a part of biochemistry that at least the fundamentals should be included in this course.
The book has been planned in such a way that it may be adapted to various courses in chemistry. The material suggested by the Curriculum Guide is covered in the first nineteen chapters and may be used in accelerated courses or where minimum time is allotted to chemistry. When the time allotted to the course is sixty to ninety hours the entire contents of the book may be used to advantage. While the book has been written especially to fit the needs of Schools of Nursing it could readily be applied in instances where students are required to take but one course in chemistry. The apathetic attitude of nonprofessional students toward a course in inorganic chemistry may well be overcome by the proper presentation of material of material selected from inorganic, organic, and biochemistry. (418 words  可读性:31.3 难度:12级)
11.  To read this book
   [A] previous courses in chemistry is necessary.
a good mastery of biochemistry is essential.
[C] one needn’t have studied chemistry before.
[D] one must first review his high school courses.
12.  The author describes the fact that
   [A] medical workers should have the know of organics.
nurses are generally short lack of text books of chemistry.
[C] experts have scored great achievements in biochemistry application.
[D] students majoring in nursing are fond of biochemistry sthdy.
13.  It can be inferred form the passage that
   [A] the negligence of inorganic chemistry is a tendency.
organic and biochemistry are fundamental to the majoring of medicine and nursing.
[C] the emphasis on organic, biochemistry is unnecessary.
[D] nurses are somewhat incompetent due to improper textbooks.
14.  From the passage we mainly learn that
   [A] the author suggests principles should be essential to the learning of the course.
the students should have a better planning of the time used.
[C] the students should study the Curriculum Guide first.
[D] the students taking only one course in chemistry are advised to use this book.
15.  Paragraph 4 suggests that one thing characteristic of this book is its
   [A] simplicity in presentation.
thoroughness in the field.
[C] vividness of the language.
[D] adaptability to various needs in the field.
词汇: urine 尿 apathetic 冷淡的

passage  4

The subject of my study is women who are initiating social change is a small region in Texas. The women are Mexican Americans who are, or were, migrant agricultural workers. There is more than one kind of innovation at work in the region, of course, but I have chosen to focus on three related patterns of family behaviour.
The patternⅠlife-style represents how migrant farm workers of all nationalities lived in the past and how many continue to live. I treat this pattern as a baseline with which to compare the changes represented by patternⅡand Ⅲ. Families in patternⅠwork on farms all year round, migrating for as many as ten months each year. They work and travel in extended kin units, with the eldest male occupying the position of authority. Families are large, eight or nine children are not unusual and all members are economic contributors in this strategy of family migration. Families in patternⅡmanifest some differences in behaviour while still maintaining aspects of migration to allow children to finish the school year. Parents in this pattern often find temporary local jobs as checkers to make up for lost farming income. PatternⅡfamilies ussaly have fewer children than do patternⅠfamilies.
The greatest amount of change from patternⅠ, however, is found in patternⅢ families, who no longer migrate at all. Both parents work full time in the area and have an average of three children. Children attend school for the entire year. In patternⅢ, the women in particular create new roles for themselves for which no local models exist. They not only work full time but may, in addition, return to school. They also assume a greater responsibility in family decisions than do women in the other patterns. Although these women are in the minority among residents of the region, they serve as role models for others, causing moderate changes to spread in their communities.
Now opportunities have continued to be determined by pre-existing values. When federal jobs became available in the region, most involved working under the direction of female professionals such as teachers or nurses. Such positions were unacceptable to many men in the area because they were not accustomed to being subordinate to women. Women therefore took the jobs, at first, because the income was desperately needed. But some of the women decided to remain at work after the family’s distress was over. These women enjoyed their work, its responsibility, and the companionship of fellow women workers. The steady, relatively high income allowed their families to stop migrating. And, as the benefits to these women became increasingly apparent, they and families became even more willing to consider changes in their lives that they would not have considered before. (460 words  可读性:49 难度:10.7级)
16.  Which of the following titles best reflects the main focus of the passage?
[A] A Survey of Three Mexican-American Families at Work in Texas.
Brought by Effects on Family Unity.
[C] Changes in the Life-styles of Migrant Farm Workers.
[D] Farming or Family: The Unavoidable Choice for Migrant Farm Workers.
17.  All of the statements tell about changes of patternⅡchildren compared with patternⅠchildren EXEDPT
   [A] they migrate for part of each year.
they spend less time contributing to family income.
[C] they spend more months in school.
[D] their parents sometimes work at jobs other than farming.
18.  According to the passage, which of the following is NOT true of women in patternⅢfamilies?
   [A] They earn a reliable and comparatively high income.
They continue to work solely to meet the urgent needs of their family.
[C] They enjoy the fellowship involved in working with other women.
[D] They serve as models models of behaviours for others in the region.
19.  The author’s attitude towards the three patterns of behaviour is best described as one of
   [A] treat admiration.
unbiased objectivity.
[C] dissatisfaction.
[D] indifference.
20.  White collar jobs are obtainable to women in the area because
   [A] women are more professional than men in their jobs.
women have to earn to support families.
[C] women are not very particular about positions.
[D] women take greater responsibility than men in their jobs.
注:当年我们那一代人参加英语考试的时候,由于条件有限,很多英语题没有详解,所以养成了自己钻研的习惯。现在很多同学喜欢
吃现成的,这个习惯很不好。故本部分略去本已写好详解,望同学们仿造前面所述方法,靠自己的努力,将你心比我心,把这些试题彻底弄明白。
答案:
passage 1    1.[A]      2.      3.      4.[C]      5.
passage 2    6.[C]      7.[D]      8.[A]      9.     10. [D]
passage 3   11.[C]     12.[C]     13.     14. [D]      15.[D]
passage 4   16.[C]     17.[A]     18.     19.     20.[C]

Unit 2
Passage 1

An important research tool in protecting wildlife involves tracking the journey of a species from its birthplace to its mating site. In the case of the green turtle, which does not start breeding until the age of 30, this has proven to be a difficult challenge. Researchers have tried to monitor its meanderings by tagging it with metal disks or wires, but the turtle’s dramatic growth over the decades—from four inches at infancy to four feet in adulthood—has hindered efforts to keep the tags in place.
Seeking an alternative tracking method, Brian W. Bowen, an evolutionary geneticist, turned instead to natural markings. Bowen analyzed mitochondrila DNA form eggs and hatchings at four green-turtle breeding sites in the Atlantic and the Caribbean. The existence of variations in DNA among geographically distinct groups has helped scientists sway the balance between different theories of the mating habits of the green turtle.
Their findings, Bowen asserts, give evidence to the “natal (birth) homing” theory proposed in the 1960s. This theory holds that while reptiles born in different regions may share common feeding grounds away from home, the animals part company at breeding time, each swimming hundreds or thousands of miles to breed and nest at its own birthplace.
At the same time, the new work undercuts a competing theory known as the social facilitation model. This approach contends that virgin female turtles randomly follow experienced breeders to nesting sites regardless of their birthplace. Bowen concludes that such “social mixing” must be rare in green turtles. Widespread interbreeding of diverse turtle groups, he points out, would have long ago erased the genetic differences highlighted by the study.
Bowen and his co-workers, however, offer several illustrations to the interpretation of their findings. They note that turtles from two of the nesting sites—Florida’s Hutchinson Island and Costa Rica’s Tortuguero sanctuary (an area where birds or animals are protected) —had indistinguishable mitochondrila DNA sequences. This finding indicates that either some social mixing occurred between these two groups that the DNA test was not sensitive enough to detect extremely subtle differences in DNA. In addition, they report that one of eight study turtles from Aves Island, off Venezuela, showed the same gengtic pattern as the Tortuguero and Hutchinson turtles in their sample. Nonetheless, Bowen’s study appears to have enhanced the idea that most populations of green turtles are genetically distinct and return to their birthplaces at nesting time. (421 words    可读性:40.0  难度:12级)
1.  We may get the probable result from Bowen’s analysis of the mitochondrial DNA that
    [A] virgin female turtles casually follow experienced breeders to nesting place.
green turtles usually go back to their birthplace during mating time.
[C] there is widespread interbreeding among turtles form different nesting places.
[D] subtle differences in DNA sequences are not detected by the current tests.
2.  It can be inferred that Bowen and his co-workers believe the following EXCEPT
   [A] green turtles seldom interbreed with those from a different birthplace.
DNA testing does not always reveal genetic differences.
[C] frequent interbreeding can lead to fewer genetic differences.
[D] subtle differences in DNA sequences are not detected by the current tests.
3.    The primary purpose of the scientists’s study may be
   [A] to prove the social facilitation model theory.
to show the non-existence of interbreeding among green turtles.
[C] to show hardships of tracking methods.
[D] to determine whether green turtles return to their birthplace during breeding time.
4.  The author regards the validity of the social facilitation model with
   [A] exact impartiality.
careful admiration.
[C] hesitant disbelief.
[D] reasoned doubt.
5.  It can be inferred from this passage that as far as scientific study is concerned
   [A] scientific findings cannot be refuted.
scientific theories should not have any defects.
[C] scientific findings may sometimes be reversed.
[D] scientists should not swing form one theory to another theory.
词汇:meanderings 漫步 mitochondrial 线粒体 reptiles 爬行动物

passage  2

A few generations ago, a poor black girl who complained in class that “Dis boy, he be boverin me” was apt to be ridiculed by her teacher for slovenly speech or, worse, written off as illiterate. With the rise in black consciousness, however, educators and linguists became confused in a spirited debate over whether black dialects be recognized as unwritten languages, compiled and even taught in school.
The issue lay dormant until it was resurrected by some black parents in Ann Arbor. They are suing the school district, charging discrimination for its failure to help their children overcome barrier. Few linguists still contend that black English is a distinct language. But they do not excuse a school system’s failure to recognize that dialects can be handicaps and cause poor children to be mistaken for dull-brained.
Despite their deviation from standard English, black dialects came to be seen in the 1960’s as one of many symbols of racial pride. To reject them was viewed as a defamation of the literacy and worth of many blacks. Advocacy groups, like Speech Therapists for Human Dignity in Washington, were formed to protect the idiom. Reform minded linguists the cause, devising classroom texts in dialect and designing teaching methods borrowed from foreign language instruction. Some went so far as to urge that schools in coastal South Carolina teach blacks in their native Gullah dialect.
Still, like so many reforms, the case for accepting black English was oversold. Textbooks written in dialect have worked on miracles in the classroom; reading scores remain alarmingly low. Black parents have resisted the propagandists, knowing that their children must become fluent in standard English to compete in the job market. The linguists who linked language with learning deficiencies are still at a loss to explain why rural Japanese children who grow up with even greater disparities their dialects and the official are not similarly hampered in school.
It is sad that the courtroom must become the forum for determining whether the Ann Arbor school system has taken due account of the unique needs of children. But both parties will miss the print if they dwell on the abstract questions of whether black English is, strictly, a language. Of course it isn’t, any more than Louisiana Cajun or Brooklynese are. The central problem, as always, is the willingness of educators to make special efforts to teach needy children—and of their parents to arouse a thirst for learning. (412 words  可读性:52.1  难度:11级)
6.  According to the passage,
   [A] if black English is taught, there is no point in teaching standard English.
the importance of deciding whether or not black English is a language, can’t be overstressed.
[C] the importance of deciding whether in dialect has caused reading scores to remain low.
[D] writing textbooks in dialect has not proved a great success.
7.  The passage does NOT support which of the following statements?
[A] Blacks must learn standard English to get ahead in the world.
Teaching dialects is one of the reforms to protect native idioms.
[C] Standard English is superior to black English.
[D] Louisiana Cajun and Booklynese are strictly languages.
8.  The black parents sued the school district because
   [A] the problem of whether black English should be taught was not solved.
the school viewed black dialects as the cause of their children’s failure in learning language.
[C] their children failed to grasp the standard English.
[D] the school thought their children to be idiots.
9.  The author felt sad about the law suit the school faced because
   [A] the parents charged the school wrongly.
the two parties didn’t get the main point of the problem.
[C] it was a shame for the school to be accused.
[D] the school would never enroll students if it lost the suit.
10.    From this passage we can infer that
   [A] Parents and educators should dedate whether they should teach black English.
Parents should not interfere in the teaching problems.
[C] Educators should not teach black English.
[D] Parents and educators should make every effort to stimulate the children to learn.
词汇: slovenly 潦草的 dormant 搁置 resurrect 复活 disparity 悬殊

passage 3

Modern technology has developed in a manner which often conflicts the environment. Nevertheless, it is possible and practical to achieve a high level of technology without altering the overall environment or depleting nonrenewable natural resources.
There are two basic methods of conserving the natural environment. One is alternative technology in which techniques (such as windmills and solar energy) must be found for providing power in a more decentralized manner. The other is to try to minimize the effects of modern technology by the more efficient use of materials and more effective pollution control. The differences between the two approaches depend upon whether today’s metropolis is too large to be economical. There comes a print when the quality of life can no longer be improved because of the high population density and the largely unsuccessful attempts to meet its demands.
Present energy policies of the western world are considering new energy sources to meet the demands of a growing population. The main sources used have been oil, gas, and coal which are in limited supply and nuclear power, which has problems concerning radioactive waste storage. Geothermal, hydroelectric, hydrothermal, wind, and solar energy are all alternative possibilities, but only solar energy is likely to become a main power source for many countries.
Many of the materials used daily in a technological society are destined to be discharged. Rags, paper, metal, glass, and plastics are all substances that could be recycled or reused. For example, ground glass can be used for road surfacing. Many industrial residues, like slag and, can be used in the same way, or as building materials. Research is being done to determine methods of separating useful products from industrial and domestic waste. Plastics and organic materials contain hydrocarbons which could be used for madding gas and oil and are presently destroyed by burning. Extracting these commodities from waste may be vital in the future, considering that oil and gas are non-renewable natural resources.
The processing of sewerage requires considerable amounts of electricity. Vast quantities of sewerage have to be pumped and treated daily and several megawatts of power are needed for the task. Many sewerage treatment plants in the U.S. have become self-sufficient by suing the methane gas given off by sewerage. This is collected in gas holders and burned to produce heat and create electricity. The solid waste itself is often used for soil enrichment. (399 words  可读性:37.6  难度:12级)
11.  From this passage we can determine that
    [A] conservation technology is a necessity.
unclear power is the energy source of the future.
[C] problems of energy conservation are not worldwide.
[D] modem technology can better the overall environment.
12.  Alternative technology suggests that
   [A] new ways be found to burn oil, coal and gas.
new energy resources be considered.
[C] new ways be found to store radioactive waste.
[D] new technology replace the traditional technology.
13.  Conservation technology can be better used in
   [A] densely-populated metropolises.
big industrial cities.
[C] satellite cities.
[D] high-lands and plateaus.
14.  We can infer from the passage that
   [A] if the cities become extremely large ,it will be difficult to tbtter the standard of living.
the metropolises of today are controlling the growth of their population in order to be economical.
[C] more and more small towns are going to appear to replace the metropolises.
[D] the larger the population, the less we need to employ alternative technology.
15.  The passage tells us
   [A] scientists have found the best way to dispose of industrial waste.
plastic waste is burned to produce gas.
[C] recovering useful materials from waste maste may be dangerous and polluting.
[D] industrial waste and house refuse are being studied to produce renewable materials.
词汇: metropolis 大都市 residues 残余 slag 熔渣 sewerage 污物处理系统 methane 甲烷

passage 4

Space research has greatly benefited the field of communications. International television is relayed around the globe by a network of communications satellite orbiting thousands of miles above the earth. International television affords a living view of such events as the Olympic games first in 1964 from Tokyo, then in 1968 from Mexico. In 1972 we saw the winter Olympics from Sapporo, Japan, and the summer games from Munich, Germany. Today, we simply take it for granted that everything from newscasts in Perth to live performances in Paris will come to our living rooms.
International TV and its debut on July 10, 1962, when many expectant viewers in the U.S. , and a few in France and England, watched a taped black-and-white picture of an American flag flapping in the Maine breeze to the recorded accompaniment of the “Star Spangled Banner”. Picture and sound, transmitted skyward over the Atlantic from a large horn-shaped antenna near Andover, Maine, were being transmitted back to Andover and to Holmdel, N.J. , from a new earth satellite, Telstar 1, launched by NASA 15 hours earlier from Cape Canaveral, Florida.
The following day the 170-pound Telstar 1, speeding around the globe every 158 minutes, relayed the first TV pictures westward form Europe, black and white form both France and England, and within a week the first in color. Two weeks later mass audiences on both sides of the Atlantic watched the first international exchange of live TV. Viewers in Europe saw the Statue of Liberty, a baseball game between the Phillies and the Cubs in Chicago, a Presidential press conference, buffalo roaming the South Dakota plains, and the Mormon Tabernacle Choir singing at Mount Rushmore. Americans, in turn, were able to glimpse Big Ben from London’s Tower Bridge, the Coliseum in Rome, the Louver in Paris, the Sistine Chapel in Vatican City, Sicilian fishermen reefing their nets, and the reindeer near the Arctic Circle in Norway.
Other prominent international events seen here by relay from communications satellites were Pope Paul XI’s coronation at the Vatican, Winston Churchill’s funeral in London, the induction of Prince Charles at Carnarvon, Wales, the opining of Expo 70 in Japan, and President Nixon’s visits to China and the Soviet Union. Most common of all are the daily broadcasts of spot news from crucial localities all over the world. Millions of Americans now watch them on their TV screens with little thought about the significant credit line, “via satellite.” (405 words  可读性:35  难度:12级)
16.  When the first international television broadcast was made
    [A] people all over the world were expecting to watch the programs.
it was kept as a secret.
[C] the American people were especially concerned about its success.
[D] only those who were working for NASA were concerned about it.
17.  From the passage we know that the first communications satellite
[A] was put into operation shortly after it was launched.
required many days of testing before it was put into operation.
[C] experienced several technical difficulties at first.
[D] could transmit the color pictures into black-and –white.
18.  In this article, the author discusses the
   [A] different functions of the communications satellite.
different types of programs beamed across the Atlantic.
[C] different purposes of the Conferences on International Television.
[D] important roles of NASA.
19.  The initial broadcasts between Europe and America suggested that
   [A] countries can cooperate with space launchings.
much effort went into achieving variety in America than in sights and colors.
[C] television is more popular in America than in Europe.
[D] the broadcasts can only be relayed between Europe and America.
20.  From the passage we can conclude that
   [A] television programs may originate from any place in the world.
only spectacular events are broadcast via satellite.
[C] the United States spends more on satellites than any other country.
[D] satellites can be put into operation immediately after they are launched.
词汇:coronation 加冕典礼
答案:
passage 1    1.     2.[D]     3.[D]     4.[D]     5.[C]
passage 2    6.[D]     7.[D]     8.[C]     9.    10.[D]
passage 3   11.[A]    12.    13.[D]    14.[A]    15.[D]
passage 4   16.[C]    17.[A]    18.    19.    20.[A]

Unit 3

Passage 1

Gold can be found in many different kinds of rocks and in many geological environments. It is often found with other metals. In fact, more than one third of the gold produced in the United States is a byproduct from mining other metallic ores. Where base metals are deposited, either in veins or as scattered mineral grains, minor amounts of gold are usually deposited with them. Deposits of this type are mined for the predominant metals, but during processing of the one, the gold is also recovered.
Some deposits of base metals are so large that even though they contain only a small amount of gold per ton, so much is mined that a substantial amount of gold is recovered. Gold recovered from copper ore mined at the vast open-pit mine at Bingham, Utah, for example, almost equals the amount of gold produced from the largest mine in the United States.
Geologists study all the factors that control the origin and emplacement of mineral deposits, including gold. Studies of volcanic rocks in the field and in the laboratory lead to an understanding of how they crystallized to solid rock, and how mineral-bearing solutions and gases formed within them. Studies of rock structures, of rock structures, such as folds, faults, fractures, and joints, and of the effects of heat and pressure on rocks suggest why and where fracturing of the crust took place and where veins might be found. Knowledge of the physical and chemical characteristics of rocks yields information on the pattern of fractures and where to look for them. Studies of weathering processes and transport of material by water enable geologists to predict the most likely places for placer deposits to form.
Research on prospecting methods has led to the development of chemical and spectrographic laboratories that are fitted whit newly designed analytical instruments capable of detecting and rapidly measuring the amount of gold and other valuable metals that may be present in the rocks and ores. These laboratories can accompany the geologist into the field and, by providing on-the-spot analyses of selected samples, guide the geologist in his search.
The occurrence of gold is not irregular; its presence in various kinds of rocks and formation under differing environmental conditions follow natural laws. As geologists increase their knowledge of the ore-forming processes they can expect to improvetheir ability to find the gold. (398 words  可读性:42.5   难度:12级)
1.  Which of the following is true according to the first paragraph?
   [A] Most of the gold are mined from the base metal ores.
Most of the gold production in the U. S. is the output from gold mines.
[C] Where there is base metal ore, there is always large amount of gold deposited.
[D] The gold output from the base metal mine equals that controls the origin of the largest gold mine in the U. S.
2.  Which is NOT a factor geologists study that controls the origin of the mineral deposits?
    [A] The properties of the rocks.
The organization of the rocks.
[C] The placer deposits.
[D] The transfer of the natural forces.
3.  Spectrographic laboratories are really.
   [A] mobile units for complicated technology.
massive structures of complicated technology.
[C] branch laboratories of colleges and universities.
[D] newly designed machines fixed in the laboratories.
4.  The author in the article that
   [A] the occurrence of gold does not follow natural laws.
the quality of base metal is improved if it is near gold.
[C] heat and pressure have a great effect on creating minerals.
[D] have the larges gold deposits copper ores.
5.  We can conclude from this passage that
   [A] most smelting plants are not equipped to recover gold.
gold deposits are usually found in base metals.
[C] much gold is lost in inefficient mining operations.
[D] geologists have improved fully their gold-finding abilities.

Passage 2

For most of us, work is the central, dominating fact of life. We spend more than half our conscious hours at work, preparing for work, traveling to and from work. What we do there largely determines our standard of living and to a considerable extent the status we are accorded by our fellow citizens as well. It is sometimes said that because leisure has become more important the indignities and injustices of work can be pushed into a corner; that because work is intolerable, the people who do it should compensate for its boredoms and frustrations by concentrating their hopes on the other part of their lives. I reject that as a counsel of despair. For the foreseeable future the material and psychological rewards which work can provide will continue to play a vital part in determining the satisfaction that life can offer. Yet only a small minority can control the pace at which they work or the conditions in which their work is done; only for a small minority does work offer scope for creativity, imagination, or initiative.
Inequality at work and in work still is one of the cruelest and most glaring forms of inequality in our society. We cannot hope to solve the mire obvious problems of industrial life, many of which arise directly or indirectly from the inequality at work. Still less can we hope to create a decent and humane society.
The most glaring inequality is that between managers and the rest. For most managers, work is an opportunity and a challenge. Their jobs engage their interest and allow then to develop their abilities. They are able to exercise responsibility; they have a considerable degree of control over their own and the other’s working lives. Most important of all, they have the opportunity to initiate. By contrast, for most manual workers, work is a boring, monotonous, even painful experience. They spend all their working lives in conditions continue. The majority have little control over their work; it provides them with no opportunity for personal development. Often production is so designed that workers are simply part of the technology. In offices, many jobs are so routine that workers justifiably feel themselves to be mere cogs in the bureaucratic machine. As a direct consequence of their work experience, many workers feel alienated from their work and their firm, whether it is in public or in private ownership. (419 words  可读性:51.5  难度:10.2)
6.  According to the author it’s true about work that
   [A] whether you are happy in life largely depends on whether your work is rewarding.
leisure becomes more and more important than work.
[C] people should try to avoid the intolerable unfairness of work.
[D] concentrating on your work is a counsel when you are in despair.
7.  What does the author think is needed to solve our industrial problems?
   [A] Equality in salaries.
A reduction in the work time.
[C] An improvement in moral standards.
[D] Amore equal distribution of responsibility.
8.  What advantages does the author say managers have over the other worker?
   [A] They cannot lose their jobs.
They can work at whatever interests them.
[C] They can get time off to attend courses.
[D] They can make their own decisions.
9.  Working conditions generally remain bad because
   [A] the workers are unwilling to change them.
no one can decide what to do about them.
[C] managers do not want to change them.
[D] office workers want to protect their position.
10.  What frustrates the workers in a modern properly.
   [A] They are incapable of doing their work properly.
Their work interferes with their private lives.
[C] They feel they are merely a part of their machines.
[D] Their life is complicated by technological advances.
词汇: cog 齿轮上的轮牙

passage 3

Everybody loathes it, but everybody does it. A recent poll showed that 40% of Americans hate the practice. It seems so arbitrary, after all. Why does a barman get a tip, but not a doctor who saves lives?
In America alone, tipping is now a $ 16 billion-a-year industry. Consumers acting rationally ought not to pay more than they have to for a given service. Tips should not exist. So why do they? The conventional wisdom is that tips both reward the efforts of good service and reduce uncomfortable feelings of inequality. The better the service, the bigger the tip.
Such explanations no doubt explain the purported origin of tipping—in the 16th century, boxes in English taverns carried the phrase “To Insure Promptitude” (later just “TIP”). But according to new research from Cornell University, tipping no longer serves any useful function.
The paper analyses data from 2, 547 groups dining at 20 different restaurants. The correlation between larger tips and better service was very weak; only a tiny part of the variability in the size of the had anything to do with the quality of service. Customers who rated a meal as “excellent” still tipped anywhere between 8% and 37% of the meal price.
Tipping is better explained by culture than by economics. In America, the custom has become institutionalized: it is regarded as part of the accepted cost of a service. In a New York restaurant, failing to tip at least 15% could well mean abuse from the waiter. Hairdressers can expect to get 15~20%, the man who delivers your groceries$ 2. In Europe, In many Asian countries, tipping has never really caught on t all.
How to account for these national differences? Look no further than psychology. According to Michael Lynn, the Cornell paper’s co-author, countries in which people are more extrovert, Sociable or neurotic tend to tip more. Tipping relieves anxiety about being served by strangers. And, says Mr. Lynn, “in America, where people are outgoing and expressive, tipping is about social approval. If you tip badly, people think less of you. Tipping well is a chance to show off.” Icelanders, by contrast, do not usually tip—a measure of their introversion, no doubt.
While such explanations may be crude, the hard truth seems to be that tipping does not work. It does not benefit the customer. Nor, in the case of restaurants, does it actually incent the waiter, or help the restaurant manager to monitor and assess his staff. Service people should “just be paid a decent wage” which may actually make economic sense. (448 words  可读性:53.7  难度:9.0级)
11.  Which is true according to the passage?
   [A] It is regulated that the customers must pay a tip if they want to get good service.
There exists the tipping custom in each country.
[C] In some countries, tipping has become an industry.
[D] More and more people are in favor of tipping.
12.  What can we know about the origin of “tip”?
   [A] It originated from the English inn service.
The original purpose of tip was to ensure that waiter could get more money.
[C] The waiter threatened the customers with bad service if no tips were given.
[D] It originated in a small English village.
13.  We can get information from the article that
   [A] tipping is very popular in European countries.
in Asian countries tipping never existed.
[C] tipping is more popular in America than in Europe.
[D] tipping is popular in America because the Americans are much richer.
14.  People who dine in a New York restaurant
   [A] are not expected to give any tip to the waiter or waitress.
had better tip more than 15% so as not to be shamed.
[C] may be looked down upon when offering a considerable tip.
[D] are thought of as generous if they tip 15%.
15.  The author thinks that
   [A] tipping can benefit greatly a country’s economic growth.
tipping can ensure the quality of service a customer receives.
[C] tipping can improve a country’s cultural environment.
[D] tipping is not conducive tertiary industry.
词汇: loathe 憎恨 purported 有涵义的 discretionary 随意的 extrovert 性格外向的人 neurotic 神经过敏的人 introversion 不爱交际

passage 4

Medicine achieved its splendid eminence by applying the principle of fragmentation to the human condition. Our bodily ills have been split up and committed to different experts: an itch to the dermatologist, a twitch to the neurologist and if all else fails, a visit to the psychiatrist. For this last, intangible function of the family doctor has been taken over by the specialist confessional.
Abroad, the family doctor is almost extinct, In Germany, every doctor “specializes”. In Israel, you queue at one desk for a cut finger, at another for a sprain, and a third for shock—if all three symptoms resulted from one accident. In Britain, both the growing importance of hospital facilities and the reluctance of General Practitioners to unite their resources has gone far towards making the surgery an over-loaded sorting place for hospital clinics. There is no room for the amateur—be it in delivering a baby or calming a neurotic.
Consultants and G. P. s begin the same way, as medical students obliged to cultivate detachment. But whereas a family doctor gets involved in the intimate details of his “parish”, the consultant need only meet aspects of the patient relevant to his specialty. The more he endeavors to specialize, the more irrelevant phenomena must be shut out. Beyond the token bedside exchanges he need not go.
Consequently, in a surgical ward, there are on people at all: only an appendectomy, a tumor, and a “terminal case”. To make impersonality easier, beds are numbered and patients are numbers. Remoteness provides the hospital with a practical working code.
Nurses too have evolved their own defense system. Since they care for individuals, they could with dangerous ease become too involved. The nursing profession has therefore perfected its own technique of fragmentation, “task assignment”. This enables one patient’s need to be split up among many nurses. One junior will go down a row of beds inserting a thermometer into a row of mouths. Whether the owners are asleep or drinking tea is irrelevant; the job comes first. In her final year, a student will undertake the pre-medication of patients on theater-list. She has by that time learnt to see them as objects for injection, not frightened people.
Nursing leaders realize the drawbacks in this system. There has talk of group-assignment to link nurses with particular patients and give some continuity. But the actual number of experiments can be counted on one hand. Nurses, as they often plead touchingly, “are only human”. They avoid responsibility for life and death. If responsibility is split into a kaleidoscope of routines, it weighs less on any one person. (437 words  可读性:48.2  难度:9.8级)
16.  People prefer to the hospital surgery because
  [A] the theory of fragmentation makes the surgeon a specialist.
  the specialist provides the patients with a confession room.
     [C] both the specialists and medical instruments are provided.
[D] General Practitioners have no financial resources.
17.  The practicing of fragmentation may result in
   [A] General Practitioners’ loss of employment.
urgent requirement of the consultants.
[C] the versatile qualities of the nurses.
[D] the transfer of specialists’ business to family doctors.
18.  By explaining the fragmentation in nursing profession, the author wants to show
   [A] the nurses are responsible for the patients.
each nurse is in charge of a different patient.
[C] the nurse are so devoted to the job that they care nothing about themselves.
[D] the nurses are ready to defend against emotionally involved with patients.
19.  We may infer from the passage that the author is
   [A] praising the British hospitals.
criticizing the British hospitals.
[C] sharing with the doctors the principles of fragmentation.
[D] giving a high opinion of the British specialists.
20.  Which of the following is DOT mentioned by the author concerning the British hospitals?
   [A] Over-specialization.
Doctors’ concern with patients.
[C] Administrative impersonality.
[D] Nurses’ insufficient involvement.
词汇: dermatologist 皮肤病专家 sprain 扭伤 appendectomy 阑尾切除术kaleidoscope 千变成化
答案:
   passage 1     1.     2.[C]     3.[A]     4.[C]     5.[C]
   passage 2     6.[A]     7.[D]     8.[D]     9.[C]    10.[C]
   passage 3    11.[C]    12.[A]    13.[C]    14.     15.[D]
   passage 4    16.[C]    17.[A]    18.[D]    19.    20.


Unit 4

Passage 1

Among the most spectacular visions in the sky is the glowing trail of a ball of frozen gases commonly known as comet. Comets are comprised of water, ammonia, methane, and carbon dioxide and are held together by small fragments of rock and metallic materials. They are created in outer space, beyond the reaches of our solar system. They are seen fairly infrequently but are nonetheless plentiful—in what time period.
Comets appear small but are in fact enormous. Some are larger than sun, but most are approximately the size of Jupiter. Many comets follow elongated orbits and are visible only when they are within Saturn’s orbit. As comets travel through the solar system toward the sun, solar energy vaporizes their gases, producing a blazing head, called the coma Some comets also develop a tail, Which can extend millions of miles. Despite their large scale, comets are believed to weigh relatively little probably less than one-billionth the earth’s mass.
Early astronomers found that as comets orbited the sun, a tail was created as solar energy forced the coma’s particles out. Today’s scientists have confirmed these beliefs, and have further identified two solar forces at work in the creation of a comet’s tail. Radiation pressure is responsible for pushing the dust particles away from the coma, and solar wind moves the coma’s ionized gases.
As the comet travels away from the sun, the tail begins to disappear due to condensation of the gases. Once these gases are separated from the comet, they are lost forever. As a result most comets survive only 100 times close orbits of the sun. When all comet’s gases have been released. all that is left of the comet is a bundle of stone and metal particles, which continues to orbit without a coma or tail.
Perhaps the most famous comet is Halley’s comet, which appears once every 76 years. The Chinese have recorded all of Halley’s appearances since 240 B.C. The comet’s 1910 sighting was remarkable for its clear visibility; it could be seen un daylight, and was observed to have developed a tail nearly one million miles long. The 1986 appearance of Halley’s comet was disappointing because it was extremely difficult to see from the Northern hemisphere. However, Halley’s next sighting is expected to be spectacular in the year 2062. (386 words  可读性:49.9  难度:10.4级)
1.  The author will probably agree to which of the following statements?
   [A] Comets are not often seen because there are only a few in the space.
The components of a comet come from the particles of the solar system.
[C] Not all of the comets can be observed near Saturn’s orbit.
[D] Most of the comets are seen once every 76 years.
2.  The comet’s coma is developed when
   [A] the comet is between Jupiter and Saturn.
the comet’s orbit travels through the solar system.
[C] the comet’s gases are steamed into vapor.
[D] the comet is in the Saturn’s orbit.
3.  Which of the following is NOT true about the creation of the tail of the comet?
   [A] The tail is created out of the coma’s particles.
One condition of the creation of the tail is the comet orbits the sun.
[C] The comet must travel towards the sun.
[D] The functioning solar force in creating the tail is gravitation.
4.  When the comet has finished its travel through the solar system
   [A] it will never come back again.
it will not be called a comet.
[C] its coma and tail will be lost.
[D] all that is left will become a meteor.
5.  It can be suggested that
   [A] the appearance of Halley’s comet can only be observed in one place.
people must observe comets only at night.
[C] observing Halley’s comet had long interested the Chinese astronomers.
[D] the appearance of a comet is always satisfactory.
词汇: ammonia 氨 elongated 延长的 ionized 电离出的 Jupiter 木星 Saturn 土星

passage 2

What do cats’ eyes and highway reflector signs have in common? They both act as retro-reflectors.
Almost everyone has noticed the eerie, frightening appearance of animals at night, as their eyes seem to glow in reflected light. One may speculate that just as reflective highway signs alert the motorist to many road hazards, so early man was often warned of danger by light from his campfire reflected in the eyes of lurking predators. In both cases, the light is retro-reflected, giving it a particularly bright appearance.
The principles of retro-reflection have been within the last sixty or seventy years, whit the widespread use of the automobile, that this area of optics has received important commercial application. Today, there are many uses for retro-reflective materials. They are purchased in very large quantities by government agencies for use on highways. They are applied to bicycles and motor vehicles to make them more visible at night. And they assist pilots by improving the visibility of runway markers.
Some users for retro-reflective materials are not related to safety. For example, these materials play a key role in helping railroads locate and make maximum use of their rolling stock. Color-coded retro-reflective numerals, strips, of dots applied to the side of railway cars identify the cars by their individual inventory numbers and by the particular type of car. Optical scanners “read” and record this information as the train passes by, thus making it possible for railroad management to locate an individual car or to determine the distribution of boxcars throughout the rail network.
Large sums are spent on retro-reflective materials, but purchasers have often had trouble deciding which specific materials will give the optimal performance. Performance is of concern not only from the standpoint of the brightness of reflected light, but also from the standpoint of its color. It is important, for example, that the yellow hues used in warning signals be of consistent color quality on all road signs so the driver can rely on color, as well as shape, to discern the sign’s meanings.
Seven colors—silver, blue, yellow, red, green, brown, and orange—are currently used for marking the interstate highway system. Since 1971, the National Bureau of Standards has been working to develop instrumental test methods that could be used to evaluate the performance of retro-reflective materials and aid in the preparation fo specifications for their purchase. (403 words  可读性:39.8  难度:12级)
6.  This article is mainly about
   [A] technology in photoelectric cells.
retro-reflection and tropism in animals and insects.
[C] natural and man-made retro-reflection.
[D] how to manufacture retro- reflective materials.
7.  Which can NOT be proved by the author according to the description of the retro-reflection?
   [A] Retro-reflective materials are used mainly to avoid traffic accidents.
Retro-reflective materials are abundantly used in traffic and transportation.
[C] The important value of retro-reflection owes to the invention of motor vehicles.
[D] The more advanced science and technology is, the more can be learned from nature.
8.  The article suggests that
   [A] the government agencies are developing inexpensive retro-reflectors.
the government manufactures retro-reflective devices.
[C] the government buys retro-reflective materials in large numbers.
[D] the government agencies export large quantities of retro-reflective materials.
9.  We can conclude from this article that
   [A] retro-reflective materials can be purchased at most hardware stores.
retro-reflective materials give off light in the dark.
[C] retro-reflective materials will be used up soon.
[D] the use of retro-reflective materials is becoming wider and wider.
10.  According to the passage, a difficult task facing the National Bureau of Standards probably is.
   [A] how to examine the properties of the retro-reflective materials.
how to budget for the purchase of the following year.
[C] how to integrate the warning signals into one consistent color.
[D] how to differentiate the signals’ meanings.
词汇: eerie 引起恐惧的 lurking 潜伏的 predators 猛兽 discern 分辨出


passage 3

Historians have only recently begun to note the increase in demand for luxury goods and services that took place in eighteenth-century England. McKendrick has explored the Wedgwood firm’s remarkable success in marketing luxury pottery; Plumb has written about the booming of provincial theaters, musical festivals and children’s toys and books. While the fact of this consumer revolution is hardly in doubt, three key questions remain: Who were the consumers? What were their motives? And what were the effects of the new demand for luxuries?
An answer to the their of these has been difficult to obtain. Although it has been possible to infer from the goods and services actually produced what manufacturers and servicing trades thought their customers wanted, only a study of relevant personal documents written by actual customers will provide a precise picture of who wanted what. We still need to know how large this consumer market was and how far down the social scale the consumer demand for luxury goods penetrated. With regard to this last question, we might note that Thompson, while rightly restoring laboring people to the invasion of capitalist consumerism in general: for example, laboring people in eighteenth-century England readily shifted from home-made beer to standardized beer produced by huge, heavily capitalized urban manufacturers.
To answer the question of why consumers became so eager to buy, some historians pointed to the ability of manufacturers to advertise in a relatively uncensored press. This, however, hardly seems sufficient answer. McKendrick favors a Veblen model of consumption stimulated by competition for status. The “middling sort” bought goods and services because they wanted they wanted to follow fashions set by the rich. Again, we may wonder whether this explanation is sufficient. Do not people enjoy buying things as a form of self-satisfaction? If so, consumerism could be seen as a product of the rise of new concepts of individualism and materialism, but not necessarily of the stiff competition.
Finally, what were the consequences of this consumer demand for luxuries? McKendrick claims that it goes a long way toward explaining the coming of the Industrial Revolution. But does it? What, for example, does the production of high-quality pottery and toys have to do with the development of iron manufacture or textile mills? It is perfectly possible to have the psychology and reality of a consumer society without a heavy industrial factor. (407 words  可读性:39.0  难度:12级)
11.  In the first paragraph, the author mentions McKendrick and Plumb in order to
   [A] contrast their views on the subject of luxury consumerism.
give examples of historians who have noticed the fact of the growing consumerism.
[C] indicate the inadequacy of historiographical approaches to 18th century English history.
[D] compare the two historians’ interests in luxury goods and services.
12.  To have a thorough understanding of the consumer market in 18th century English
   [A] possible by a study of the sales records.
      possible by a study of the documents by the relevant producers.
      [C] almost impossible since there are no relevant documents.
[D] not an easy task.
13.  According to Thompson, laboring people in eighteenth-century England were
   [A] curious about the luxuries.
ready to try the luxury goods and services.
[C] hostile to the capitalist consumerism.
[D] too stubborn to change their consumption custom.
14.  According to the passage, a Veblen model of consumption has been used to
   [A] explain the motivation of consumers to buy luxury goods.
investigate the extent of the demand for luxury goods.
[C] classify the kinds of luxury goods desired by consumers.
[D] prove that consumerism is a product of individualism.
15.  The author probably agrees with the statement that
   [A] the demand for luxuries for luxuries was a major factor in the coming of the Industrial Revolution.
the Industrial Revolution exploited the already existing demand for luxuries.
[C] the demand for luxuries may have helped bring about the Industrial Revolution.
[D] there is no reason to believe that the Industrial Revolution was directly driven by the demand for luxuries.
词汇:provincial 地方上的 uncensored 未经审查的

passage 4

American federalism has been described as a neat mechanical theory. The national government was said to be supreme in certain areas of governmental concern, such as the regulation of interstate commerce. State governments were said to be supremacy in certain other areas, such as regulation of intrastate commerce and exercise of the police power. One writer has described this as the “layer cake” concept of American federalism. In the top layer are nearly compacted all the powers of the national government; in the bottom layer are found the separate and distinct functions and powers of state governments.
How nice it would be if the American federal system could be so easily and conveniently analyzed. But Professor Martin Grodzins of the University of Chicago has gone on to describe federalism in practice as more like a marble cake, with an intermingling of functions, than like a layer cake, with functions separate and distinct. This intermingling can be seen best, perhaps, by examining the example of railroad traffic. If it crosses a state line, it constitutes interstate commerce, coming under commerce, coming under control of the national government. Rail shipments originating and ending within a single state constitute intrastate commerce, thus—the theory tells us—falling under regulation of state government. However, both the interstate and intrastate shipments may have moved over the same rails. In this simple example, one might easily read the urgent necessity for close cooperation between state and national governments. This need has not gone unrecognized by administrators of governmental programs at the state, local, and national levels.
Nonetheless, national and state interests often conflict in the political arena. Pressures may be brought to bear on state legislators which differ from those felt by members of the national Congress. Disagreement over the proper division of powers between states and the national government often lies beneath a conflict of interests. But no “best” formula has been discovered for been discovered fro drawing a dividing line between state powers and national powers.
The men who wrote the United States Constitution did the best they could in the face of circumstances which confronted them at the time. The state-national power dispute has raged persistently ever since. What are “states’ rights”? It is obvious that, throughout United States history, “states’ right” has arisen repeatedly as the painful cry of any interest which felt is was being treated unsympathetically at a given moment by the national government. The source of the cry would seem to depend on whose ox is being wounded. (413 words  可读性:42.6  难度:11.7级)
16.  What would probably be the basic problem in establishing clear guidelines related to interstate commerce?
   [A] As the power of the federal government increases, the power of the states decreases.
State and national governments have not effectively cooperated in defining their areas of control.
[C] Federal authorities have abused their constitutional power to regulate commerce.
[D] The rapid proliferation of transportation networks has undermined federal authority.
17.  An ideal federalism is described as
    [A] dividing the powers of he federal government into three distinct branches.
a system that provides for continuous continuous checks on federal authority.
[C] a system that limits central government power.
[D] a system in which the state and federal governments have distinct functions.
18.  The author implies that modern federalism
   [A] has limited the states’ police powers.
can be categorized as a distinct two-tier system of government control.
[C] is best explained as a system with mixed functions.
[D] has not recognized the legitimate need for states’ rights.
19.  Since there is no clear-cut formula for dividing state and national powers
   [A] state governments have been forced to delegate certain functions to the federal government.
the conflict of interests between states and the national government is inevitable.
[C] interstate cooperation is primarily a thing of the past.
[D] the role of the state has not been significantly altered.
20.  According to the passage, the state-national power dispute
   [A] has been violent in American history.
results from the incompetence of the American Constitution makers.
[C] exists because the national government has been unsympathetic.
[D] has caused a lot of people’s death in fighting.
词汇: intermingling 混合
答案:
passage 1     1.[C]     2.[C]     3.[D]     4.[C]     5.[C]
passage 2     6.[C]     7.[A]     8.[C]     9.[D]    10.[A]
passage 3    11.    12.[D]    13.[C]    14.[A]    15.[D]
passage 4    16.    17.[D]    18.[C]    19.    20.[A]

Unit 5

Passage 1

Schooling of fish is one of the most familiar forms of animal social behavior, but until recently it was little understood, possibly because of the impossibility of observing minute changes in the position and the velocity of fish in a school under natural conditions. The fact that over the ages a great many species of fish have developed the ability to congregate in schools suggests that the behavior offers a considerable evolutionary survival advantage aver those species that do not form schools. How the school is formed and maintained, however, is only beginning to be understood.
One of the most persistent misconceptions about fish schools is that have a regular geometric form. Our work shows that the structure is a loose and probabilistic one, and that it results from each fish applying a few simple rules. The first rule is that for each species there is a characteristic minimum approach distance within whish neighbors do not come. The distance is usually about three-tenths of a body length. The minimum approach distance is not the distance that is generally maintained within the fish in a school, however. In each species there is a typical preferred distance to the nearest neighbor, which is usually about one body length. The spatial relations among the fish in a school change constantly as the fish adjust their speed and direction. For this reason the distance to the nearest neighbor is not uniform, even for a single fish. The preferred distance is a statistical abstraction, found by averaging the actual distances over a long period.
Fish of a particular species also tend to keep their nearest neighbor at particular angle with respect to their body axis. Like the preferred distance, the preferred angle is a statistical quantity. At any given moment only a few fish may have their nearest neighbors at a preferred angle, but over the long period the preferred angle dominates. One useful measure of the degree of a school is the average ratio between the distance to the nearest neighbor and the distance to the second nearest neighbor. The closer the ratio to 1, the more uniform the structure. In a cubic lattice the ratio is exactly 1. The ratio varies considerably among species. For herring it is about 1. 1,for pollack 1. 3, and for cod 1.5. A ratio of 1.6 would be observed if the fish in the school took positions at random. (402 words  可读性:49.1  难度:11.1级)
1.  The author’s observation that schooling is an evolutionary development suggests that]
    [A] schooling of fish is a specific learned skill.
fish that do not swim in schools will become extinct.
[C] fish that do not swim in schools spend all their time in formation.
[D] schooling provides more chance of survival.
2.  It can be inferred that the author is now able to stud schooling in fish because
   [A] he employed a new method for observing and measuring small changes in school formation.
the author proved that evolution is responsible for the longer periods of time.
[C] new technology makes underwater study possible.
[D] new developments in statistical theory make the prediction of schooling possible.
3.  By “probabilistic” (para.2) the author means that
    [A] his conclusions are uncertain.
the structure of a school forms somewhat by chance.
[C] there are statistical methods for determining how a school will form.
[D] even species that usually swim in schools occasionally break formation.
4.  All of the following are behavioral rules used by fish in the school EXCEPT
   [A] the characteristic minimum approach distance.
the typically preferred distance to the nearest neighbor.
[C] the depth at which the school swims.
[D] the speed and direction of movement of the school.
5.  We can infer from the passage that
   [A] herring has a more uniform structure of school than cod and pollack.,
cod has a more uniform structure of school than herring and pollack.
[C] the school of herring has a regular geometric form.
[D] the structures of various fish schools have no great difference.
词汇: lattice 格子框架 herring 鲱鱼 Pollack 绿鳕 cod 鳕

passage 2

There is an alarming possibility that our American economy is moving in the direction of what some people call a two-tier society: a large population of people with middle-class or higher incomes and values, with a considerable increase at the top, and a large of people who have been economically and culturally uncoupled from the main society.
What’s most alarming is that the ladder that has connected the bottom to the top is mow missing some important cross-bars. There were certain industries, like the steel and auto industries, that provided more or less continuous ladders into secure, unionized, better-paid jobs. But now these industries have been seriously put in danger, and their place as employers has been replaced by what I call the McDonald’s employers. More people work for McDonald’s than work for U.S. Steel, but McDonald’s has no ladders. The problem is serious.
A great many economists, myself included, feel uneasy about the fact that 70 percent of the economy does what is called service work and only 30 percent does what is called goods-related work. New technology keeps entering the economy and bringing employment into disorder. When you look back at how the American economy developed, you see a migration off the farm into the factory and out of the factory into the office. The main push has come from technology. There has been relatively little new machinery to push people out of the office, but that’s changing now. If the computer creates jobs in the office, people out of work, I don’t know where they are going to go.
Personally, I think American optimism is in a very severe challenge. We have always considered ourselves virtually to have a right to be number one in the world. But of course we don’t have any such right or assurance. And we have to be patiently prepared for the unsettling fact that we are number two, or three, or our self-image.
In the next 20 years the government will have to take active steps in providing work and income for the bottom one third of the population. The government unwillingly provides some sort of income, but it doesn’t provide work.
And work is essential for people’s self-respect and also for the building of many kinds of basic frameworks that are needed in the country. (448 words  可读性:54.4  难度:10.7级)
6.  Among those that are going downhill in the United States is
   [A] McDonald’s.
U. S. Steel.
[C] the service sector.
[D] industries.
7.  In the opinion of the author, the main cause of farm-factory-office migration is
   [A] the technology of computer.
the technology policy.
[C] new technology in economy.
[D] unemployment.
8.  The author thinks that
   [A] American optimists should be criticized.
America’s leading position in the world is endangered.
[C] Americans have a right to be number one in the world.
[D] Americans are prepared for a big blow to their self-image.
9.  What the author worries about is
   [A] that new technology may push people out of the office.
that there will be no service work for people.
[C] that the government is unwilling to help people at the bottom.
[D] that the service sector will increase in size.
10.  It can be inferred that
   [A] the government will provide more work for people at the bottom.
there will be more goods-related work for people.
[C] it will be much easier for people to get employment in the service sector.
[D] it will be more difficult for people to rise from the bottom.

Passage 3

How exactly, does science work? How do scientists go about “doing” science? Ordinarily we think science proceeds in a straightforward way. Ideally scientists make observations, formulate hypotheses, and test those hypotheses by making further observations. When there is a difference between what is observed and what is predicted by the hypothesis, the hypothesis is revised. Science proceeds in this way, which is a gradual method of finding the best fit between observation and prediction.
But this idealized version of how one “does” science is na?ve. Although science demands proof that observations made by one observer be observable by other observers using the na?ve. Although science demands proof that observations made by one observer be observable by other observers using the same methods, it is by no means clear that, even when confronted with identical phenomena, different observers will report identical observations. And it is most certain that, even if the same observations are made, the conclusions as to the meaning of the observations frequently differ.
The fact is that all of us, scientists included, see differently. Variations in human perception are well known and have been studied extensively. Distortions in perception are frequently seen among observers, even though they may be in identical settings viewing identical phenomena.
A documented misperception from history can be found in the experience of Darwin. His ship, Beagle, after anchoring off the Patagonian coast, sent off a landing party in small row-boats. Amazingly, the Patagonian natives watching from shore were blind to the Beagle, but could easily see the tiny rowboats! They had no prior experience of huge sailing ships, but small rowing vessels were an everyday part of their life. Rowboats fit their model of the world and huge ships did not. Their model determined their perceptions.
Our idea that science proceeds on an utterly objective and straight-forward basis ignores the distortions of reality imposed by our own perceptual apparatus. In many cases we see what we have been trained to see, what we are used to seeing. If a subject is fitted with special glasses that are designed to invert the visual field, at first the subject sees everything upside down. After a period of time, as the glasses continue to be worn, a correction is made by our perceptual mechanism and the image is reversed, so that the world once again appears as it is. (375 words  可读性:49.6  难度:10.5级)
11.  According to the passage when observation can’t prove the hypothesis in scientific work
[A] scientists can do nothing but revise the hypothesis.
scientists will certainly make another prediction.
[C] the hypothesis must be changed.
[D] the hypothesis is not necessarily false.
12.  Which of the following cannot be concluded from paragraph 2?
   [A] When people face the same phenomena, they will have the same observations.
Variations in observation and conclusion are related to human perception.
[C] People using the same methods of observation can still make different observations.
[D] People often reach different conclusions even when they make the same observations.
13.  According to the author, the distortions of reality
   [A] are related to the method of observation.
can never be avoided.
[C] are caused by a human perceptual mechanism.
[D] can easily be corrected.
14.  According to the passage, human perceptual mechanism
   [A] can never be changed.
provides objective and straightforward way for observation.
[C] can provide some sort of correction on the basis of experience.
[D] always distorts reality.
15.  What is the focus of the passage?
   [A] The research methods used by scientists.
Observation and human perception variation.
[C] The relation between hypothesis and observation.
[D] The human perceptual mechanism.

Passage 4

Why does the Western movie especially have such a hold on our imagination? Chiefly, because it offers serious insights into the problem of violence such as can by found almost nowhere else in our culture. One of the well-known peculiarities of modern civilized opinion is its refusal to acknowledge the value of violence. This refusal is a virtue, but like many virtues it involves a certain deliberate blindness and it encourages hypocrisy. We train ourselves to be shocked or bored by cultural images of violence, and our very concept of heroism tends to be a passive one: we are less drawn to the brave young men who kill large numbers of our enemies than to the heroic prisoners who endure torture without surrendering. And in the criticism of popular culture, the presence of images of violence is often assumed to be in itself a sufficient ground for condemnation.
These attitudes, however, have not reduced the element of violence in our culture but have helped to free it from moral control by letting it take on the air of “liberation”. The celebration of violence is left more and more to the irresponsible. The gangster movie, with its numerous variations, belongs to a cultural “underground” which praises violence and sets it against all our higher social attitudes. It is a more “modern” style than the Western movie, perhaps even more profound, because it confronts industrial society on its own narrow logic. But it is anti-social, resting on fantasies of irresponsible freedom. If we are brought finally to concede to the denial of these fantasies, it is only because they have been shown to be dangerous, not because they have given way to higher values of behavior.
In war movies, certainly, it is possible to present violence within a framework of responsibility. But there is the disadvantage that modern war is a co-operative enterprise in which violence is largely impersonal and heroism belongs to the group more than to the individual. The hero of a war movie is most often simply a leader, supposed to be brave, you are supposed to get the job done and stay alive (this too, of course, is a kind of heroic posture, but a new—and “practical”—one). At its best, the war movie may represent a more civilized print of view than the Western, and if it were not continually spoiled by ideological sentimentality we might hope to find it developing into a higher form of drama. But it cannot supply the values we seek in the Western. (446 words  可读性:41.4  难度:12级)
16.  The author considers gangster movies to be
   [A] subversive.
incredible.
[C] illogical.
[D] artistic.
17.  What’s the result of modern civilized opinion about violence?
   [A] the positive gain of a more emancipated attitude.
a less responsible attitude to violence.
[C] violent celebration of various kinds.
[D] some small reduction in violence.
18.  According to the author, the concept of heroism in war films is expressed in terms of
    [A] tremendous personal courage.
an indifference to violence.
[C] adopts a more serious attitude toward violence than the Western film.
[D] is a higher form of art.
19.  According to the passage, the war movie
   [A] tends to be sentimental.
presents violence in an appropriate way.
[C] adopts a more serious attitude toward violence than the Western film.
[D] is a higher form of art.
20.  The author probably agree with the opinion that
   [A] the presence of images of violence should not be denied completely.
violence is a symbol of freedom in modern societies.
[C] impersonal violence should be not be presented in movies.
[D] violence should be praised in movies.
词汇: hypocrisy 虚伪 concede承认

答案:
passage 1     1.[D]     2.[A]     3.     4.[C]     5.[A]
passage 2     6.     7.[C]     8.     9.[A]    10.[D]
passage 3    11.[D]    12.[A]    13.[C]    14.[C]    15.
passage 4    16.[A]    17.    18.[C]    19.[A]    20.[A]
   
Unit 6

Passage 1

Clinicians at a recent psycho-analytic conference brought forth interesting evidence that guilt, far from being the psychic impediment generally conceived, has the potential to inspire creativity, and enhance sensitivity.
Tests of prison inmates have shown significantly low scores on guilt scales, measured by psychologist-researcher Donald L. Mosher. The Mosher scales measure the tendency to feel guilt in three forms: sex guilt, hostility guilt, and general guilt, called morality conscience. Prisoners who had committed sex crimes scored low on sex guilt, those who were imprisoned for violent crimes scored low on hostility guilt; those incarcerated for crimes against property scored low on morality conscience.
Other studies conducted in the armed forces corroborate the findings that men accused of brutality toward those they command feel little or no sense of remorse or guilt, but tend do defend vigorously the “connectness” of the actions.
That guilt can be a lonely and lacerating burden has long been known. The ancient Greeks understood the redemptive feelings and cathartic benefits of watching the tragic. hero struggle with guilt. Hamlet plots to “catch the conscience of the King” O’ Neill recreates the ancient themes and adds to them contemporary guilt. The Judeo Christian ethic transmits this heavy burden, commencing with “original sin” and continuing with the need for confession and atonement.
Although in the past lots of psycho-analysts, joined by many recent authors, seem to have been dedicated to eliminating the sense of guilt, some clinicians hold that guilt is the necessary price for socialization.
Still others agree with Dr. Karl Menninger in the value of appropriate, or rational, guilt, and feel that a prime objective of therapeutic intervention should be to help the patient differentiate between guilt feelings that are unwarranted and unfounded, based perhaps on distorted perceptions of past occurrences, and those which are well-founded responses to real situations. The child, it is felt, should not be made to feel guilty about exploring his body, just as the adult should not be ashamed of his or her sexuality. But this freedom must not be viewed as license. When the individual’s desires or needs can be fulfilled without coming into conflict with societal needs, people should not feel guilty.
It is this new approach, this compromise, which we find surfacing in twentieth-century literature. Herzog and Willy Loman battle their needless guilt, and their experiences help us to cope. (390 words  可读性:42.6  难度:12级)
1.  The best title for this passage is
   [A] How to Help Guilty Persons.
A New Approach to Guilt.
[C] Guilt—A good Master.
[D] Guilt—Nothing but A Burden.
2.  We can infer from the passage that
   [A] guilt serves to punish the person who commits offense against individuals or society.
some people never suffer from the feeling of guilt.
[C] unfortunately, many people who have been judged guilty feel little guilt.
[D] sensitive people are guiltier than those who are less sensitive.
3.  The author mentions Hamlet and O’Neill to illustrate that
   [A] the ancient Greeks understood how to cope with guilt.
guilt is a common theme in Western literature.
[C] guilt has been regarded as a harmful burden for a long time.
[D] guilt can sometimes benefit human beings.
4.  According to the passage, a crucial concern of the therapist should be to
   [A] help patients evaluate appropriate and inappropriate responses of guilt feelings.
help the patient conquer guilt.
[C] direct the patient to understand that guilty feelings are unwarranted and unfounded.
[D] help the patient fulfill his needs without social interference.
5.  What is the new idea in the 20th century literature?
   [A] Guilt feelings are necessary for modern society.
Efforts must be made to eliminate the sense of guilt.
[C] Guilt is a psychic impediment.
[D] People should get rid or needless guilt.
词汇: impediment 障碍 incarcerated 监禁 corroborate 证实 lacerating 使苦恼 cathartic 感情净化作用 redemptive 救赎 atonement 赎罪 therapeutic治疗的

passage 2

One of the many theories about alcoholism is the learning and reinforcement theory, which explains alcoholism by considering alcohol drinking as a reflex respones to some stimulus and as a way to reduce an inner drive state such as fear or anxiety. Characterizing life situations in terms of approach and avoidance, this theory holds that persons tend to drawn to pleasant situations or repelled by unpleasant ones. In the latter case, alcohol drinking is said to reduce the tension or feelings of unpleasantness and to replace them with the feeling of pleasure generally observed in most persons after they have consumed one or more dinks.
Some experimental evidence tends to show that alcohol reduces fear in an approach-avoidance situation. Conger trained one group of rats to approach a food goal and trained another group to avoid electric shock. After and injection of alcohol the pull away from the shock was measurably weaker, while the pull toward food was unchanged.
The obvious troubles experienced by alcoholic persons appear to contradict the leaning theory in the explanation of alcoholism. The discomfort, pain, and punishment they experience should presumably discourage the alcoholics from drinking. The fact that alcoholic persons continue to drink in the face of family discord, loss of job, and illness is explained by the proximity of the drive reduction to the consumption of alcohol; that is, alcohol has the immediate effect of reducing tension while the unpleasant consequences of drunken behavior came only later. The learning pattern, therefore, favors the establishment and repetition of the resort to alcohol.
In fact, the anxieties and feelings of guilt caused by the consequences of excessive alcohol drinking may themselves become the signal for another time of alcohol abuse. The way in which the desire for anther drink could be caused by anxiety is explained by the process of stimulus generalization: Conditions or events occurring at the time of reinforcement tend to acquire all the features of stimuli. When alcohol is consumed in association with a state of anxiety or fear, the emotional state itself takes on the properties of a stimulus, thus triggering another time of drinking.
The role of punishment is becoming increasingly important in explaining a cause of alcoholism based on the principles of learning theory. While punishment may serve to suppress a response, experiments have shown that in some cases it can serve as a reward and reinforce the behavior. Thus if the alcoholic person has learned to drink under conditions of both reward and punishment, either type of condition may trigger renewed drinking. (425 words  可读性:36.1  难度:12级)
6.  The main purpose of the passage is to
   [A] introduce some theories about alcoholism.
help alcoholics and others to understand the cause of alcoholism.
[C] explain the application of a psychological approach to alcoholism.
[D] show the most effective new treatment of alcoholism.
7.  The description of Conger’s experiment with two groups of rats was intended to
   [A] show that alcohol drinking does not affect appetite.
confirm the findings of other academic researchers.
[C] show that alcohol minimizes fear.
[D] disprove learning and reinforcement theory.
8.  We can learn form Para.3 that
   [A] the learning theory contradicts itself in some fields.
drinking alcohol can solve the problem of family discord.
[C] drive reduction appear first after drinking alcohol.
[D] disprove learning and reinforcement theory.
9.  The author provides enough information to answer the question of
   [A] why alcoholics continue to drink the unpleasant consequences.
how Conger explained the behavior of alcoholic in terms of shock therapy.
[C] under what circumstances an alcoholic benefits from anxiety attacks.
[D] which is the best treatment of alcoholism.
10.  It can be inferred from the passage that
   [A] the behavior of alcoholics contradicts the approach-avoidance theory.
the behavior of most alcoholics proves the learning theory.
[C] punishment may become the stimulus for another time of drinking.
[D] frequent excessive drinking makes alcoholics indifferent to punishment.
词汇: proximity 接近

passage 3

Some people would say that the Englishman’s home is no longer his castle; that is has become his workshop. This is partly because the average Englishman is keen on working with his hands and partly because he feels, for one reason or another, that he must do for himself many household jobs for which, some years ago, he would have hired decorators’ costs have reached a level which makes them prohibitive for labour has meant that builders’ and decorators’ costs have reached a level which make them prohibitive for horse-proud English people of modest means. So, if they wish to keep their houses looking bright and smart, they have to tackle some of the repairs and decorating themselves. As a result, there has grown up in the post-war years what is sometimes referred to as the “Do-It-Yourself Movement”.
The “Do-It-Yourself Movement” began with home decorating but has since spread into a much wider field. Nowadays there seems to be very few things that cannot be made by the “do-it-yourself” method. A number of magazines and handbooks exist to show hopeful handymen of all ages, just how easy it is to build anything from a coffee table to a fifteen foot (4.5 metres) sailing boat. All you need, it seems, is a hammer and a few nails. You follow the simple instructions step-by-step and, before you know where you are, the finished article stands before you, complete in every detail.
Unfortunately, however, it is not always quite as simple as it sounds! Many a budding “do-it-yourself” has found to his cost that an amateur cannot learn a skilled craftsman’s job overnight. How quickly one realizes, when doing it oneself, that a job which takes the skilled man an hour or so to complete takes the amateur handyman five or six at least. And then there is the question of tools. The first thing the amateur learns is that he must have the right many people have come to the conclusion that the expense of paying professionals to do the work is, in the long run, more economical than “doing it oneself”. (378 words  可读性:63.5  难度:9.3级)
11.  The most important reason why many Englishmen now “do it themselves” is that
   [A] they like working with their hands.
they feel that they must do for themselves many household jobs.
[C] people with modest means want to save money.
[D] they have no financial resources.
12.  The “Do-It-Yourself Movement” developed at a time when
   [A] the Second World War grew up unfavourably.
the cost of labour was very high for the average English man.
[C] it was difficult to hire professional help.
[D] the Englishman’s home was no longer his castle.
13.  The author mentions the magazines and handbooks on making things to mean that
    [A] they play an important role in the “Do-It-Yourself Movement”.
how popular the “Do-It-Yourself Movement” is.
[C] working with hands is a thing easy to say but difficult to perform.
[D] the instructions in the books are easy to follow.
14.  Which of the following is the disadvantage of “Do-It-Yourself”?
   [A] It is energy-consuming work.
It is a waste of time.
[C] It makes people nervous.
[D] People have to tackle some of the repairs themselves.
15.  The passage points out at the end that
   [A] the cost of labour would go down.
many British people would prefer to pay the high cost of labour.
[C] many British would do the decorating themselves.
[D] the “Movement” world spread into a much wider field.
词汇: budding 新获得成功的

passage 4

As is the case in many cultures, the degree to which a minority group was seen as different from the characteristics of the dominant majority determined the extent of that group’s acceptance. Immigrants who were like the earlier settlers were accepted. The large numbers of immigrants with significantly different characteristics tended to be viewed as a threat to basic American values and the American way of life.
This was particularly true of the immigrants who arrived by the millions during the late nineteenth and early twentieth centuries. Most of them came from poverty-stricken nations of southern and eastern Europe. They spoke languages other than English, and large numbers of them were Catholics or Jews.
Americans at the time were very fearful of this new flood of immigrants. They were afraid that these people were so accustomed to lives of poverty and dependence that they would not understand such basic American values as freedom, self-reliance and competition. There were so many new immigrants that they might even change the basic values of the nation in undesirable ways.
Americans tried to meet what they saw as a threat to their values by offering English instruction for new immigrants and citizenship chasses to teach them basic American beliefs. The immigrants, however, often felt that their American teachers disapproved of the traditions of their homeland. Moreover, learning about American values gave them little help in meeting their most important needs such as employment, food, and a place to live.
Far more helpful to the new immigrants were the “political bosses” of the larger cities of the northeastern United States, where most of he immigrants first arrived. Those bosses saw to many of the practical needs of the immigrants and were more accepting of the different homeland traditions. In exchange for their help, the political bosses expected the immigrants to keep them in power by voting for them in elections.
In spite of this, many scholars believe that the political bosses performed an important function in the late nineteenth and early twentieth centuries. They helped to assimilate large numbers of disadvantaged white immigrants into the larger American culture. The fact that the United States had a rapidly expanding economy at the turn of the century made it possible for these new immigrants, often with the help of the bosses, to better their standard of living in the United States. As result of these new opportunities and new rewards, immigrants came to accept most of the values of the larger American and were in turn accepted by the great majority of Americans. For white ethnic groups, therefore, it is generally true their feeling of being a part of the larger culture, that is, “American” is much stronger than their feeling of belonging to a separate ethnic group—Italian, Polish, etc. (468 words  可读性:41.8  难度:12级)
16.  A minority group’s acceptance to the country was determined by
   [A] the difference they showed from the majority.
the time when they arrived at the new land.
[C] the background conditions they came belonged.
[D] the religious group to which they belonged.
17.  The immigrants’ flushing in was considered a threat to American value mainly because
    [A] the immigrants came from poverty-stricken nations of southern and eastern Europe.
the immigrants had been accustomed to poverty and dependence.
[C] the immigrants had different homeland traditions and other particular characteristics.
[D] the immigrants did not speak English.
18.  “Citizenship classes” (Para.4) were offered because Americans
   [A] wanted to help the immigrants to solve their practical needs.
would not accept any groups with different traditions.
[C] wanted the immigrants to deal with the threat to the American values.
[D] wanted the immigrants to learn about and to keep the American values.
19.  The political bosses helped the new immigrants for the main purpose of
   [A] showing off their political powers and advantages.
getting support in elections.
[C] assimilating the minority into the majority.
[D] showing their generosity.
20.  The living standards of the new immigrants were improved in the late nineteenth and early twentieth centuries mainly because
   [A] they kept the political bosses in power.
the political bosses gave them a lot of practical help.
[C] they had a much stronger feeling of being a part of the larger culture.
[D] there was a rapid growth in American economy at that time.
答案:
passage 1     1.     2.[C]     3.[C]     4.[A]     5.[D]
passage 2     6.     7.[C]     8.[C]     9.[A]    10.[C]
passage 3    11.[C]    12.    13.[C]    14.[A]    15.
passage 4    16.[A]    17.[C]    18.[D]    19.    20.[D]

Unit 7

Passage 1

This issue of the writer’s identity is one aspect of writing that makes it so complex a task. Because your written language is a from of communication divorced from your physical presence in most instances, who “you” are on the page is open to question. The written matter on the page isn’t “you”, and yet it does have some unique characteristics, personality, beliefs, or values. As readers, we cannot assume that the voice speaking to us through prose is identical to the actual author. The actual author of a piece of writing and the authorial voice that author has assumed in the writing are not necessarily, or even usually, the same.
This phenomenon has implications for you as a writer. It means that writing can be a liberating activity, one that frees you from your usual social self and voice. This is probably one of writing’s most appealing features, and may account for the vast numbers of people who enjoy writing for themselves, without the slightest intention of seeking publication or an audience beyond their own eyes. Awareness of writing on this level also carries a kind of responsibility with it, however. As a college student, you are asked to write with this understanding of writing. It’s not that instructors are uninterested in your personal opinion in your essays; rather, you are expected to understand writing on a more sophisticated or perhaps more professional level. In your essays, in most academic contexts, you will be expected to employ this notion of authorial voice, and to express your views whatever they are.
In most of your academic courses, college students are expected to have the skill of assuming different authorial voices to meet your different writing situations. It is perhaps human nature to assume that the reason one instructor gave you an A on an essay and another instructor gave you or liked what you had to say in the paper, and the other had personal or ideological reasons for grading the paper down—the instructor disagreed with you and so lowered the grade. But the issue may actually not be one of unfair or inconsistent standards; it may be an issue more related to your writing authorial voice. (388 words  可读性:49.1  难度:11.2级)
1.  The authorial voice is
   [A] the actual voice of the author.
the voice which speaks to readers through the writing.
[C] the voice the same author uses in his other writings.
[D] the written matter which the author employs on the very page.
2.  According to the author, many people enjoy writing for themselves because writing
    [A] can free one from any kind of social responsibilities.
often frees the person who writes from his usual social self.
[C] can make anyone who wants to be famous popular socially.
[D] can appeal to vast audience.
3.  According to the last paragraph, instructors
   [A] may be unfair to their students in some cases.
have inconsistent standards in grading students’ papers.
[C] usually emphasize more on personal ideas in a paper.
[D] usually emphasize more on the authorial voice used in a paper.
4.  We can infer that in writing papers it’s most important for college students to
   [A] assume appropriate authorial voices.
express their personal ideas clearly.
[C] have a good understanding of the instructors’ demands.
[D] know the instructors’ personal views beforehand.
5.  The best title for this passage might be
   [A] Writer’s Identity.
Actual Author.
[C] Liberating Activity.
[D] Authorial Voice.

Passage 2

When television is good, nothing is better. But when television is bad, nothing is worse. I invite you to sit down in front of your television set when your station goes on the air and stay there without a book, magazine, news-paper, or anything else to distract you and keep your eyes glued to that set until the station sings off. I can assure you that you will observe a vast wasteland. You will see a procession of game shows, violence, audience participation shows, formulas comedies about totally unbelievable families, blood and thunder, more violence, sadism, murder, Western badmen, Western goodmen, private eyes, gangsters and so on. And endlessly, commercials that scream flatter and offend. And, boredom. True, you will see a few things you will enjoy. But they are very, very few. And if you think I exaggerate, try it.
Is there no room on television to teach, to inform, to uplift, to stretch, to enlarge the capacities of our children? No room for programs to deepen the children’s understanding of children in other lands? No room for a children’s news show explaining something about the world for them at their level of understanding? No room for reading the great literature of the past, teaching them the great traditions of freedom? There are some fine children’s shows, but they are drowned out in the vast sea of cartoons and violence. Must these be your trademarks? Search your conscience and see whether you cannot offer more to our children whose future you guard so many hours each and every day.
There are many people in this county, and you must serve all of us. You are right when you say that, given a choice between a Western and a symphony, more people will watch the Western. I too like them, but a steady diet for the whole country is obviously not in the public interest. We all know that people would more often prefer to be entertained than stimulated or informed. But your duties are free to communicate ideas as well as to give relaxation. You must provide a wider range of choices, more diversity, more alternatives. It is not enough to serve the nation’s whims, you must also serve her needs. The people own the air. They own it as much in prime evening time as they do at 6o’clock in the morning. For every hour that the people give you—you owe them something. I intend to see that your debt is paid with service. (443 words  可读性:69.4  难度:7.2级)
6.  “The wasteland” (Para. 1) most probably describes
   [A] Western badmen and Western goodmen.
average television programs.
[C] television programs whit violence and murder.
[D] television shows with desert settings.
7.  According to the passage, average people would more often prefer
   [A] programs which can teach them.
programs which can uplift their spirits.
[C] programs for amusement.
[D] programs which can enlarge their capacities.
8.  The statement that “the people own the air” implies that
   [A] citizens have the right to insist on worthwhile television programs.
television should be socialized.
[C] the government may build something above present structures.
[D] since air is worthless, the author is telling his listeners that.
9.  It can be inferred that the author is telling his listeners that
   [A] they had better provide more informative programs than entertaining ones.
they should ban the formulas comedies.
[C] they had better mend their ways.
[D] the programs for children are worthless.
10.  The author’s tone of the passage is one of
   [A] firmness.
rage.
[C] hopelessness.
[D] disgust.
词汇: sadism 性虐待狂 whims 新想法

passage 3

Auctions are public sales of goods, conducted by an officially approved auctioneer. He or she asks the assembled crowd in the auction-room to make offers, or “bids”, for the various items on sale. He encourages buyers to bid higher figures, and finally names the highest bidder as the buyer of the goods. This is called “knocking down” the goods, for the bidding ends when the auctioneer bangs a small hammer on a table at which he stands. This is often set on a raised platform called a rostrum.
The ancient Romans probably invented sales by auction, and the English word comes from the Latin auction meaning “increase”. The Romans usually sold in this way the spoils taken in war; these sales were called “sub hasta” meaning “under the spear”, a spear being stuck in the ground as a signal for a crowd to gather. In the eighteenth and nineteenth centuries goods were often sold “by the candle”, a short candle was lit by the auctioneer, and bids could be made while it stayed alight.
An auction is usually advertised beforehand with full particulars of the articles to be sold and where and when they can be viewed by prospective buyers. If the advertisement cannot give full details, catalogues are printed, and each group of goods to be sold together, called a “lot”, is usually given a number. The auctioneer need not begin with Lot 1 and continue in numerical ordre. He may until he registers the fact that certain dealers are in the room and then produce the lots they are likely to be interested in.
The auctioneer must know fairly accurately the current market values of the goods he is selling, and he should be acquainted with regular buyers of such goods. He will not waste time by starting the bidding too low. He will also play on the rivalries among his buyers and succeed in getting a high price by encouraging two business competitors to bid against each other. It is largely on his advice that a seller will fix a “reserve” price, that is, a price below which, the goods cannot be sold. Even the best auctioneers, however, find it difficult to stop a “knock-out”, whereby dealers illegally arrange beforehand not to bid against each other, but nominate one of themselves as the only bidder. In the hope of buying goods at extremely low prices. If such a “knock-out” comes off, the real auction sale takes place privately afterwards among the dealers. (416 words  可读性:61.2  难度:9.8级)
11.  The end of the bidding is called “knocking down” because
   [A] the auctioneer bangs the table with a hammer.
the auctioneer knocks the rostrum down.
[C] the auctioneer finally names the highest bidder as the buyer of the goods.
[D] the auctioneer encourages buyers to bid higher figures.
12.  The Romans used to sell by auction
   [A] the worn-out weapons in war.
the property taken from the enemy.
[C] the spoilt goods taken in war.
[D] the spears.
13.  A candle is used to burn at auction sales
   [A] to keep the auction-room light.
to limit the time when offers could be made.
[C] as a signal for the crowd to gather in the eighteenth century.
[D] to keep the sales secret.
14.  The auctioneer may decide to sell the “lots” out of order because
   [A] he sometimes wants to confuse the buyers.
he wants to keep certain people waiting.
[C] he wants to reduce the number.
[D] he knows from experience that certain people will want to buy certain items.
15.  A “knock-out” is arranged to
   [A] keep the price in the auction-room low.
allow only one dealer to made a profit.
[C] increase the auctioneer’s profit.
[D] help the seller.
词汇: rostrum 讲台

passage 4

This is a skeptical age, but although our fait in many of the things which our forefathers fervently believed has weakened, our confidence in the curative properties of the bottle medicine remains the same as theirs. This modern faith in medicines is proved by the fact that the annual drug bill of the Health Services is mounting to astronomical figures and shows no signs at present of ceasing to rise. The majority of the patients attending the medical out-patients departments of our hospitals feel that they have not received adequate treatment unless they are able to carry home with them some tangible remedy in the shape of a bottle of medicine, a box of pills, or a small jar of ointment, and the doctor in charge of the department is only too ready to provide them with these requirements. There is no quicker method of disposing of patients than by giving them what they are asking for, and since most medical men in the Health Services are overworked and have little time for offering time-consuming and little-appreciated advice on such subjects as diet, right living, and the need for abandoning bad habits etc. the bottle, the box, and the jar are almost always granted them.
Nor is it only the ignorant and ill-educated person who has such faith in the bottle of medicine. It is recounted of Thomas Carlyle that when he heard of the illness of his friend, Henry Taylor, he went off immediately to visit him, carrying with him in his pocket what remained of a bottle of medicine formerly prescribed fro an indisposition of Mrs. Carlyle’s. Carlyle was entirely ignorant of what the bottle in his pocket contained, of the nature of the illness from which his friend was suffering, and of what had previously been wrong with his wife, but a medicine that had worked so well in one form of illness would surely be of equal benefit in another, and comforted by the thought of the help he was bringing to his friend, he hastened to Henry Taylor’s house. History does not relate whether his friend accepted his medical help, but in all probability by bid.
The great advantage of taking medicine is that it makes no demands on the taker beyond that of putting up for a moment with a disgusting taste, and that is what all patients demand of their doctors—to be cured at no inconvenience to the themselves. (407 words  可读性:33.3  难度:12级)
16.  According to the author, patients feel that they have been adequately treated only if they
    [A] can take home some medicine doctors prescribe.
have confidence in the curative properties of a bottle of medicine.
[C] are given careful check-ups by the doctor.
[D] are given some remedy.
17.  It can be learned from the passage that most doctors
   [A] feel morally bound to give patients advice about diet and right living.
are not glad to provide patients with their requirements.
[C] are confident in the curative properties of the bottle of medicine.
[D] are inclined to satisfy patients’ demand for tangible medicine.
18.  The word “them” (Last Sentence, Para .1) refers to
   [A] the doctors in the medical out-patients departments.
the medical men in the Health Services.
[C] patients.
[D] the bottle, the box and the jar.
19.  The anecdote about Thomas Carlyle is used to illustrate that
   [A] a remedy that will cure one illness is not necessarily good for another.
Thomas Carlyle was also an ignorant person who has faith in medicine.
[C] even educated people can be misled into believing that one can’t be cured by medicine.
[D] educated people may also have confidence in the bottle of medicine.
20.  The passage mainly tells us that
   [A] we are in a skeptical age.
people’s faith in the bottle of medicine has not weakened in the modern age.
[C] there are some problems in modern medical service.
[D] patients incline to impose their own will on doctors.
词汇: fervently 热烈地 ointment 药膏 indisposition 小病
答案:
passage 1     1.     2.     3.[D]     4.[A]     5.[D]
passage 2     6.     7.[C]     8.[A]     9.[C]    10.[A]
passage 3    11.[A]    12.    13.[C]    14.[D]    15.[A]
passage 4    16.[A]    17.[D]    18.[C]    19.[D]    20.
第五部分考研阅读笑傲江湖篇
—————最新考题全真测试
回复

使用道具 举报

1

主题

308

帖子

914

积分

中级战友

Rank: 3Rank: 3

精华
0
威望
45
K币
869 元
注册时间
2009-11-29
发表于 2010-8-3 11:10 | 显示全部楼层
回复一下 慢慢看
回复

使用道具 举报

6

主题

109

帖子

311

积分

一般战友

Rank: 2

精华
0
威望
30
K币
281 元
注册时间
2010-7-21
发表于 2010-8-4 13:04 | 显示全部楼层
楼主辛苦了
回复

使用道具 举报

91

主题

1950

帖子

1万

积分

开国大老

╄→我愛米蘭!v1p°

Rank: 5Rank: 5

精华
9
威望
2313
K币
9326 元
注册时间
2007-8-2
发表于 2010-8-4 13:24 | 显示全部楼层
嗯 楼主威武  比老大的完整了一些
真正的成功都跟做贼一样猥琐得偷偷摸摸。。。。
在重拾梦想的孤独里,猥琐而快乐的寂寞着。。。
回复

使用道具 举报

0

主题

40

帖子

66

积分

新手上路

Rank: 1

精华
0
威望
0
K币
66 元
注册时间
2010-6-6
发表于 2010-8-4 13:42 | 显示全部楼层

回复 楼主 newking2 的帖子

谢谢 楼主
位极人臣
回复

使用道具 举报

0

主题

78

帖子

104

积分

一般战友

Rank: 2

精华
2
威望
46
K币
58 元
注册时间
2010-7-2
 楼主| 发表于 2010-8-4 16:55 | 显示全部楼层
此贴 完全超越老大 还免币 为何 没人顶[em:15]
回复

使用道具 举报

0

主题

173

帖子

1689

积分

中级战友

Rank: 3Rank: 3

精华
0
威望
1078
K币
611 元
注册时间
2010-8-2
发表于 2010-8-4 18:25 | 显示全部楼层
好吧,假装我看完咧~~~~顶一下
回复

使用道具 举报

8

主题

456

帖子

2944

积分

中级战友

Rank: 3Rank: 3

精华
0
威望
788
K币
2156 元
注册时间
2010-4-10
发表于 2010-8-4 23:05 | 显示全部楼层
好贴留名
回复

使用道具 举报

11

主题

997

帖子

2712

积分

中级战友

Rank: 3Rank: 3

精华
4
威望
370
K币
2342 元
注册时间
2008-3-5
发表于 2010-8-4 23:37 | 显示全部楼层
回复

使用道具 举报

3

主题

102

帖子

370

积分

一般战友

文精野体

Rank: 2

精华
0
威望
10
K币
360 元
注册时间
2009-10-20
发表于 2010-8-4 23:51 | 显示全部楼层
这不是老妖那个付费帖吗?呵呵,抢了他生意啦!
一点精
二点灵
三点扬名四海
四点天下太平
回复

使用道具 举报

您需要登录后才可以回帖 登录 | 注册 人人连接登陆

本版积分规则   

关闭

您还剩5次免费下载资料的机会哦~

扫描二维码下载资料

使用手机端考研帮,进入扫一扫
在“我”中打开扫一扫,
扫描二维码下载资料

关于我们|商务合作|小黑屋|手机版|联系我们|服务条款|隐私保护|帮学堂| 网站地图|院校地图|漏洞提交|考研帮

GMT+8, 2024-3-29 18:29 , Processed in 0.203518 second(s), Total 9, Slave 8(Usage:11.5M, Links:[2]1,1_1) queries , Memcache On.

Powered by Discuz!

© 2001-2017 考研 Inc.

快速回复 返回顶部 返回列表
× 关闭